Text
                    ПЕРЕВОДНАЯ НАУЧНО-ПОПУЛЯРНАЯ ЛИТЕРА ТУРА

Йоханнес Леман
У ВЛЕК А ТЕЛЬНАЯ
МАТЕМАТИКА
liURZWElL \*4
DURCH МАТНЕ

Йоханнес Леман УВЛЕКАТЕЛЬНАЯ МАТЕМАТИКА Johannes Lehmann KURZWEIL BURCH MATHE
Johannes Lehmann KURZWEIL DURCH MATHE URANIA-VERLAG LEIPZIG JENA BERLIN
Йоханнес Леман УВЛЕКАТЕЛЬНАЯ МАТЕМАТИКА Перевод с немецкого Ю.А. Данилова Издательство „Знание" Москва 1985
ББК 22.1 Л 44 Научный редактор и автор предисловия — А. П. Савин. Перевод с немецкого 10. А. Данилова Леман И. Л 44 Увлекательная математика. Пер. с нем.— М.: Знание, 1985.— 272 с., ил. 50 к. 100 000 экз. Автор книги Йоханнес Леман, главный редактор научно-популярного математического журнала ГДР «Альфа», собрал в ней несколько сотен задач, среди которых и совсем свежие, и такие, возраст которых исчисляется тысячелетиями. Большое количество иллю- страций делает книгу особенно привлекательной для школьников, которым в первую очередь она и адре- сована. 1702010000—037 л ББК 22.1 Л 073(02)—85 26—84 51 © Urania-Verlag Leipzig, Jena, Berlin. Verlag fur popularwissenschaftliche Literatur, Leipzig, 1980. © Издательство «Знание», 1985 г. Перевод на русский язык, комментарии, предисловие
ПРЕДИСЛОВИЕ Когда мы перелистываем страницы науч- но-популярного журнала, то вольно или не- вольно взгляд, скользнув по иллюстрациям, останавливается на занимательных задачах. Необычность ситуации, неочевидность ответа на поставленный вопрос заинтриговывает нас, и мы начинаем нелегкий поиск пути, ведуще- го к решению задачи. Многие считают занимательные задачи средством для приятного времяпрепровожде- ния, отдыха, но если вдуматься, то становит- ся ясной их гораздо более важная роль. Не- сомненно, что именно занимательные задачи являются одним из самых мощных инстру- ментов развития человеческого интеллекта. Если человеку в течение жизни приходится, скажем, десяток раз оказаться в затрудни- тельном положении, выход из которого мож- но найти с помощью логических рассуждений, то задачи предоставляют ему такую возмож- ность сотни раз уже в детстве и юности, имен- но тогда, когда формируется его интеллект. Не зря люди передавали эти задачи устно и письменно из поколения в поколение. Одной из таких эстафетных палочек явля- ется и книга, которую вы держите в руках. Автор собрал в ней несколько сотен задач, среди которых и совсем свежие, и такие, воз- раст которых исчисляется тысячелетиями. Множество иллюстраций делает книгу осо- бенно привлекательной для школьников, ко- торым она в первую очередь и адресована. 5
Автор книги Йоханнес Леман, главный ре- дактор научно-популярного математического журнала ГДР «Альфа», считает, что наиболь- шую пользу занимательные задачи приносят детям 12—13 лет, поскольку до этого у них еще нет достаточного жизненного опыта и запаса знаний. Представляя книгу И. Лемана, хочется отметить чрезвычайно широкую палитру за- дач, включенных в сборник: от «чисто математических» до таких, которые, казалось бы, к математике не имеют прямого отноше- ния, например: «Как выглядит с изнанки шнуровка тапочек, изображенных на рисун- ке?» Не менее широк и спектр трудности за- дач: от совсем легких, решаемых «в уме» первоклассниками, до таких, которые могут заставить надолго задуматься и специалиста с высшим образованием. Правда, нужно от- метить, что для решения задач, как правило, достаточно знаний школьников 5—б-х клас- сов. Задачи проверяют не знания, а умение логически рассуждать, они учат этому умению. В заключение хочется отметить большое количество коротких занимательных историй на математические темы, умело вкрапленных автором в текст книги. Желаю читателям получить максимум удо- вольствия в занятиях «интеллектуальной гим- настикой», возможность для которой предо- ставляет читателю эта книга. А. 77. Савин
Часть 1 ЗАДАЧИ Голубыми цифрами набраны номера стра- ниц, на которых начинаются главы с условия- ми задач в первой части книги, черными — номера страниц с решениями в конце книги. 9 17 25 32 39 Пестрые картинки из разных стран 148 Античные этюды 153 Школьные истории 160 Старое и новое из практики 166 Минуту на размышление 173 47 Арифметика с птичьего полета 180 55 Занимательная геометрия 189 63 Тренировка по современной мате- матике 195 7! 79 86 94 Математика на каждом шагу 204 Слово знаменитым математикам 213 Наша секция игр 220 Путь, время, скорость 222 101 I08 IJ6 Беседы на естественнонаучные темы 230 Напряженное расписание 236 Вокруг циркуля и линейки 245 124 132 139 Игра с числами 253 Огонь математических олимпиад 256 По разным странам 263 7
Не знание, а изучение, не обладание, а приобретение, не существующее, а гря- дущее доставляет величайшее наслаж- дение. Из письма К. Ф. Гаусса Яношу Бойяи.
ПЕСТРЫЕ КАРТИНКИ
Математика не принадлежит какому-ни- будь одному народу, она поистине ин- тернациональна. Нет ни одной страны, которая не поддерживала бы с матема- тикой дружеских отношений, не приум- ножала ее сокровищ и славы. А. И. Маркушевич 1. Франция. Во время летнего пикника че- тыре супружеские пары выпили 32 бутылки лимонада. Жены выпили: Жанна — 1 бутыл- ку, Жаклин — 2 бутылки, Колетта — 3 бу- тылки и Анетта — 4 бутылки. Мужья не усту- пили женам: месье Пон выпил столько же, сколько его жена, месье Дюбуа — вдвое боль- ше своей жены, месье Пейзан — втрое и месье Фонтен — вчетверо больше своих жен. Как зовут мадам Пон, Дюбуа, Пейзан и Фонтен? 2. Вьетнам. Эта задача известна с давних времен. Во вьетнамских деревнях старики-ри- соводы любят задавать ее молодежи. Так за- дача переходит от поколения к поколению. Для кормления 100 буйволов заготовили 100 охапок сена. Стоящий молодой буйвол съедает 5 оха- пок сена. Лежащий молодой буйвол съедает 3 охап- ки сена. Старые буйволы втроем съедают 1 охапку сена. Сколько молодых буйволов стоят, сколько лежат и сколько буйволов старых? 3. Югославия. Расставьте числа (в рамке слева) и знаки арифметических действий to
(в рамке справа) так, чтобы в каждой строке получились правильно решенные арифметиче- ские примеры. метровые столбы. Шоссе ведет из пункта А в пункт В. На каждом столбе указано рас- стояние в километрах как от пункта А, так и от пункта В. Расстояние от Л до В состав- ляет 999 км. На скольких километровых столбах обе надписи используют только 2 различные цифры? 5. Австрия. «Подальше положишь — по- ближе возьмешь». Перед нами хитрый крот. Между своей спальней А и выходом Е он проложил хитроумную систему ходов и камер. Каждое утро крот следует из Е в А и по до- роге проходит через свою запасную кладовую. Интересно, что отыскивает он ее по опреде- ленному правилу. Если крот достигает выхо- да Е, миновав 3, 5, 7, 9 или 11 промежуточ- ных остановок (обозначенных на плане круж- ками), то кладовая остается в стороне. Если же крот добирается до выхода Е после четно- го числа промежуточных остановок, то по до-
роге он непременно наталкивается на запас- ную кладовую. Между какими двумя камерами располо- жена запасная кладовая хитрого крота? 6. Болгария. Отец по имени Николай с сы- ном и отец по имени Петр с сыном отправи- лись удпть рыбу. Число рыб, пойманных Ни- колаем, оканчивается на 2, а число рыб, пойманных его сыном, — на 3, число рыб, пой- манных Петром, также оканчивается на 3, а число рыб, пойманных его сыном,— на 4. Число рыб, пойманных нашими рыболовами вместе, совпадает с квадратом некоторого на- турального числа. Как зовут сына Николая? 7. Дания. Рыбаки Адам, Бауэр, Кристиан- сен и Дазе (сокращенно А, В, С и D — по первым латинским буквам их имен), взвесив свой улов, установили следующее: (1) D поймал больше, чем С. (2) А и В вместе поймали столько же, сколько С и О (вместе). (3) А и D вместе поймали меньше, чем В и С (вместе). 12
Расположите результаты взвешиваний уловов а, Ь, с и d рыбаков А, В, С и D по величине. Математик или футболист? Однажды братья — физик Нильс Бор и математик Гаральд Бор — вместе с прия- телем отправились на прогулку по ули- цам Копенгагена. К удивлению прияте- ля, прохожие довольно часто здорова- лись с Гаральдом, Нильса же никто не приветствовал. «По-видимому, математи- ки в Копенгагене котируются высоко»,— заметил приятель. Нильс Бор возразил ему: «Не математики, а Гаральд. Ведь он любимый футболист нашего города!» 8. Венгрия. Учитель начертил на классной доске четырехугольник. Янош утверждал, что это квадрат. Имре считал, что четырехуголь- ник — трапеция. Мария думала, что на доске изображен ромб. Эва назвала четырехуголь- ник параллелограммом. Выслушав каждого и обстоятельно изучив свойства четырехуголь- ника, учитель установил, что ровно 3 из 4 утверждений истинны и ровно 1 утверждение ложно. Какой четырехугольник начертил учитель на классной доске? 9. Греция. Четырежды пять — двадцать: составьте из любых четырех фигур пентами- но прямоугольник со сторонами 4\5. ABCDEF G Н I J К L 13
Сколько решений допускает эта задача? (Сторона клетки считается равной единице. Каждая фигура пентамино состоит из 5 квад- ратов.) 10. США. Джон Гаррис из г. Санта- Барбара изобрел но- вую игру. «Путеше- ствие перекатывающем- ся игральной кости». Для того чтобы нам легче было следить за маршрутом игральной кости, выкрасим одну из ее граней в какой-нибудь цвет. С одного поля шахматной доски на соседнее игральная кость «путешествует», перекатываясь через ребро, совмещенное с общей стороной этих двух полей. А теперь решим задачу. Поставьте игральную кость на левое верх- нее поле шахматной доски цветной гранью вверх. Можете ли вы указать маршрут, «пу- тешествуя» по которому, игральная кость по- бывает по одному разу на всех полях шах- матной доски и окажется в правом верхнем углу цветной гранью вверх? Во время путе- шествия из угла в угол цветная грань играль- ной кости (так гласят правила игры) нигде, кроме начального и конечного поля, не долж- на быть обращена вверх. 11. ФРГ. На рисунке вы видите различ- ные предметы (бокалы, чашки, бутылки и кувшины), уравновешенные на чашах на- стольных равноплечих весов. Сколько бокалов, чашек и бутылок пона- добится, чтобы уравновесить 3 кувшина? 14
12. Бельгия. Найти трехзначные числа вида abc, цифры которых удовлетворяют уравнению а2 — Ь2 — с2=а — b — с (все 3 цифры числа должны быть различны). 13. Италия. Разделить заданный треуголь- ник с помощью зигзагообразной ломаной на 5 равновеликих частей. 14. ГДР. Во время перемены в классе оставались Ангелика, Бернд, Вольфганг и Мануэла. Кто-то из них разбил стекло. Учи- тель опросил ребят и получил от каждого по три ответа. Ангелика: 1. Окно разбила не я. 2. Я сидела в классе и читала. 3. Мануэла знает, кто разбил окно. Бернд: 1. Это сделал не я. 2. С Мануэлой я уже давно не разговариваю. 3. Окно разбил Вольфганг. 15
Вольфганг: 1. Я не виноват. 2. Окно разбила Мануэла. 3. Бернд лжет, когда утвержда- ет, будто окно разбил я. Мануэла: 1. Окно разбила не я. 2. Ангелика разбила окно. 3. Бернд знает, что я не вино- вата, ведь на перемене мы с ним играли вместе. Кроме того, каждый из них признался, что из трех ответов два истинных и один лож- ный. Кто разбил окно?
АНТИЧНЫЕ этюды
Когда радость особенно велика? Когда удается достичь желаемого. Фалес Милетский 1. Фу Ши (около 3000 г. до н. э.). В обо- значениях Фу Ши знак ттт -~: означал 6, знак — — означал 1 и знак EEF-zEE означал 3. а) Что означал знак ~ —? б) Сколько и каких знаков можно соста- вить из трех сплошных или состоящих из двух разделенных промежутком отрезков ли- ний? 2. Из Древнего Вавилона (около 2000 г. до н. э.). Длина и '/< ширины вместе состав- ляют 7 ладоней, а длина и ширина вместе — 10 ладоней. Сколько ладоней составляют длина и ши- рина в отдельности? 3. Из древнеиндийской математики (около 2000 г. до н. э.). В Древней Индии матема- тика распространялась как своего рода спорт. Для решения сложных задач устраивались состязания в присутствии многочисленных зрителей. Некоторые индийские руководства математики были написаны как учебные по- собия по проведению подобных состязаний — 18
для повышения мастерства любителей ум- ственного спорта. Автор одного из таких учеб- ников писал: «Следуя приведенным здесь правилам, можно придумать тысячи других задач. Подобно тому, как солнце затмевает своим сиянием звезды, слава ученого челове- ка, поставившего и решившего алгебраиче- скую задачу, затмевает славу других ученых в многолюдном собрании». Весь учебник это- го автора написан в стихах. Приведем лишь одну из задач, но не в стихотворном, а в про- заическом варианте. «Пчелы числом, равным квадратному кор- ню из полного числа их во всем рое, сели на куст жасмина, 5 * * 8/» пчел полетели назад к рою. И только одна пчела из того же роя кружи- лась над цветком лотоса, привлеченная жуж- жанием подруги, неосторожно угодившей в ловушку сладко благоухающего цветка. Сколько всего пчел было в рое?» 4. Арифметика древних китайцев (2000 г. до и. э.). В центре квадратного пруда шири- ной 10 шагов растет камыш, возвышающийся на I шаг над поверхностью воды. Если, стоя на берегу водоема, притянуть камыш к сере- дине любой из сторон, то он как раз касается края пруда. Какова глубина пруда? 5. Пифагор Самосский (около 580—501 гг. до н. э.). Поликрат (известный из баллады Шиллера тиран с острова Самос) однажды спросил на пиру у Пифагора, сколько у того учеников. «Охотно скажу тебе, о Поликрат»,— отвечал Пифагор.— «Половина моих учеников изучает прекрасную математику, четверть ис- следует тайны вечной природы, седьмая часть 2* 19
молча упражняет силу духа, храня в сердце учение. Добавь еще к ним трех юношей, из которых Теон превосходит прочих своими спо- собностями. Столько учеников веду я к рож- дению вечной истины». Сколько учеников было у Пифагора? 6. Греческий крест (около 500 г. до н. э.). Название этой фигуры \ связано с тем, что ___________ древние греки чертили \ такой крест на хлебах, \ считая его символом \ жизни. Вырежьте из кар- тона (или фанеры) гре- ------- ческий крест, разрежь- те его, как показано на рисунке, и сложите из получившихся частей квадрат. 7. Евклид (около 300 г. до и. э.). Однажды мула и осла нагрузили зерном. По дороге мул сказал ослу: «Если бы ты уступил мне одну меру своего груза, то я нес бы вдвое больше зерна, чем ты. А если бы я уступил тебе одну меру своего груза, то мы оба несли бы зерна поровну». Сколько мер зерна нес мул и сколько осел? 8. Из папируса Ринда (около 1700 г. до и. э.). Этот папирус, найденный в конце про- шлого века англичанином Риндом, представ- ляет собой фрагмент другого более древнего египетского труда по математике, относящего- ся, по-видимому, к III тыс. до и. э. Приведем две задачи из папируса Ринда. а) Некий математик насчитал на выгоне 20
70 коров. «Какую долю от всего стада состав- ляют эти коровы?» — спросил математик у пастуха. «Я выгнал пастись две трети от тре- ти всего стада»,— отвечал пастух. Сколько голов скота насчитывается во всем стаде? Встречаются в древнем папирусе и чисто формальные задачи, например следующая: б) Найдите х из уравнения Г/ , 2 \ , 1 / 2 \] I Lc+——х +—-х + —-х —=10 I \ О / О \ О О 9. Гиппократ Хиос- ский (около 440 г. до --- и. э.). Гиппократ на- ---\ чертил квадрированные &i \ \ им лунки и установил: I- ’// \ I сумма площадей двух \1/ Мз луночек М\ и М2 (за- а ---------------- полнены точками) рав- на площади треугольника АВС. Докажите это утверждение. 10. Шен Кан (ум. в 152 г. до н. э.). Три снопа хорошего урожая, 2 снопа среднего уро- жая и 1 сноп плохого дают 39 доу (старинная китайская мера) зерна. Два снопа хорошего урожая, 3 снопа среднего и 1 сноп плохого дают 34 доу зерна. Один сноп хорошего, 2 сно- па среднего и 3 снопа плохого урожая дают 36 доу зерна. Спрашивается: сколько доу зерна дает 1 сноп хорошего, 1 сноп среднего и 1 сноп пло- хого урожая? 11. Архимед (287—212 гг. до н. э.). Сущест- вует общая формула для вычисления площа- ди двух «арбелосов», или «сапожных ножей», 21
покрытых на рисунке точками. Эту формулу вывел Архимед. Вот она: 3= лГ' . 8 где t — длина отрезка АВ. Выведите формулу Архимеда. 12. Герои Александрийский (I в. до н. э.). Из-под земли бьют четыре источника. Первый заполняет бассейн за 1 день, второй — за 2 дня, третий — за 3 дня и четвертый — за 4 дня. За сколько времени наполнят бассейн все 4 источника вместе? 13. Римский математик (около I в. до н. э.). Адвокаты в Древнем Риме имели обыкнове- ние задавать друг другу задачи. Одна из та- ких задач гласит: «Некая вдова должна разделить оставшее- ся после смерти мужа наследство в размере 3500 динариев с еще не родившимся ребен- ком. По римским законам, если родится сын, то мать получает половину причитающейся ему доли, а в случае рождения дочери мать 22
получает вдвое больше нее. У вдовы родились близнецы — сын и дочь. Как разделить наследство, чтобы все тре- бования закона были соблюдены?» 14. Диофант Александрийский (III в. н. э.). По двум данным числам 200 и 5 найти третье число, которое, если его умножить на одно из них, дает полный квадрат, а если его умно- жить на другое число, дает квадратный ко- рень из этого квадрата. 15. Великолепную задачку мы находим в собранных много веков назад арабских сказ- ках «1001 ночь» (ночь 458-я): «Стая голубей подлетела к высокому де- реву. Часть голубей села на ветвях, а другая расположилась под деревом. Сидевшие на вет- вях голуби говорят расположившимся внизу: «Если бы один из вас взлетел к нам, то вас стало бы втрое меньше, чем нас всех вместе, а если бы один из нас слетел к вам, то нас с вами стало бы поровну». Сколько голубей сидело на ветвях и сколь- ко под деревом? 16. В старинной персидской легенде «Ис- тория Морадбальса», также вошедшей в сборник «1001 ночь», мудрец задает юной де- ве следующую задачу. «Одна женщина отправилась в сад собрать яблоки. Чтобы выйти из сада, ей нужно было пройти через 4 двери, у каждой из которой стоял стражник. Стражнику у первых дверей женщина отдала половину сорванных ею яб- лок. Дойдя до второго стражника, женщина отдала ему половину оставшихся яблок. Так же она поступила и с третьим стражником; а 23
когда она поделилась яблоками со стражни- ком у четвертых дверей, то у нее осталось лишь 10 яблок. Сколько яблок она собрала в саду?» Ученик спросил у Евклида: «Какая вы- года от того, что изучу все это?» Евклид позвал своего раба и приказал, указывая на юношу: «Дай ему 3 обола! Несчастный должен непременно получать выгоду от того, что он изучает!»
школьные ИСТОРИИ
Умение решать задачи — такое же прак- тическое искусство, как умение плавать или бегать на лыжах. Ему можно нау- читься только путем подражания или упражнения. Дьердь Пойа 1. Учащиеся одного класса написали кон- трольную по математике. Треть из них невер- но решила по 1 задаче, четвертая часть клас- са неверно решила по 2 задачи, ‘/б — по 3 за- дачи и */s— неверно решила все 4 задачи. Сколько учеников правильно решили все задачи, если в классе не более 30 человек? 2. Лучшего математика из 5 «А» класса попросили отгадать натуральное число, о ко- тором его друзья высказали следующие ут- верждения. Вольфганг: Это, число простое. Карин: Это число 9. Петер: Это число четное. Росвита: Это число 15. Известно, что Вольфганг и Карин вместе высказали ровно одно истинное утверждение (так же, как Петер и Росвита). Что это за число? 3. Для школьной мастерской купили 29 де- талей на 29 марок. Детали были трех сортов: по 10 марок, по 3 марки и по 50 пфеннигов за штуку. Каждого сорта куплено не менее 1 де- тали. Деталей других сортов среди купленных не было. Сколько деталей каждого из трех сортов куплено? 26
4. Для нумерации страниц учебника, по словам учителя, потребовалось 6869 цифр. Его хитроумные ученики тотчас же сообрази- ли, сколько страниц в книге. Как они подсчитали, сколько страниц в учебнике? 5. Двоим друзьям потребовалось вычислить 42—З2. Они заметили, что результат — число 7 — равен сумме оснований квадратов — чи- сел 4 и 3. Проверив свое открытие на числах 10 и 11, друзья установили, что оно подтверж- дается и вэтом случае: II2—102=21 = Н + 10. После этого друзья нашли все пары (а, Ь) на- туральных чисел а>Ь, для которых разность а2—Ь2 равна сумме a-j-b. Как друзьям удалось найти все такие чис- ла (а, &)? 6. Рози тренируется в школьном спортив- ном кружке. Одно из упражнений — ритмич- ная ходьба с пружинящими наклонами. Уп- ражнение выполняется на дорожке длиной 30 м, в начале и в конце которой стоят флаг- штоки. Рози выполняет упражнение так: два шага вперед, наклон, шаг назад, два шага вперед, наклон, шаг назад и т. д. Сколько шагов она успевает сделать от флагштока до флагштока, если каждый шаг ее равен 50 см? 7. Войдя в класс после перемены, ребята увидели на доске арифметические примеры с пропущенными кое-где числами и знаками действий (недостающие числа и знаки заме- нены звездочками). Эти примеры придумал учитель математики, большой любитель задач на смекалку. 27
Восстановите недостающие числа и ариф- метические действия. 8. Отец обещал сыну за каждую правиль- но решенную задачу опускать в копилку по 10 пфеннигов. За каждую неправильно решен- ную задачу сын должен возвращать отцу по 5 пфеннигов. После того как было решено 20 задач, у сына в копилке оказалось 80 пфен- нигов. Сколько задач сын решил неправильно и сколько без единой ошибки? 9. Учитель математики предложил учени- кам обойти ходом коня клетки с вписанными частями слов и вспомнить определение, встре- ецией вы о X пук против аллель трап ко поло ны ается ны четыре пар уголь ро лый две ыв жные тор наз сто ого ник 28
чавшееся на одном из уроков. Поразмыслив, ученики справились с задачей. Что за определение скрыто в клетках таб- лицы? 10. Перелистывая страницы старинного учебника арифметики «цифирных дел масте- ра» Йоханнеса Альберта «Пером по линей- кам» (около 1750 г.), библиотекарь нашел сле- дующую интересную задачу (по замыслу ав- тора учебника она предназначалась для «лю- дей низкого и подлого сословий и любителям арифметики»). Два подмастерья одновременно отправи- лись пешком из Виттенберга в Испанию. Пер- вый подмастерье ежедневно проходил по 7 миль. Второй в первый день прошел 1 милю, во второй — 2 мили, в третий — 3 мили и т. д.—в каждый следующий день на 1 милю больше, чем накануне. Через сколько дней подмастерья встрети- лись? 11. Три девочки задали на перемене своей подруге задачку на сообразительность. Вот что они ей сообщили. У Уты вдвое больше цветных карандашей, чем у Регины, а у Сабины на 13 карандашей меньше, чем у Регины. Сколько цветных карандашей у каждой из нас, если, пересчитав все карандаши, мы полу- чили простое число? Оно меньше 50, а сумма его цифр равна 11. Итак, сколько цветных карандашей у Уты, Регины и Сабины в отдельности ii у всех вме- сте? 29
Ум и остроумие Еще в школьные годы К- Ф. Гаусс неод- нократно поражал учителя своим умом и остроумием. Однажды учитель спросил его: «Гаусс, я сейчас задам тебе два во- проса. Если на первый ты ответишь пра- вильно, то на второй можешь не отве- чать. Итак, скажи мне, сколько иголок на рождественской елке?» Гаусс без про- медления ответил: «67534». «Как ты так быстро сосчитал иголки?»— изумился учитель. «А это уже второй вопрос, гос- подин учитель»,— улыбнулся Гаусс. 12. От числа одну восьмую Взяв, прибавь ты к ней любую Половину от трехсот, И восьмушка превзойдет Не чуть-чуть — на пятьдесят Три четвертых. Буду рад, Если тот, кто знает счет, Мне число то назовет. Эту задачу более 200 лет назад задавал своим ученикам учитель арифметики Иоганн Хемелинг. 13. Учителя Альтман (А), Брендель (В) и Клаузнер (С) преподают в одном классе ма- тематику (М), физику (Ф), химию (X), био- логию (Б), немецкий язык (Н) и историю (И). Каждый учитель ведет по 2 предмета. Учитель химии живет в одном доме с учителем мате- матики. Альтман — самый молодой из трех преподавателей. Учитель математики часто играет в шахматы с Клаузнером. Учитель фи- зики старше учителя биологии, но младше 30
Бренделя. Тот из трех учителей, кто старше двух других, живет дальше всех от школы. Какие предметы преподает каждый из трех учителей? 14. В гимнастическом зале стоит несколь- ко одинаковых по длине скамей. Если спорт- смены попытаются сесть по 6 человек на скамью, то одна скамья окажется незапол- ненной: на ней сядут лишь 3 спортсмена. Если же спортсмены попытаются сесть по 5 чело- век на скамью, то 4 спортсменам места не хватйт. Сколько спортсменов и сколько скамей в гимнастическом зале? 15. Для вручения наград всех победителей математической олимпиады выстроили в ряд на сцене. «Шестой слева,— заметил Карл,— единственный среди участников олимпиады, набравший полное число баллов». «Совершен- но верно,— согласилась Аннерозе,— он стоит десятым справа». Сколько было победителей олимпиады? Можете ли.вы обобщить задачу? 16. После экскурсии на завод учитель предложил ребятам следующую задачу. Сов- ременная техника позволяет изготовлять из металла проволоку толщиной всего 0,002 мм. Какой длины получится проволока кругло- го сечения диаметром 0,002 мм из 2 г серебра?
СП новое ПРАКТИКИ
Много знать еще не означает многое понимать. Гераклит 1. Греческий математик Митродор (III в. до н. э.) поставил следующую задачу: «Цар- ская корона имеет массу 60 мин (1 мина= 100 драхм='/бо таланта) и отлита из сплава золота, меди, свинца и железа. На золото и медь приходится %, на золото и свинец —2/з> на золото и железо —% массы короны. Сколько мин золота, меди, свинца и желе- за в царской короне? 2. Эратосфен (умер около 195 г. до н. э. в Александрии) с поразительной точностью измерил окружность Земли. Эратосфен знал, что в Сиенне (нынешнем Ассуане), располо- женном в Верхнем Египте, Солнце в полдень самого продолжительного дня в году стоит в зените. Измерив в этот день точно в полдень угол, под которым Солнце видно в Алексан- дрии, Эратосфен обнаружил, что направление на Солнце отклоняется от отвесной линии на 7,5°. Из собственных измерений Эратосфену было известно, что Александрия расположена в 5000 египетских стадиях к северу от Сиенны. Пользуясь этими данными, Эратосфен вычис- лил окружность Земли. а) Чему равна окружность Земли в египет- ских стадиях? б) Перевести полученную величину в ки- лометры, считая, что 1 египетская стадия равна 184,72 м. в) Сравнить полученное Эратосфеном зна- чение с современным (около 40 000 км). 3 Ф-73 33
3. Задача Этьенна Безу (1730—1783). По контракту работникам причитается по 48 франков за каждый отработанный день, а за каждый неотработанный день с них взыски- вается по 12 франков. Через 30 дней вы- яснилось, что работникам ничего не причи- тается. Сколько дней они отработали в течение этих 30 дней? 4. Мебельная фабрика каждый месяц вы- пускает на 10 столов больше, чем в предыду- щий. За год фабрика выпустила 1920 столов. Сколько столов было выпущено в июне и в декабре? 5. Шнур для электрической проводки электрик скручивает из двух медных проводов («жил*). Провода могут иметь изоляцию 8 различных цветов: зеленого (3), голубого (Г), белого (Б), красного (К), черного (Ч), жел- того (Ж), серого (С) и фиолетового (Ф). Каждый шнур можно маркировать двумя бук- вами — по цвету изоляции жил, из которых он скручен. Цвета изоляции должны быть не- пременно различные: если одна жила имеет зеленую изоляцию, то изоляция другой может быть любого цвета, кроме зеленого. Сколько шнуров различных маркировок может скрутить электрик из жил с изоляцией 8 цветов? 6. Помещение освещается 32 лампами на- каливания общей мощностью 1800 Вт. На складе имеется достаточно большой запас ламп по 40, 60 и 75 Вт. Никаких других ламп на складе нет. Сколькими различными способами можно удовлетворить заявки, на комплекты ламп для освещения этого помещения? 34
7. Детали 1—7 вдвинуты в футляр сверху (каждая деталь двигалась строго по верти- кали). В какой последовательности производилась укладка? 8. Деревянная балка круглого сечения ве- сит 300 Н. Сколько бы весила такая балка, если бы она была вдвое толще и вдвое короче? 9. Томас Алва Эдисон (1847—1931) обла- дал тонким чувством юмора. Его многочис- ленные посетители часто удивлялись, почему калитка в саду перед домом великого изобре- тателя открывается с трудом. Наконец, один из друзей спросил у Эдисона: «Неужели та- кой технический гений, как ты, не может от- регулировать какую-то калитку?» «Калитка отрегулирована именно так, как надо,— сме- ясь возразил Эдисон.— Я сделал от нее при- вод к цистерне, и каждый, кто приходит ко мне, накачивает в цистерну 20 л воды». Если бы каждый посетитель вместо 20 л накачивал в цистерну 25 л воды, то для за- полнения цистерны понадобилось бы на 12 человек меньше. Сколько воды вмещает цистерна? з* 35
10. Какой из этих 8 рисунков маляр нанес на стену изображенным здесь валиком? 11. Двумя грузовыми машинами требуется перевезти 143 т сыра. Грузоподъемность од- ной машины в 1,5 раза больше, чем другой. Для перевозки всего груза полностью гружен- ной машине меньшей грузоподъемности пона- добится совершить 31 рейс и 27 рейсов маши- не большей грузоподъемности. Сколько тонн сыра перевозит за 1 рейс каждая машина? 12. Даны проекции тела на две взаимно перпендикулярные плоскости — вид спереди и вид сверху. Можно ли по этим двум проекциям одно- значно восстановить тело или существуют не- сколько тел, имеющих одинаковый вид спе- реди и сверху? 36
13. Сколько носильщиков необходимо на- нять путешественнику, намеревающемуся со- вершить шестидневный переход по пустыне по замкнутому маршруту, если и он сам, и каж- дый из носильщиков могут нести четырех- дневный запас пищи и воды для 1 человека? 14. В одном из залов «Дома занимательной науки» в Ленинграде* посетителям показы- вали карту. Требовалось обойти все 17 мостов, соединяющих острова и берега Невы, на ко- торых стоит Ленинград, так, чтобы каждый мост оказался пройденным не более 1 раза. Возможно ли это? * Подробнее о замечательной деятельности этого уникального научно-просветительного учреждения, ор- ганизованного сотрудником комбината наглядной аги- тации Ленсовета В. А. Камским, известным популяри- затором науки Я. И. Перельманом, страстным пропа- гандистом астрономии В. И. Прянишниковым, писате- лем Л. В. Успенским и художником А. Я» Малковым, можно прочесть в статье Г. И. Мишкевича «Дом зани- мательной науки» (Наука и жизнь, 1968, № 3, с. 71— П).—При меч. пер. 37
15. Объем древесины некоторого участка леса увеличивается за год в среднем на 4%. Предположим, что при благоприятных усло- виях годовой прирост древесины составляет 5%, а при неблагоприятных условиях—всего лишь 3%. На сколько процентов увеличится объем древесины за 2 года, если вслед за годом с благоприятными условиями выдастся год с неблагоприятными условиями? 16. Бюро услуг принимает заказы на изго- товление копий с архивных документов. Изго- товить 3 копии стоит 6 марок, 5 копий — 9 марок и 9 копий — 15 марок. Во сколько марок обойдется заказчику из- готовление п копий? (Цена складывается из платы, взимаемой за пользование архивным документом, и стоимости изготовления копий.) 17. В книге 152 страницы, на каждой стра- нице в среднем по 45 строк, в каждой строке (также в среднем)— по 68 знаков. Сколько страниц понадобится при наборе того же текста более крупным кеглем, если на странице в среднем умещается по 32 стро- ки, а в каждой строке в среднем — по 51 зна- ку? 18. Собираемый пчелами нектар примерно на 70% состоит из воды. Производимый пче- лами мед содержит около 17% воды. Сколько нектара необходимо для того, что- бы получить 1 кг меда? 19. Имеется 1 000 000 стальных шариков диаметром 1 мм. Сможет ли один человек поднять их, если все шарики упаковать в ящик? 38
МИНУТУ на РАЗМЫШЛЕНИЕ
i Искра знания возгорается в том, кто до- I стигает понимания собственными силами. | Из трактата «Лилавати» i индийского математика Бхаскары (XII в.) j 1. В одном старинном задачнике суд Па- )иса описан следующим образом. Богини Ге- )а, Афродита и Афина пришли к юному Па- >ису, чтобы тот решил, кто из них прекраснее. Представ перед Парисом, богини высказали следующие утверждения. Афродита: Я самая прекрасная. (1) Афина: Афродита не самая прекрасная. (2) Гера: Я самая прекрасная. (3) Афродита: Гера не самая прекрасная. (4) Афина: Я самая прекрасная. (5) Парис, прилегший отдохнуть на обочине доро- ги, не счел нужным даже снять платок, кото- рым прикрыл глаза от яркого солнца. Но бо- гини были настойчивы, и ему во что бы то ни стало нужно было решить, кто из них самая прекрасная. Парис предположил, что все ут- верждения прекраснейшей из богинь истинны, а все утверждения двух остальных богинь ложны. Мог ли Парис, исходя из такого предпо- ложения, вынести то решение, которого ожида- ли от него богини, и если мог, то кто прекрас- нейшая из богинь? 2. «Абракадабра»— волшебное слово. В прошлом его нередко можно было встретить выгравированным на амулетах: по существо- вавшему некогда поверью, оно ограждало вла- дельца амулета от болезней и прочих несчас- тий. 40
Перед вами 2 конфигурации. Сколькими способами можно прочитать в них слова «аб- ракадабра» и «математика», спускаясь от каждой буквы на одну строку вниз или на од- но место правее или левее? А Б Б Б Р Р Р Р Р А А А А А А А ккккккккк ААААААААААА ддддддддд А А А А А А А Б Б Б Б Б Р Р Р К М М М М М А А А А А А Т Т Т Т Т Т Т ИИИИИИИИ К К К К К К К К аааааааааа А 3. — Куда ты так спешишь? — На шестичасовой поезд. Сколько еще минут осталось до отправления? — 50 минут назад с 3 часов прошло вчет- веро больше минут, чем сейчас осталось до отправления шестичасового поезда. Во сколько часов и минут происходил этот диалог? 4. За обеденным столом собрались один дедушка, одна бабушка, два отца, две матери, четверо детей, три внука, один брат, две сес- тры, два сына, один свекр, одна свекровь и одна невестка. На сколько персон минимально должен быть накрыт стол? 5. Маневровая задача. Локомотив должен прибыть на станцию в правом верхнем углу рисунка. Для этого ему необходимо пройти пронумерованные стрелки — одни в прямом направлении, другие в обратном. Железнодо- рожный путь в некоторых местах прегражда- ют препятствия. 41
Как должен маневрировать локомотив, чтобы прибыть на станцию? 6. В вагоне метро рядом сидят 5 девушек. Анетта сидит через столько же человек от Ба- бетты, как Колетта. Доретта сидит через столь- ко же человек от Анетты, как Колетта. Пре- красная Жанетта сидит между двумя своими лучшими подругами. Как их зовут? 7. Для того чтобы отгадать задуманное целое число, его необходимо умножить на сле- дующее за ним число и из произведения вы- честь задуманное. Как по полученному результату найти за- думанное число? 42
8. а) В верхнем магиче- ском квадрате требуется рас- ставить 9 степеней двойки так, чтобы произведение чисел, стоящих в каждой строке, в каждом столбце и на каждой диагонали, были равны. б) В нижнем магическом квадрате 9 чисел, вписанных в клетки, требуется располо- жить так, чтобы произведения чисел, стоящих в каждой стро- ке, в каждом столбце и на каждой диагонали, были рав- ны а3Ь3, после чего положить а=2, Ь = 3. 21 22 2з 24 2s 26 27 28 29 1 а а2 ь аЬ а2Ь ь2 аЬ2 а2Ь2 9. В каждой из четырех серий фигур верх- него ряда требуется найти в нижнем ряду ту из четырех фигур, обозначенных буквами А, В, С, D, которой можно, не нарушая логи- ческой последовательности, заполнить правый пустой квадрат в верхнем ряду. А А D А В С D D А В С D
. 10. «Четырехзначный номер автомашины моего учителя математики очень легко запом- нить,— сообщил Ганс своему приятелю.— Но- мер симметричен, а сумма его цифр совпада- ет с числом, образуемым первыми двумя циф- рами». Какой номер у автомашины учителя мате- матики? 11. Моника попросила Марию-Луизу: «На- зови мне любое трехзначное число, все циф- ры которого отличны от нуля и попарно раз- личны. Запиши его на листке бумаги и при- бавь к нему все трехзначные числа, которые получаются из него при перестановке цифр»; И не успела Мария-Луиза вычислить сум- му, как Моника назвала ответ. Как ей удалось так быстро получить ре- зультат? 12. Задача из английской газеты «Обс^р- вер». Все звери в зоопарке находятся не в своих клетках. Служителю необходимо как можно быстрее разместить животных по их клеткам. Поскольку все звери, кроме осла,— хищники, их нельзя помещать вдвоем в одну клетку или выпускать вдвоем в общий вольер, в который открываются клетки. Какое наименьшее число «переселений» должен произвести служитель зоопарка, что- 44
бы ее звери оказались в своих клетках? (Предполагается, что в присутствии служите- ля звери друг на друга не нападают.) 13. Известный англий- ский составитель задач по занимательной мате- матике Барнард назвал эту игрушку, известную у нас под названием «мо- биль», эквилябром (про- изводное из «эквилибри- стики» и «канделябра»). Какие 2 предмета (2 рыбки, 2 шарика, 2 коло- кольчика, 2 коромысла или какая-то комбинация из 2 различных предме- тов) могут удержать систему в равновесии вместо вопросительного знака? (Вес нитей в отличие от веса коромысла пренебрежимо мал.) 14. В одну из разновидностей игры с иг- ральными костями играют по следующим правилам. Если при бросании кости выпадает четное число, то игрок получает столько «плюс очков», сколько выпало. Если при очередном бросании кости выпадает нечетное число оч- ков, то игрок получает столько «минус очков», сколько выпало. Игрок бросил кость 5 раз подряд. Два раза выпало одно и то же число очков, а в трех остальных случаях число очков было различ- ным. При подсчете число «плюс очков» сов- пало с числом «минус очков». По скольку очков выпадало в отдельных бросаниях? 45
15. Слова, приводимые ниже под номерами (1) — (4), составлены из тех же букв, что и названия профессий, требующих основатель- ной математической подготовки. Что это за профессии? (1) ДОЕНИЕ, РАНЖИР (2) ТРОТИЛ, СМЕСЬ, ОТО* (3) ВАЛ, МЕТЕЛЬ, ПОРА (4) РОСТ, ПИ, ГРАММ 16. Задача Л. Эйлера. Некий чиновник ку- пил лошадей и быков за 1770 талеров. За каждую лошадь он уплатил по 31 талеру, а за каждого быка — по 21 талеру. Сколько лошадей и сколько быков купил чиновник? Сколько решений — одно или не- сколько — допускает задача? Общая теория относительности.
арисрметика С ПТИЧЬЕГО ПОЛЕТА
Человек, смолоду приученный к догма- тическому мышлению, к чрезмерному почтению к абстрактным формулам, об- речен на постоянные и не очень приятные для него столкновения с жизнью. Э. Ильенков* В эпоху Возрождения в XVI в. математика достигала своего расцвета. Введя буквенные обозначения неизвестных величин, Франсуа Виет (1540—1603) формализовал алгебраи- ческие преобразования и тем самым сделал решающий шаг. Работы Виета позволили при- менить алгебраические методы к задачам, в которых величины принимают числовые зна- чения, что намного упростило вычисления. Большие заслуги в развитии арифметики принадлежат Михаэлю Штифелю (1487— 1567). В 1544 г. вышло из печати его трех- томное сочинение «Arithmetica Integra» (Пол- ная арифметика). В нем автор систематичес- ки изложил весь свод математических знаний своего времени. Эту главу мы начинаем с одной из задач Штифеля. 1. Сумма двух чисел равна 19, а сумма их квадратов равна 205. Что это за числа? 1. Пусть х и у — натуральные числа. В следующих примерах требуется заменить звездочки знаками >, < или = так, чтобы получились истинные высказывания. * Ильенков Э. В. Учитесь мыслить смолоду. М.. Знание, 1977, с. 59. 48
а) Если x>8, то x-f-3 *10. б) Если 60x=50i/, то х * у. в) Если 5х>10 и у>х, то у * 3. г) Если х>у, то у+2 *х-}-5. д) Если х>у, то 60 — х *75 — у. е) Если у<.5, то Зу *17. 3. Найдите 2 различных натуральных числа, удовлетворяющих следующим усло- виям: а) их среднее геометрическое на 4 больше меньшего из чисел; б) их среднее арифметическое на 6 мень- ше большего из чисел. 4. Выясните, существуют ли натуральные числа, обладающие следующими свойствами. Остаток от деления такого числа на 3 равен 1, на 4 равен 2, на 5 равен 3, на 6 равен 4. Если такие числа существуют, укажите наименьшее из них, обладающее всеми нуж- ными свойствами. 5. Сумма двух натуральных чисел равна 90. Сумма 25% первого и 75% второго числа равна 30. Требуется найти оба числа. 6..В ыберите значения переменных а, Ь и с из чисел 13, 15 и 20 так, чтобы значение вы- ражения а(с~Ь) Ь—а было целым положительным числом. 4 Ф-73 49
7. Какое значение принимает выражение а (а+2) +с (с — 2) — 2ас при а — с=7? 8. Дано множество U={3, 4, 5, 6, 7, 8, 9} и два его подмножества Л={4, 6, 7} и В={4, 5, 6, 8}. Из каких элементов состоят следующие множества *? а) ЛОВ, в) Б—U -^В. д) Л^В б) ЛПВ, г) и=и^и. Математические головоломки Магический квадрат. Какие целые поло- жительные значения следует придать пере- менным х, у и z, чтобы суммы трех чисел, стоящих в каждой строке, в каждом столбце и на каждой диагонали, были равны? Най- денное решение нетрудно проверить: числа, стоящие в четырех угловых клетках, четны и * Для тех, кто не знаком с обозначениями опера- ций, принятыми в теории множеств, поясним, что ХЦ У — множество (называемое объединением мно- жеств X и У), состоящее из элементов, каждый из ко- торых принадлежит хотя бы одному из множеств X и У, ХПУ — множество (называемое пересечением множеств X и У), состоящее из элементов, принадлежащих как множеству X, так и множеству У; Х^ У — множество (разность множества X и множества У, порядок мно- жеств существен!), состоящее из тех элементов мно- жества X, которые не принадлежат множеству У.™ Примеч. пер. 50
образуют арифметическую прогрессию, сумма которой в 4 раза больше числа, стоящего в центральной клетке. X х + у -10 4z - х X + Z ' У Z Z + у - X 2z + у — х 27 Кросснамбер. Дрезденский математик К. Хайнрих предложил следующий «числовой кроссворд» (кросснамбер). В каждую из 36 клеток следует вписать по одной' из цифр О, 1, 2, 9. Возникающие при этом двух-, трех- и четырехзначные числа «простираются» по горизонтали или по вертикали от клетки с номером до ближайшей ограничительной черты или края большого квадрата. Все эти числа должны удовлетворять следующим условиям. 1 2 3 4 5 6 7 8 9 10 12 ТГ”" 15 ТУ” Ч 18 19 20 По горизонтали. 1. Кратное числа, стоя- щего под номером 3 по горизонтали. 3. Квад- рат числа, стоящего под номером 3 по верти- кали. 6. Кратное суммы цифр числа, стояще- 4* 51
го под номером 7 по вертикали. 8. Число с та- кой же суммой цифр, как у числа, стоящего под номером 16 по вертикали. 9. Кратное квадратному корню из числа, стоящего под номером 14 по вертикали. 10. Кратное числа, стоящего под номером 14 по горизонтали. 12. Число, стоящее под номером 9 по гори- зонтали. 14. Простое число. 15. Квадрат, являющийся одновременно кубом., 17. Число, стоящее под номером 14 по вертикали. 19. Простое число. 20. Число с суммой цифр, как у числа под номером 3 по вертикали. По вертикали. 1. Произведение чисел, стоя- щих под номером 14 по горизонтали, и под номером 18 по вертикали. 2. Кратное числа, стоящего под номером 16 по вертикали. 3. Ку- бический корень из числа, стоящего под но- мером 5 по вертикали. 4. Кратное числа, стоящего под номером 3 по горизонтали. 5. Число, одинаково читаемое слева направо и справа налево. 7. Простое число. 10. Крат- ное числа, стоящего под номером 14 по гори- зонтали. 11. Кратное числа, стоящего под номером 19 по горизонтали. 13. Простое число. 14. Число, стоящее под номером 17 по гори- зонтали. 16. Квадратный корень из числа, стоящего под номером 15 по горизонтали. 18. Простое число. 9. При каких целых неотрицательных зна- чениях а, Ь, х, у и z истинны следующие вы- сказывания? (В каждом случае укажите все множество допустимых значений.) а) 5<а<60, г) 30—(z«z)=z, б) (х+3)4=4х+12, д) 3(&+1)<10. в) (5у)+у-4=9у, 52
Есть ли еще вопросы? Английский физик П. А. М. Дирак при- вык выражаться ясно и четко. В конце одного из своих докладов он спросил, обращаясь к аудитории: — Есть ли еще вопросы? — Я не понял, откуда следует вон та формула,— отозвался один из слушате- лей. — Это не вопрос, а утверждение,— воз- разил Дирак и, обращаясь к аудитории, повторил: — Есть ли еще вопросы? 10. Найдите все двузначные натуральные <исла, которые равны утроенной сумме своих цифр. 11. Какими цифрами следует заменить а и b (одинаковым переменным соответствуют одинаковые цифры), чтобы выполнялось ра- венство (а-|-а)-|-3(&-[-6) =а“-|-&а? 12. При каком натуральном числе п про- изведение предшествующего числа и числа, следующего за п, равно 2208? 13. Какие натуральные числа х и у удов- летворяют уравнению — + — + -1—= 1? х у ху 14. Можно ли заменить девять неизвест- ных а, Ь, с, ..., h, i в системе уравнений 53
я+^4“с=^4"е+/=£4“Л“Н девятью цифрами 1, 2, 3, ..., 8, 9 так, чтобы равенства не нарушались? 15. Сколькими способами можно заменить переменные а и b числами от 0 до 20, не на- рушив неравенства а<_Ь? 16. При каких натуральных числах а>Ь>0 выполняется неравенство 17. Укажите любые 2 значения перемен- ных х, t и z, при которых выполняются сле- дующие соотношения: а) — : х> —, б) 7 : t<7, в) -2-: z4= -1. 2 2 ’ 2 2 18. Найдите множество решений неравен- ства х2+ (х4-1 )2+ (х+2)2> (х+3)2+ + (х+4)2+(х+5)2. 19. Найдите множество решений уравнения (Х2+д _|_ 1) (2х2-|-2х — 3) = — 3 (1 — х — х2). 20. Найдите все упорядоченные пары (х, у) неотрицательных целых чисел х и у, удовлет- воряющих системе неравенств х+у<4, (1) 2х4-5у>10. (2) 21. Найдите вЬе упорядоченные тройки (х, у, z) неотрицательных целых чисел х, у и z, удовлетворяющие системе уравнений 7x-]-5w — z=8, y+z— 11. 54
ЗАНИМАТЕЛЬНАЯ ГЕИЕ1ПЖ
О «сеансах», или беседах, проводимых из- вестным польским математиком Стефаном Банахом (1892—1945), ныне ходят легенды. Банах приглашал своих единомышленников— профессиональных математиков и студентов Львовского университета — в «Шотландское кафе», где обсуждал с ними различные инте- ресные и нередко весьма необычные матема- тические идеи и проблемы. Банах считал, что в непринужденной атмосфере среди отдыхаю- щей публики под звуки музыки и уличного шума, доносящегося из-за зеркальных витрин заведения, его коллегам — юным и зрелого возраста — легче отрешиться от скованности. Накал страстей во время научных споров не- редко бывал весьма высоким. К великому не- удовольствию официантов необходимые вы- кладки производились спорящими сторонами тут же на столиках белого мрамора. Во избе- жание скандала Банах завел толстую книгу. В нее стали записывать наброски решений, которые затем постепенно дополнялись и усо- вершенствовались. За особо изящные и ори- гинальные решения участники круглого стола устанавливали специальные призы — от чаш- ки кофе «мокко» до живого гуся. Частым гостем этих бесед был близкий друг Банаха замеча- тельный польский математик Гуго Штейнгауз, известный не только своими научными дости- жениями, но и как автор многочисленных задач по занимательной математике. Эту главу мы хотим начать с одной из задач, поставленных Гуго Штейнгаузом. 1. Измерить с помощью линейки длину пространственной диагонали кирпича, имею- щего форму прямоугольного параллелепипе- 56
да, т. е. расстояние между двумя его наибо- лее удаленными вершинами. Предлагаемый способ измерения диагона- ли должен быть применимым на практике, т. е. таким, чтобы его можно было внедрить на производстве. Пользоваться теоремой Пи- фагора не разрешается. 2. Даны 2 развертки куба: (1) и (2). Какие из изображенных здесь кубов мож- но сложить из этих разверток? 3. Каждую из изображенных здесь раз- верток сложите в куб. (Куб должен быть «замкнут со всех сторон».) 57
4. а) Трапеция, которую вы видите на ри- сунке* составляет часть равностороннего тре- угольника? Разделите эту трапецию на 4 равновели- кие части. б) Разделите этот невыпуклый шести- угольник на 4 равновеликие части. 5. Кому из девушек адресовано письмо, конверт которого в развернутом виде изобра- жен слева? 58
6. От одного кубика, помеченного круж- ком на верхней грани, требуется пройти к другому, т. е. проложить путь либо слева направо, либо справа налево. Ходить можно только по горизонтали и вертикали на столь- ко шагов, сколько очков свыпало» на исход- ном кубике, т. е. стоит на его верхней грани. (Например, если на верхней грани стоит 5 очков, то при очередном ходе следует сде- лать 5 шагов, а если на верхней грани стоит 2 очка, то 2 шага.) С каждым ходом направ- ление продвижения меняется. Если при оче- редном ходе оказывается невозможным сде- лать нужное число ходов в выбранном на- правлении, то направление выбрано неверно. В подобных случаях следует попытаться про- двинуться в другом направлении. рядовом ВиядоябрВ ШШРЯЯИЙМ KassiH 59
Требуется найти ЗЙ •л 12 60
В каждом случае элементы фигуры, кото- рые требуется найти, изображены голубым <4 цветом. 7. Фигуры а, Ь, с, d и е разрезаны на части 1—12, изображенные на рисунке. Части какой фигуры вы видите на каж» дом из этих рисунков? 61
8. Как сложить из квадрата правильный шестиугольник? (Квадрат разрешается как угодно перегибать и складывать. Размеры шестиугольника несущественны.) 9. Четыре стержня А, В, С и D имеют соответственно длину а, Ь, с и d. Если к стержню А приставить стержень В, то полу- чится стержень такой же длины, как и стер- жень С, а стержень В имеет такую же длину, какую имеет стержень, составленный из стержней А и D. Наконец, длина стержня D — лишь % длины стержня С. Определить длины Ь, с и d, выбрав за еди- ницу длины а. 10. Из одного листа бумаги требуется сде- лать заготовку для 16-страничной тетрадки и, не разрезая, заранее пронумеровать стра- ницы. Левая половина каждого разворота должна иметь четный номер, а правая — не- четный. Как сложить лист и пронумеровать стра- ницы будущей тетради? 11. Два соседних угла А и В шахматной доски A BCD соединены отрезками прямых с серединой М противоположной стороны. Образовался треугольник AMD. Сколько полей шахматной доски не имеют с треугольником AMD общих внутренних точек?
ТРЕНИРОВКА ПО СОВРЕМЕННОЙ математике
Не клянись именем своего учителя, а приведи доказательство. Древняя поговорка 1. При повторении темы «Рациональные и иррациональные числа» ученики среди про- чих высказали следующие утверждения: а) Произведение двух различных иррацио- нальных чисел есть всегда иррациональное число. б) Сумма двух различных иррациональ- ных чисел есть всегда иррациональное число. в) Сумма рационального и иррациональ- ного чисел есть всегда иррациональное число. Какие из этих утверждений истинны и ка- кие ложны? 2. Доказать, что удвоенное произведение произвольного натурального числа и числа, следующего за ним, на 1 меньше суммы квад- ратов этого числа и числа, следующего за ним. 3. Прямоугольник составлен из 7 конгру- энтных квадратов. Доказать, что углы аир удовлетворяют неравенству 26,5°<а+Р<26,6° а 1 4. Знания учащихся в ГДР оцениваются по 5-балльной системе: высшей считается 64
оценка 1, затем в порядке убывания идут оценки 2, 3 и т. д. В конце учебного года ученик решил под- считать свой средний балл. Он знал, что по 6 предметам ему поставили единицу, а по 3 другим предметам — тройку. Оценки по 3 остальным предметам еще не были извест- ны, но он твердо рассчитывал получить по ним либо 2, либо 3. Какие оценки ему следовало бы получить, чтобы средний балл был лучше, чем 2? 5. Две задачи Исаака Ньютона. а) Даны 3 члена геометрической прогрес- сии. Их сумма равна 19, а сумма их квадра- тов— 133. Определить эти члены. б) Даны 4 члена геометрической прогрес- сии. Сумма двух крайних членов равна 13. двух средних 4. Определить эти члены. 6. Дано линейное неравенство 8(2*+Р. <3x4-2 5 а) Решить это неравенство в области дей- ствительных чисел. б) Найти следующие множества, перечис- лив их элементы: 1) множество решений. неравенства в области натуральных чисел; 2) множество решений Ра неравенства в целых числах, удовлетворяющих условию —4<х<1; 3) множество М всех элементов, входящих как в Pit так и в Р2. 5 Ф-73 65
7. На автобусной остановке можно сесть на автобусы, движущиеся по различным марш- рутам с интервалами движения 5, 2, 10 и 15 мин. Какова вероятность того, что первым по- дойдет автобус, следующий по маршруту с ин- тервалом движения 2 мин? Как кошка с собакой а) Построив графики следую- щих функций в прямоугольной системе коорди- нат, вы получите некое «произведе- ние искусства». Авали |цчсск<« выражение Функция Область определения fi 4s£x<8 8sgxs£9 9<х^10 ЮСх<16 16<х< 18 4<х=С5 5sgxsg8 8<х<10 11 sgx< 15 16s£x< 18 —-—<xsg7 2 ll<x<12 14s£xsgl5 x=8 </= -7Г+11 y = 2x—1 y—~ 7x4-80 y=x—6 0= -2x4-21 ’=4+4 0=-~6x-}-60 0=6 0==6x—96 2 51 У----5~ x+ T i/=-6x+72 0=6x—84 i/=I4 66
б) Перед вами «портрет» кошки. Найдите 14 линейных функций, описывающих отрезки, из которых составлена картинка. Для тех, кто готовится стать математи- ком Юный студент-математик обратился к из- вестному специалисту по теории чисел Э. Ландау, заявляя, будто ему удалось найти доказательство великой теоремы Ферма, утверждающей, что ни при каком целом п>2 нельзя найти 3 целых числа, х, у и z, которые бы удовлетворяли урав- нению xn4-^"=zn. Хотя математики всего мира на протяже- нии вот уже 300 лет пытаются доказать эту теорему, их усилия пока не увенча- лись успехом. Ландау терпеливо выслу- шал студента, затем усмехнулся и попро- сил его решить совсем простенькую ма- тематическую задачу, которую тут же продиктовал. Как ни старался студент, решить задачу Ландау ему так и не удалось. И тогда известный профессор дал ему ценный совет: «Прежде чем пы- таться потрясти основы науки, необхо- димо их изучить!» Ji* 67
8. Известная теоре- ма гласит: если плос- кость е пересекает две параллельные плос- кости а и р, то линии пересечения параллель- ны. & i'' Кто сумеет доказать эту теорему на языке теории множеств? 9. Куб рассечен плоскостью так, что сече- ние имеет форму равностороннего треугольни- ка, образованного диагоналями некоторых гра- ней куба. а) Начертить куб с проведенным сечением в косой проекции. б) Построить развертки обоих тел, на кото- рые распадается куб. в) Как называется меньшее из этих тел? 10. Перед нами коробка, вмещающая 160 круглых карандашей, если их укладывать в 8 рядов по 20 штук в каждом. Можно ли и как именно уложить каранда- ши по-другому, чтобы их вошло в коробку больше 160? 11. В одном проектном институте имеется 3 отдела: А, В и С. При обсуждении одного проекта руководство института условилось придерживаться следующего порядка. а) Если в обсуждении не принимает учас- тия отдел В, то отдел А также не участвует в обсуждении. б) Если в обсуждении проекта принимает участие отдел В, то отделы Л и С также уча- ствуют в обсуждении. Должен ли при этих условиях отдел С принимать участие в обсуждении проекта, если в обсуждении участвует отдел Л?
12. Пусть А, В и С — множества натураль- ных чисел, относительно которых известно следующее: (1) ЛЦВ={2, 3, 4, 5, 6, 7, 8}; (2) BJC= {1,2, 4, 6,8}; (3) С1|Л={1, 2, 3, 4, 5, 7, 8}; (4) ЛЛВ={2}; (5) ВПС={2, 4, 8} ; (6) СПЛ={2}; Какие числа принадлежат каждому из мно- жеств Л, В и С? Примечание. Под Л[)В принято пони- мать объединение множеств Л и В, т. е. мно- жество, содержащее те и только те элементы, которые принадлежат по крайней мере одно- му из множеств Л и В. Под ЛПВ принято по- нимать пересечение множеств Л и В, т. е. множество, содержащее те и только те эле- менты, которые принадлежат как множеству Л, так и множеству В. (И в объединении, и в пересечении каждый элемент включается только один раз.)* 13. Спичечный коробок имеет длины ребер а—17 мм, 6=37 мм и с=52 мм. Какой должна быть развертка упаковки для 10 коробков, чтобы расход оберточной бу- маги был минимальным? 14. В некоторой местности требуется про- ложить дороги между четырьмя населенными пунктами Л, В, С и D. При обсуждении воз- * Смысл обозначений теоретико-множественных опе- раций пояснен в примечании на с. 50.— Примеч. пер. 69
можных вариантов были высказаны следую- щие пожелания. а) Одним хотелось бы, чтобы из населен- ных пунктов А и В выходило по 3 дороги, из населенного пункта С — 2 дороги и из насе- ленного пункта D — 4 дороги. б) Другие отдавали предпочтение вариан- ту, в котором из каждого населенного пункта выходило бы по 3 дороги. в) Третьи считали более удобным, чтобы из населенного пункта А выходила одна доро- га, а из каждого из остальных населенных пунктов — по 2 дороги. Каждое ли пожелание могут учесть проек- тировщики? Должны ли дороги пересекаться?
ЖИИЯЙ на КАЖДОМ ШАГУ
Ни один человек еще не научился думать, читая в готовом виде записанные мысли другого человека. Научиться думать можно, лишь размышляя самостоятельно. Михай Эминеску* 1. Однажды мы невольно стали свидетеля- ми следующего разговора. — Правильно ли я тебя понял? Ты утверж- даешь, что состоишь членом шахматного клу- ба вдвое дольше, чем я. — Совершенно верно. — Но насколько мне помнится, ранее ты говорил, что состоишь членом шахматного клуба втрое дольше, чем я? — Два года назад? Но тогда мой стаж как члена клуба действительно был в 3 раза больше твоего, а сейчас лишь в 2 раза. Сколько лет каждый из двух собеседников состоит членом шахматного клуба? 2. По обычаю одной восточной страны же- не запрещается оставаться без мужа в об- ществе мужчин. Однажды трем супружеским парам понадобилось переправиться с северно- го берега реки на южный. Единственное под- ручное средство — лодка, вмещающая Двух человек. В какой последовательности они должны были переправляться, чтобы соблюсти стро- гий обычай? 3. При распродаже коврижек по 12 и по 17 марок за штуку в кассу за короткое время * Михай Эминеску (1850—1889) —румынский и молдавский поэт.— Примеч. пер. 72
поступило 478 марок. Коврижек каждого сор- та было распродано более 10 штук. Сколько коврижек каждого сорта было рас- продано? 4. Из 100 коллекционеров 70 собирают ста- ринные монеты, 75 — значки, 80 — этикетки со спичечных коробков и 85 — марки. Сколько из них увлекаются всеми четырь- мя видами коллекционирования сразу? 5. Одна швейцарская община насчитывает 50 членов. Родной язык всех 50 членов об- щины — немецкий, но 20 из них говорят еще и по-итальянски, 35 из них владеют французским и 10 не знают ни итальянского, ни француз- ского. Сколько членов общины говорят и по- французски, и по-итальянски? 6. Замечательный венгерский эстрадный вычислитель Ференц Патаки, способный за считанные секунды Иеремножить в уме 2 трехзначных числа, выступая в 1979 г. по те- левидению, продемонстрировал следующий фокус. «Умножьте свой размер обуви на 2, при- бавьте к произведению 39, умножьте получен- ную сумму на 50, прибавьте к произведению 29, вычтиТе из суммы год своего рождения»,— попросил Патаки. К удивлению зрителей каж- дый нз них получил четырехзначное число, две первые цифры которого задавали номер обуви, а две последние — возраст зрителя в конце календарного года. Объясните, на чем основан фокус Ференца Патаки. 73
7. Ящик можно обвязать веревкой тремя различными способами. В каком из трех случаев потребуется са- мая короткая и в каком самая длинная ве- ревка? (Размеры ящика удовлетворяют нера- венству а-}-Ь>2с.) 8. Гастрономическая задача. Рыбу прекрасно готовят тут, Форель отварная — король всех блюд. Вот принят заказ. Все готово. Несут! По порции рыбы на стол подают. Но что там за шум? То кричат повара, «Для порции нам не хватает стола, И по две на стол мы подать не смогли бы, Остался бы стол чей-то вовсе без рыбы». Вы назвать бы не сумели Нам число порций форели И количество столов Там, где хвалят поваров? 9. Хотя стоимость входного билета в цир- ке была снижена на 30%, выручка оказалась прежней. На сколько- процентов возросло число зри- телей? 74
10. Дерево отбрасывает тень длиной 10 м. Столб длиной 3 м отбрасывает тень длиной 2 м. Чему равна высота дерева? 11. Как выгля- дит шнуровка левой и правой туфли из- нутри? 12. Проехав треть пути, пассажир скоро- го поезда' наконец заснул. Когда он проснул- ся, до станции назначения оставалось про- ехать половину того расстояния,, которое от- деляло его от станции назначения, когда он заснул. Какую часть пути проехал поезд, пока пассажир спал? 13. Группе туристов нужно попасть из де- ревни А в деревню В. 75
Какой путь им выбрать, чтобы добраться побыстрее? (Числа на плане показывают, сколько минут необходимо затратить на про- хождение соответствующего участка пути.) 14. Какие из изображенных на рисунке деталей выпилены любителем головоломок и какие добавлены художником? Шедевр Математик и поэт, сочинитель эпиграмм Абрахам Готхельф Кестнер (1719—1800) в бытность свою студентом учился не- обычайно легко, словно играючи, и даже накануне выпускных экзаменов мог по- зволить себе вместо того, чтобы Сидеть дома, уткнувшись носом в книги, отпра- виться на прогулку с дочкой своего про- фессора. Когда же профессор упрекнул его в легкомыслии, Кестнер тотчас же возразил: «Господин профессор, вы же сами рекомендовали своим студентам неукоснительно следовать лучшим образ- цам. Вашу дочь я считаю совершен- ством». 76
15. Предлагаем вниманию читателя четы- ре примера на сложение, в которых цифры заменены буквами (одинаковые буквы соот- ветствуют одинаковым цифрам, различные — различным). Требуется восстановить цифро- вую запись всех 4 примеров (знать немецкий язык для этого не обязательно). а) , EBER, б) , SCHI, в) . AAL г) , 1СН Т ENTE, ’’’LIFT, + AAL + BIN + £ANS. SCHON. FANG LIEB T RABE. TI ERE, 16. 45% поступлений лотереи расходуется на выплату выигрышей. Сколько билетов стоимостью по 5 франков должны распространить организаторы лоте- реи, если на выплату выигрыша предполага- ется израсходовать 87 300 франков? 17. На заседании правления садового коо- ператива обсуждается вопрос о стоимости пригласительного билета на праздник урожая. Расходы на проведение праздника, по мнению членов правления, должны составить не ме- нее 420 марок. Ожидается, что в празднике примут участие 150 членов кооператива и 100 гостей. Расходы решено покрыть за счет продажи пригласительных билетов. Сборы должны составить сумму, несколько превы- шающую 420 марок. Члены правления счита- ют, что стоимость пригласительного билета для гостей должна превышать стоимость приг- ласительного билета для членов кооператива, но не более чем в 2 раза. Сколькими способами можно назначить це- ны на пригласительные билеты? 77
18. Сколькими способами можно разме нягь 1 марку, имея достаточный запас монет достоинством в 1 пфенниг, 5 пфеннигов, 10 пфеннигов, 20 пфеннигов и 50 пфеннигов? 19. Старинная задача на бытовые темы. Ее задавал своим ученикам учитель арифметики Якоб из Кобурга, чей учебник был напечатан в 1599 г. во Франкфурте. Расстояние между 2 городами составляет 260 миль. Из обоих городов навстречу друг другу выходят 2 гонца. Один из них ежеднев- но проходит на 2 мили больше, чем другой. Через 12 дней гонцы встречаются. Сколько миль проходит ежедневно каж- дый гонец?
слово ЗНАМЕНИТЫМ МАТЕМАТИКАМ
Ничего не сделано, если хоть что-то ос- талось недоделанным. Карл Фридрих Гаусс 1. Бхаскара I (VI в.). Найти натуральные числа, дающие при делении на 2, 3, 4, 5 и 6 остаток 1 и, кроме того, делящиеся на 7. 2. Брахмагупта (около 600 г.). Следующая задача заимствована из трактата этого индий- ского математика. «Если число дней умень- шить на 1, затем разделить на 6 и прибавить 3, то получится 7s первоначального числа дней». Сколь велико число дней? 3. Ал-Хорезми (около 780 г.—850 г.). Раз- ложить число 10 на 2 слагаемых, сумма квад- ратов которых равна 58. 4. Алку ин (около 800 г.). Король Карл был большим любителем наук и всячески поощрял их развитие. Беседы за его круглым столом нередко переходили в состязания на лучшее решение хитроумных головоломок. Самым из- вестным из участников этих собраний был ма- тематик Алкуин, ученый монах из Ирландии. Он написал несколько элементарных учеб- ников по математике. Однажды король и Алкуин отдыхали вме- сте после охоты, и Алкуин в шутку предложил королю прикинуть, за сколько прыжков его гончая настигнет зайца, если первоначально их разделяет расстояние 150 футов, заяц с каждым прыжком удаляется от собаки на 7 футов, а собака бежит быстрее зайца и с 80
каждым прыжком приближается к нему на 9 футов. Карл был не только искусным охот- ником, но и знал толк в арифметике. Что ответил король Алкуину? 5. Леонардо Пизанский (XIII в.)., Этот итальянский математик, известный под име- нем Фибоначчи, т. е. сына Боначчи, поставил в своей «Книге абака» следующую задачу. Имеется 5 гирь, позволяющих взвесить любой предмет с массой от 1 до 30 кг, если она выра- жается целым числом килограммов. Все гири при взвешивании разрешается ставить только на одну и ту же чашу весов. Какие гири понадобятся для, взвешивания 1 кг, 2 кг, .... 30 кг? 6. Абу-ль-Вафа (X в.). Два из трех равно- великих квадратов разрезать на 8 частей так, чтобы из них и из третьего равновеликого квадрата можно было составить квадрат боль- шего размера. 7. Бхаскара II (1114—1185). Одна треть, одна пятая и одна шестая цветов лотоса в венке посвящена богам Шиве, Вишну и Сурье, одна четвертая — Бхавани. Остальные 6 цве- тов предназначаются почитаемому праведни- ку. Сколько цветов лотоса сплетено в венок? 8. Адам Рис (1492—1559). Трое подмас- терьев хотели купить дом за 204 гульдена. На покупку первый дал втрое больше денег, чем второй, а второй дал вчетверо больше, чем третий. Сколько гульденов внес на покупку дома каждый из трех подмастерьев? 6 Ф-73 81
9. Мануэль Мосхопулос (околЬ 1453 г.). Этот ученый из Константинополя придумал следующую задачу: расположить числа 0, 1, 2, 14, 15 в вершинах внутреннего и внеш- него кубов так, чтобы сумма чисел, стоящих у вершин любой замкнутой четырехзвенной ломаной, была равна 30. 10. Иоганн Бутев (1549 г.). В его книге по математике под названием «Логистика» есть такая задача: «Если стоимость 9 яблок, умень- шенная на стоимость 1 груши, составляет 13 динаров, а стоимость 15 груш, уменьшен- ная на стоимость 1 яблока, составляет 6 ди- наров, то сколько, спрашиваю я, стоит 1 гру- ша и 1 яблоко?» 11. Георг Мор (1640—1697). Этот датский математик показал, как с помощью циркуля и линейки разделить данную окружность на 4 равные дуги. 12. Исаак Ньютон (1642—1727). В своей «Универсальной арифметике» Ньютон вы- сказал убеждение, что «при изучении наук при- 82
меры полезнее, нежели правила». В «Ариф- метике» Ньютона среди прочих есть и такая задача. На трех лугах площадью З'/з, Ю и 24 га трава растет одинаково, т. е. с одинаковой густотой и с одним и тем же приростом. Пос- ле того как на первом лугу 12 коров паслись 4 недели, а на втором лугу 21 корова паслась 9 недель, трава оказалась съеденной настоль- ко, что оба пастбища на время пришлось за- бросить. Сколько коров можно пасти на третьем лугу в течение 18 недель? 13. Альберт Эйнштейн (1879—1955). Даже став всемирно известным ученым, Эйнштейн продолжал предлагать вниманию читателей газеты «Франкфуртер цайтунг» различные за- дачи на сообразительность. Вот одна из них. Девять кружков образуют вершины 4 ма- лых и 3 больших равнобедренных треуголь- ников. Требуется вписать в эти кружки числа от 1 до 9 так, чтобы суммы чисел, стоящих в вершинах каждого из 7 равнобедренных треугольников, были равны. 14. Христиан Гольдбах (1690—1764) вы- сказал гипотезу о том, что любое четное чис- ло, большее 2, представимо в виде суммы 2 простых чисел. 6* 83
Проверьте гипотезу Гольдбаха для всех четных чисел, меньших 50. (Христиан Гольд- бах высказал свою гипотезу в письме Леонар- ду Эйлеру. Доказательство ее до сих пор не- известно.) 15. Сриниваса Рамануджан (1887—1920). Английский математик Г. Г. Харди однажды отправился навестить своего друга и сотруд- ника индийского математика С. Рамануджана в кебе с номером 1729. — Весьма скучное число,— заметил в раз- говоре Харди. — Напротив!—живо возразил Рамануд- жан.— Это очень интересное число: 1729— наименьшее из чисел, представимых в виде двух кубов двумя различными способами. Найдите оба разложения числа 1729 в сум- му двух кубов. 16. К. Ф. Гаусс (1777—1855) имел обыкно- вение кодировать свои записи. Знаменатель- ные события своей жизни он кодировал номе- рами дней, отсчитываемых с рождения до соответствующей даты. # Защитив 16 июля 1799 г. ученую степень доктора, Гаусс закоди- ровал эту дату числом 8113. Самым ранним знаменательным событием, отмеченным в личных записках Гаусса, был день, когда 15-летний Гаусс занялся проблемой распре- деления простых чисел. Это произошло на 5343-й день после его рождения, и дата вошла в записи под кодом 5343. На какой год, месяц и день приходится эта дата? 84
Пальто Эйнштейна Эйнштейн встретил как-то раз знакомого. — Господин Эйнштейн, вам непременно следует купить себе новое пальто!— по- советовал тот. — Зачем?— удивился Эйнштейн.— Ведь в этом городе меня никто не знает. Через несколько лет они снова встрети- лись в том же городе. Эйнштейн был в том же пальто, давно отслужившем свой век. Знакомый снова посоветовал Эйн- штейну купить новое пальто. — Зачем?— возразил Эйнштейн,— теперь меня здесь знает каждый.
НАША СЕКЦИЯ ИГР
Предмет математики столь серьезен, что не следует упускать ни одной возмож- ности сделать его более занимательным. Блез Паскаль 1. Построй мостик. Американский матема- тик Дэвид Гейл изобрел следующую игру. Игровое поле («до- ска») состоит из черных • • • • • точек, расставленных в • • |? |* |*—• вершинах квадратов, ко- , % || % торыми «выложена» часть , |__,| ,| | плоскости. Между ряда- •—| |?—J {• |* ми черных точек распо е ложены такие же ряды ,,| , ,| i , голубых точек. Играют • • • • |* • вдвоем: игрок А прово- • • • • • дит линии черным каран- дашом, игрок В — голубым Делая очередной ход, каждый игрок соединяет горизонтальным или вертикальным отрезком две соседние точки своего цвета в одном ряду. Игрок В стремится прозести сплошную черную линию, соединяющую левый край игрового поля с правым. Игрок А стремится соединить сплошной голубой линией верхний край игро- вого поля с нижним. Игроки совершают ходы по очереди. Выигрывает тот, кому удастся провести сплошную ломаную от одного своего края до другого — построить «мостик». На этом рисунке вы видите партию в игре Гейла, в которой победителем оказался игрок, делавший ходы голубым карандашом. Играть можно на «досках» любых размеров. 87
2. Тетракубы. Из обычных детских куби- ков, склеивая их гранями по 4, можно изго- товить набор так называемых тетракубов (некоторые из них изображены на рисунке). Предположим, что все тетракубы склеены из игральных костей. Для каких тетракубов сумма очков на видимых гранях наибольшая и для каких наименьшая? 3. Крестики-нолики. В эту игру играют вдвоем. Каждый из игроков стремится вы- строить (по горизонтали, по вертикали, или по диагонали) цепочку из 4 идущих подряд кле- ток. Ходы делают по очереди. За каждый ход игрок помечает I клет- ку. Чтобы клетки мож- но было отличать «по принадлежности», один игрок помечает клетки крестиком, другой — ноликом. Выигрывает тот, кто первым построит связ- ную цепочку из 4 кле- ток. На рисунке пока- зана партия в крестики-нолики, которую вы- играл тот, кто ставил крестики. 4. Хальма-соло. Цель игры—перевести 13 фишек, стоящих на серых полях, за возможно меньшее число ходов на голубые поля. Тот, кому удастся осуществить перестроение менее 88
чем за 20 ходов, может поздравить себя с ус- пешным началом. Не- много # потренировав- шись, попытайтесь осу- ществить перестроение всего за 13 ходов. Через фишку, стоя- щую на пути, разреша- ется перепрыгивать. 5. Лиса и гуси. Четырех «гусей» расставля- ют у верхнего края шахматной доски (доска может быть ориентирована любым образом — соблюдение шахматных правил не обязатель- но). Ходят гуси, как обычные шашки,— по диагонали на одно поле от своего края доски. На одном из полей у противоположного края доски гусей поджидает «лиса». Она ходит, как пешка, прошедшая в дамки,— по диагонали на любое число полей как Брать гусей запреща- ется. Лиса делает пер- вый ход и стремится достичь противополож- ного края доски. Ее противник (гусиное ста- до) выигрывает в том случае, если гуси смо- гут поймать лису пре- жде, чем она достигнет противоположного края доски. 6. Французская головоломка. Необходимые детали вырезают из дерева или картона (раз- меры каждый выбирает по своему усмотре- нию) и укладывают в коробку, днище которой имеет форму прямоугольника, стороны а и b которого относятся, как 4:5. Вырезать необ- 89
ходимо следующие детали: 4 квадрата со сто- роной а/4, 6 прямоугольников со сторонами а/4Хя/2 и 1 квадрат со стороной а/2. Извлекая из полного набора то одни, то другие детали, можно создавать различные головоломки на перестроение фигур. Приведем лишь 2 из них. а) Передвигая детали, но не вынимая их из коробки, перевести большой квадрат из уг- ла А в угол В. б) Передвигая детали, но не вынимая их из коробки, перевести большой квадрат А на место, занимаемое прямоугольником К и дву- мя малыми квадратами G и Н. 7. Пентамино. Каждая фигура пентамино состоит из 5 квадратов, склеенных сторонами. 90
8. Неусыпное наблюдение. На дорожках го- родского парка инспектор Леклерк расставил 6 полицейских так, чтобы те могли держать под наблюдением все дорожки парка. Один из полицейских стоит иа перекрестке 34. Где должны стоять остальные полицейские? 9. Магический квад- рат. В китайском ва- рианте игры в домино на клетках 3 костей встречается любое чи- сло очков от I до 9. , Составьте из этих костей магический квад- рат. 10. Как угадать исходы бросаний играль- ных костей. Иоганн Хемелинг, признанный придворный поэт и учитель арифметики и письма города Ганновера, в своей книге «На- чала арифметики» (1729 г.) предложил люби- телям такую задачу. Некто бросает 3 игральные кости. Ежели ты пожелаешь угадать, сколько очков выпало на каждой кости, то попроси его проделать следующее. Удвоить число очков, выпавших 91
на первой кости, а затем прибавить 5. Сумму умножить на 5, после чего прибавить к произ- ведению 10. К этому результату пусть он при* бавит число очков, выпавших на второй кос* ти, а сумму умножит на 10. Наконец, пусть к полученному результату прибавит число оч- ков, выпавших на третьей игральной кости. Попроси его назвать окончательный ре- зультат, вычти в уме 350, и по результату (трехзначному числу) ты сможешь сказать, сколько очков выпало на каждой кости (пер- вая цифра трехзначного числа совпадает с ис- ходом бросания первой кости, вторая — вто- рой и третья — третьей). (Примеры см. в решении.) 11. Лабиринт. Требуется пройти из А в В так, чтобы символы вдоль маршрута встреча- лись в той же последовательности, в какой они изображены слева от плана лабиринта (считая сверху вниз). в весе Ф эв в вс вш э’вя |Э в 3 с о ЕЕВЭВЕВВЕШЕ ведвЕвеввеви ВДДДЕБДВИЕВИ □ЕЕ1ВВЕ ЗВ ЕЕЫИВВВШ ЖйЩ 12. Играем в кости. Предположим, что мы бросаем 2 игральные кости: красную и белую. 92
Сколько различных исходов бросаний мы мо- жем получить? (Исход бросания, при котором на красной кости выпадает 1 очко, а на белой 4 очка, легко отличить от исхода бросания, при котором на красной кости выпадает 4 оч- ка, а на белой 1 очко: исходы этих бросаний различны.) Сколько различных исходов бросаний воз- никает в том случае, когда кости не отличают- ся по цвету?
sun, ВРЕМЯ, СКОРОСТЬ
Я люблю математику не только потому, что она находит применение в технике, но и потому, что она красива. Роже Петер 1. Кто не слышал об одном из самых быс- трых поездов мира? Называется он «Хикари» (звук) и курсирует на участке железной до- роги Токио—Явата протяженностью 1176,5 км. «Хикари» развивает наивысшую скорость око- ло 210 км/ч. Интервалы между поездами на магистрали составляют 12 мин. Вычислить среднюю скорость, развивае- мую экспрессом «Хикари» на участке Токио— Нагоя (отправление fe 6.00, прибытие в 8.03). Расстояние между этими двумя городами рав- но 366 км. Какую среднюю скорость развивает «Хи- кари» на всем участке от Токио до Яваты (от- правление в 6.00, прибытие в 13.01)? 2. Телевидение транслирует оперный спек- такль из миланского театра «Ла Скала» в Норвегию. Кто первый услышит начало увертюры к опере: зритель, сидящий в зале театра на рас- стоянии 25 м от сцены, или телезритель в норвежском городе Хаммерфесте? (Расстоя- ние Милан — Хаммерфест — около 2900 км, скорость звука 340 м/с, скорость распростра- нения электромагнитных волн 300 000 км/с.) 3. а) Парашютист пролетает в свободном падении, не раскрывая парашюта, 80 м. С какой скоростью он падает в момент раскрытия парашюта (сопротивлением возду- ха пренебречь)? 95
б) С какой скоростью входит в воду спорт- смен, прыгающий с 5-метрового трамплина? 4. Легкомысленный мотоциклист несется по улице большого города со скоростью 50 км/ч и прежде, чем успевает затормозить, врезается в «мертвое» препятствие. С какой высоты он должен был бы лететь в свободном падении, чтобы удар о землю ока- зался таким же, как при наезде на препят- ствие? 5. Камертон совершает гармонические ко- лебания. Их можно наблюдать и записать на фотопленке, если к ножке камертона прикре- пить зеркальце. При проведении такого экс- перимента с камертоном, совершающим ко- лебания с частотой 440 Гц, было записано 50 колебаний. Сколько времени длилась запись? 6. Во время летних Олимпийских игр 1976 г. в Монреале бегунья Иоганна Шаллер из ГДР пробежала 100 м с барьерами за 12,77 с. Ее результат был всего лишь на 0,01 с лучше, чем результат советской бегуньи Тать- яны Анисимовой. На сколько Иоганна Шаллер опередила Татьяну Анисимову на финише, если считать, что обе спортсменки бежали дистанцию с по- стоянной скоростью? 7. Две яхты принимают участие в гонке. Требуется пройти туда и обратно дистанцию 24 км. Одна яхта проходит всю дистанцию с постоянной скоростью 20 км/ч, другая про- ходит первую половину дистанции со ско- ростью 16 км/ч, а вторую — со скоростью 24 км/ч. Почему гонку выиграла первая яхта? 96
Вопросы на экзамене | В бытность студентом Гёттингенского 1 университета будущий лауреат Нобелев- ской премии по физике Макс Борн как-то раз сдавал экзамен астроному Карлу Шварцшильду. Между экзаменатором и экзаменующимся произошел следующий диалог. < Шварцшильд: Что бы вы стали делать, если бы увидели падающую звезду? Борн: Загадал бы желание. Шварцшильд: Прекрасно! А потом? Борн: Потом я взглянул бы на часы, за- метил время, определил, из какого со- звездия летит падающая звезда, направ- ление ее движения, длину оставляемого ею светящегося следа и т. д., а вернув- шись домой, вычислил бы в соответствую- щем приближении ее орбиту. Профессор не задал более ни единого вопроса: ответы экзаменующегося его вполне удовлетворили. Вокруг часов 8. На часах ровно 9. Через сколько минут стрелки часов (минут- ная и часовая) совпадут? 9. Минутная стрелка часов имеет длину 2 см, а часовая — 1,5 см. Найти отношение скоростей, с которыми движутся концы стрелок. 10. Сколько раз в течение суток минутная и часовая стрелки часов образуют прямой угол? 7 ф 73 97
11. Стрелки часов только что совпали. Через сколько минут они будут «смотреть» в противоположные стороны? 12. На часах Клауса минутные деления на- несены небольшими штрихами. Взглянув на часы в шестом часу пополудни, Клаус обнару- жил, что большая стрелка отстает от малой на 3 деления. Сколько было на часах? 13. Какова минимальная скорость движе- ния, при которой искусственные спутники не падают на Землю? Эту скорость, называемую первой косми- ческой скоростью для Земли, можно найти, предположив, что спутник обращается вокруг Земли непосредственно по ее поверхности (без учета трения). В этом случае радиус орбиты был бы равен 6378 км. Ускорение свободного падения можно принять равным 9,81 м/с2. 14. Плутон — самая дальняя планета на- шей Солнечной системы. Она находится на расстоянии около 5 910 000 000 км=5,91 млрд, км от Солнца. Сколько времени требуется свету, чтобы преодолеть это расстояние? (Скорость света в вакууме равна 300 000 км/с.) Выразить полученный результат в мину- тах и секундах. 15. Криптарифмы. Заменив буквы цифра- ми от 0 до 9, расшифруйте следующие приме- ры на сложение (одинаковым буквам соответ- ствуют одинаковые цифры, различным буквам 98
соответствуют различные цифры): , TEMPO , WEG + TEMPO + ZE IT + TEMPO ---------EI LE " HEKTIК (Обе надписи — значащие. Первый пример утверждает: «Темп-(-темп+темп=суета», вто- рой — «Путь4-время=спешка».) 16. Хутора А и В и городок С лежат на одной проселочной дороге (в той последова- тельности, в которой они перечислены). Из В в С ровно в б утра отправляется возница на телеге. Лошади идут со средней скоростью 10 км/ч. ZB тот же день в 7 ч утра из Л в С въезжает велосипедист, движущийся со сред- ней скоростью 15 км/ч. Сколько километров отделяет хутор В от городка С, если расстояние между хуторами А и В равно 5 км и велосипедист прибывает в С на 20 мин раньше возницы? Когда и на каком расстоянии от С вело- сипедист догоняет возницу? 17. Спортсмен-саночник уходит со старта с начальной скоростью 18 км/ч и движется с постоянным ускорением 0,8 м/с2. Через сколько секунд и на каком расстоя- нии от старта он разовьет скорость 90 км/ч? 18. При постоянном режиме работы ма- шины теплоход проходит некоторое расстоя- ние вверх по течению реки за 41/2 ч, а вниз по течению такое же расстояние за 3 ч. За какое время преодолеет то же расстоя- ние пустая бутылка, плывущая по течению реки? 7* 99
19. Машинист пассажирского поезда про- ехал туннель за 7 мин 30 с, а его коллега, водивший товарные поезда, проехал тот же туннель за 9 мин 30 с.- Скорость пассажир- ского поезда на 4 м/с больше, чем товарного. Какой длины туннель? 20. Два пассажирских поезда, следующих в противоположных направлениях, встречают- ся в пути. Скорость первого поезда составляет 45 км/ч, скорость второго — 36 км/ч. Пасса- жир второго поезда засек по часам, что встречный поезд промелькнул за окном всего за 6 с. Какой длины был первый поезд?
БЕСЕДЫ НА ЕСТЕСТВЕННОНАУЧНЫЕ
Существует еще одна причина, по кото- рой математику надлежит ценить высо- ко: именно математика придает естест- венным наукам степень достоверности, недостижимую без нее. Альберт Эйнштейн 1. Французскими специалистами (в сотруд- ничестве с их коллегами из СССР) разрабо- тан проект шара-зонда для исследования ат- мосферы Венеры. Требованиям, предъявляе- мым к оболочке шара, лучше всего отвечает трехслойный материал. Он состоит из алю- миниевой фольги на фторуглеродной основе, полиэфирной пленки и ткани из волокон ара- мида. Один квадратный метр этой легкой обо- лочки имеет массу 240 г. Для транспортиров- ки полезного груза диаметр шара должен со- ставлять 8 м. Велика ли масса оболочки. шара-зонда? 2. Пакет из 7 труб наружного диаметра 10 см необходимо связать как можно более короткой лентой. Чему равна минимальная длина ленты, идущей на обвязку пакета? (Припуск на сое- динение концов ленты в расчет не принима- ется.) 3. К клеммам полностью заряженной ак- кумуляторной батареи мопеда (6В; 4,5А-ч) присоединена лампа (6В; 0,6 Вт). Сколько времени будет гореть эта лампа, если к батарее не присоединены никакие другие нагрузки? 102
4. Какие силы действуют на нсрхний и нижний стержни и нити системы в точках Л, В L, М? 5. Футбольный мяч массой 700 г от удара приобретает скорость 18 м/с. С какой силой бьет по мячу футболист, если продолжительность удара считать рав- ной 0,02 с? 6. Время цикла ЭВМ составляет 1,3 мкс, т. е. для выполнения одной операции ЭВМ требуется 1,3 мкс. Какой частоте соответствует такое быстро- действие? Сколько операций в среднем может выполнить ЭВМ за 1 мин? 7. Микрокалькулятор — вещь необычайно полезная. За считанные доли секунды он вы- полняет разнообразнейшие математические операции. Но микрокалькулятор — прибор универсальный и может служить не только для серьезных дел, но и для забав. Вот, напри- мер, как можно с помощью микрокалькулято- ра проверить, знаете ли вы немецкий язык. Попытайтесь самостоятельно ответить на каж- дый из вопросов, а затем, нажав клавиши мик- рокалькулятора в указанной последователь- ности и повернув микрокалькулятор на 180° юз
(«вверх тормашками»), прочтите правильный ответ на индикаторе. Копытное животное, широко распростра- ненное в южных странах (7; 3; 5; 3). Приправа (3; 0; 0; 0; 0; +; 5; 5; 0; 5; =). Слово, противоположное по смыслу слову «темный» (7;-; 1; 0; 0; 0; +: 7; 3; 4; = ). 8. Двое рабочих переносят груз весом 981 Н (100 кг) на шесте. Расстояние от плеча одного рабочего до плеча другого -2 м. Груз подвешен на расстоянии 80 см от плеча рабочего, идущего сза- ди. С какой силой да- вит шест на плечи каж- дого из рабочих? 9. Какие из грузов /1, В, С и D поднимут- ся и какие опустятся, если рукоять повернуть в направлении, указанном стрелкой? 104
10. Пробковый спасательный круг весит 35,ЗН (3,6 кг). Велика ли его грузоподъемность? (Плот- ность пробки р=0,2 г/см3.) 11. Петер весит 35 кг (343 Н). При подтя- гивании на перекладине он поднимает свое тело на 38 см. а) Какую работу затрачивает Петер на подъем своего тела, подтянувшись 6 раз на перекладине? б) Какую работу затрачивает Петер, при- неся по просьбе матери ведро брикетов весом 10 кг (98,1 Н) из погреба на первый этаж (разность уровней 7,20 м)? в) В каком случае работа больше? 12. В ледниковый период ледник принес в Центральную Германию множество валунов. В 1903 г. при возведении памятника на мес- те Битвы народов под Лейпцигом жители ок- рестных мест принесли 100 валунов почти круглой формы. Из них воздвигли прямую квадратную пирамиду с основанием шириной 5 м и длиной ребра 6,1 м. Промежутки между камнями (составлявшие около 45% объема) для большей прочности заполнили бетоном. Сколько камня пошло на пирамиду, если его плотность составляет р=2,6 г/см3? Метод решения доведен до совершенства, если мы с самого начала в состоянии предвидеть и даже доказать, что он поз- воляет нам достичь поставленной цели. Г. В. Лейбниц 105
13. Криптарифмы. Расшифровать следую- щие арифметические примеры, заменив буквы цифрами (одинаковым буквам в каждом примере соответствуют одинаковые цифры, различным буквам — различные цифры). VOLVO ' FIAT MOTOR MOON MEN CAN REACH (1) РАДАР—PPP-PPP, (2) РАДАР—(PPP)A, (3) радар=(“-)а- («Вольво-|-Фиат=мотор», «Люди могут достичь Луны».) 14. Театральный бинокль имеет длину 14 см и дает пятикратное увеличение. Какое фокусное расстояние у его окуляра и объектива? 15. Диаметр заднего колеса велосипеда 70 см. Число зубцов в шестерне, вращаемой педалями, 46, в шестерне на втулке заднего колеса — 16. Сколько оборотов нужно сделать педаля- ми велосипедисту, чтобы, не используя холос- той ход, проехать 120 км? 16. Золото обладает замечательным свой- ством: его можно раскатать в тончайшие лист- ки толщиной примерно Vsooo мм (так называе- мое сусальное золото).* Сколько золота пойдет на изготовление 1 м2 сусального золота, если плотность золота р—19,3 г/см3? 106
17. 105 столбов нужно уложить в штабель 6 рядами так, чтобы в каждом ряду было на 2 столба меньше, чем в предыдущем. Сколько столбов следует уложить в осно- вание штабеля? 18. Сколько весит чемодан массой 25,00 кг на высоте уровня моря а) на широте 45° (g=9,81 м/с2); б) на экваторе (g=9,78 м/с2); в) на Северном полюсе (g=9,83 м/с2)?
НАПРЯЖЕННОЕ РАСПИСАНИЕ
Работайте и ищите, не надеясь на мо- литвы, и вы непременно найдете. Якоб Штейнер 1. Перед вами выписка из расписания за- нятий в пятых классах: Понедельник 5-й класс «А» 5-й класс «Б» 1-й урок Немецкий яз. География 2-й урок История Немецкий яз. 3-й урок Физкультура Физика 4-й урок География 'Рисование 5-й урок Физика Математика 6-й урок Рисование Биология Вторник 5-й класс «А» 5-й класс «Б» 1-й урок Физика Немецкий яз. 2-й *урок Математика Физика 3-й урок Математика Физкультура 4-й урок Немецкий яз. Математика 5-й урок Биология Немецкий яз. 6-й урок Физкультура — Определить, какие предметы преподают герр Райхельт, фрау Хельмерт, фрейлен Фи- шер и герр Вальтер, если известно следую- щее. а) Каждый учитель ведет по 2 предмета, б) Каждый учитель преподает оба своих 109
предмета и в 5-м классе «А», и в 5-м клас- се «Б». в) Фрейлен Фишер занята по вторникам и в 5-м классе «А» и в 5-м классе «Б» только в первые 2 урока. г) У герра Райхельта по вторникам биб- лиотечный день, так как он учится на заоч- ном отделении аспирантуры. д) Фрау Хельмерт проводит по понедель- никам только 2 урока в 5-м классе «Б», а в остальное время ее можно найти в учитель- ской. е) У учителя физики рабочий день по по- недельникам начинается с третьего урока. 2. Физкультура. На тренировке в школьном спортивном клубе велосипедист получил от тренера задание отработать частоту вращения педалей 120 об/мин при передаче 91,8. За сколько времени он проезжает отрезок в 200 м и какова его средняя скорость? (При передаче 91,1 велосипедист за один полный оборот педальной шестерни проезжа- ет 7,26 м.) 3. Русский язык. Составьте из следующих слогов слова, определения которых приведены ниже. Прочитав сверху вниз первые буквы слов, вы узнаете, как называется современ- ная наука, принадлежащая к числу матема- тических и широко применяемая в различных областях науки и техники: ал — ан — бра — вен — вер — ге — де — ди — е — же — и — ин — ир — ко — коб — куль — ла — ле — ма — мно — му — наль — не — ни — ность — пе — пре — ра — ре — ромб — са — си — ство — те — фор — ца — ци — цир — эдр — я. ПО
а) Доказываемое утверждение. б) Наименьшее натуральное число. в) Задание объекта. г) Четырехугольник с четырьмя равными попарно параллельными сторонами. д) Правильный многогранник с 20 граня- ми. е) Имя одного из членов знаменитого се- мейства математиков. ж) Геометрическое преобразование. з) Отношение между двумя числами. и) Математическое выражение, к) Действие. л) Угловая мера. м) Совокупность элементов. н) Раздел математики. о) Прибор для геометрических построений. п) Свойство чисел. р) Повторение одной или нескольких опе- раций. 4. Французский язык. , ONZE Sachant que: NEUF ONZE est divisible par 11, VINGT NEUF est divisible par 3, et VINGT est divisible par 5. 5. Экономическая география. В одном го- роде численность пенсионеров составляет 40% численности трудоспособного населения и 25% численности всех жителей. Какую часть всего населения составляют пенсионеры, Остальные взрослые, а также де- ти и подростки? 11)
По следам Якоба Штейнера Песталоцци принял будущего знамени- того геометра, которому тогда едва ми- нуло 18 лет, в свое учебное заведение, не взяв с него никакой платы. Очень скоро преподаватели обратили внимание на необыкновенные способности своего воспитанника.* Так, он мгновенно нашел решение следующей предложенной ему I геометрической задачи на построение. 1 6. Разделить правильный пятиугольник ! на 2 равновеликие части прямой, парал- лельной одной из его сторон. Эта задача далеко не простая. Просле- див шаг за шагом решение Штейнера, читатель сможет в полной мере оценить * ясность его мышления. Встречались I Штейнеру задачи и потруднее. Если за- дачу не удавалось решить сразу, воспи- I танник Штейнер упорно продолжал I искать решение до тех пор, пока его I усилия не приводили к успеху. Сохрани- I лась приписка Штейнера к решению од- ; ной задачи: «Найдено в воскресенье, i 10-го Христова месяца * 1814 г. в час । ночи; решал 34-3+4 часа». 7. Физика, а) Какое давление оказывает стоящий человек весом 588,6 Н (60 кг) на пол, если площадь его подошв составляет 150 см2? Предположим, что этот человек занима- ется зимними видами спорта. б) Какое давление он оказывает при ходь- * Декабря — прим, перев. 112
бе на лыжах на снежную поверхность, если длина лыж равна 2 м, а средняя ширина составляет 10 см? в) Округлив^ до ближайших целых чисел, найти отношение между давлением, оказы- ваемым стоящим человеком на пол, и давле- нием, оказываемым тем же человеком на снежную поверхность при ходьбе на лыжах. 8. Английский язык. A Dissection Puzzle: A triangle ABC has been dissected into parts X, Y, Z, along lines through M, the mid-point (centre) of AB that are parallel respectively perpendicular to the base BC. Show how the three pieces can be fitted together to make a rectangle, respecti- vely two different parallelograms. 9. Математика. Учитель, решив прове- рить знания своих учеников, предложил им решить следующую занимательную задачу: а—b — с d+e=f g+.h—' О числах а, Ь, ..., i известно, что йН-Л=/, а — сумма всех целых чисел от 2 до 193; &=22 - 52 - 72; 8 Ф-73 113
с — среднее арифметическое чисел 23 105, 13 830 и 4525; з,----- d^V 17 576, е —6,25% от 5248; f — удовлетворяет пропорции; 148 _ 34 112 . 37 f ’ g — число, квадрат которого равен 518 400; h — число, удовлетворяющее соотношению (ft-H)-13=59 488; i — наименьшее общее кратное чисел 4, 27 и 49. В книжном магазине Луиза: У вас действительно можно ку- пить любую книгу, которая может пона- добиться в школе? Продавщица: Конечно! Луиза: Тогда дайте мне, пожалуйста, сборник решений всех задач по матема- тике за 7-й класс! 10. Биология. Сердце тренированного спортсмена в течение коротких промежутков времени может развивать огромную мощ- ность. В моменты наивысшего напряжения сердце спортсмена совершает за минуту ра- боту порядка 932 Дж. Какова мощность, развиваемая сердцем в моменты наивысшего напряжения? 11. Химия. Памятная медаль состоит из сплава меди и серебра. Масса медали равна 20,9 г, а ее объем составляет 2123 мм3. 114
Из скольких долей серебра и меди (в ра- счете на 1000) состоит сплав, из которого из- готовлена памятная медаль? (Плотность се- ребра 10,5 г/см3.) 12. Астрономия. На какую высоту над поверхностью Земли следует запустить искус- ственный спутник, чтобы он мог «висеть» над одной и той же точкой? (Воспользуйтесь одним из законов Кеп- лера, а в качестве тела сравнения — Луной. Период обращения Луны вокруг Земли равен примерно 27,33 суток, средний радиус лун- ной орбиты составляем 384 000 км.) 13. География. Однажды ребята всем клас- сом отправились в поход по горам. Один из школьников начертил профиль рельефа вдоль маршрута, а снизу на узких полосках изобра- зил возвышенности темными участками, а до- лины светлыми. Его одноклассники сразу же нашли, какая из полосок /1, В. G соответ- ствует изображенному ниже горному массиву. А вы? 8*
вошг ЦИРКУЛЯ и линеани
То, чем в прежние эпохи занимались лишь зрелые умы ученых мужей, в бо- лее поздние времена стало доступно по- ниманию мальчишек. Гегель 1. Астроном Г. К. Шумахер из Альтоны сообщил 22 сентября 1836 г. своему другу К. Ф. Гауссу, что получил от гамбургского астронома К. Л. Румкера следующую задачу вместе с решением. Дан эллипс и в плоскости эллипса вне его точка Р. Не пользуясь циркулем, провести из точки Р касательные к эллипсу. Решение Румкера. Проведем через точку Р любые 4 прямые, пересекающие эллипс. Пусть А и Л', В и В', С их пересечения с эл- липсом. Соединим от- резками прямых точки А и В', В и А', С и О', D и С'. Пусть Q — точ- ка пересечения прямых АВ' и Л'В, R — точка пересечения прямых CD' и C'D. Прямая, проходящая через точ- ки Q и /?, пересекает эллипс в точках Е и F. Точки Е и F принадле- жат касательным, про- веденным к эллипсу из точки Р. и С'. D и D' — точки 117
В письме к Гауссу Шумахер добавил, что Румкер провел слишком много прямых и что можно обойтись тремя прямыми, проходящи- ми через точку Р и пересекающими эллипс. Через несколько дней пришел ответ от Гаусса. По мнению великого математика, Румкер действительно провел слишком много прямых, и можно было бы обойтись тремя прямыми. Но и трех линий слишком много: достаточно двух прямых, проходящих через точку и пересекающих эллипс. 1 Располагая этими сведениями, найдите предложенное Гауссом решение. На какой геометрической теореме оно основано? 2. Выдающийся польский физик Леопольд Инфельд в своей книге «Э. Галуа — избран- ник богов» рассказал об, одном эпизоде, про- исшедшем с главным героем, когда шестнад- цатилетний Галуа учился в Колледже Людо- вика Великого. Ученики, слушавшие допол- нительный курс математики, получили зада- ние на неделю, которое, как это часто бывает, показалось им очень трудным. В первой за- даче требовалось по известным сторонам а, Ь, с и d вписанного в окружность четырех- угольника найти его диагонали х и у. В задание входили еще две задачи. К ве- ликому удивлению преподавателя математи- ки, полагавшего, что уча- ___ щимся придется затра- с \с тить на выполнение за- //\ дания не один час, Галуа // \ решил все три задачи за |/ ь] 15 мин! / Сможете ли вы ре- АV ’—/ шить первую задачу? Она \. Ix" не так уж проста. --- 118
3. Среди глав государств своего времени Наполеон Бонапарт (1769—1821) был един- ственным, кто получил научное образование. Он принимал участие в заседаниях Париж- ской академии. Во время похода в Египет вместе с наполеоновской армией на берега Нила отправилась научная экспедиция. На- полеон также немного был и математиком. В частности, его интересовала геометрия. По преданию, ему удалось сформулировать и до- казать приводимую ниже теорему (хотя ныне авторство Наполеона представляется нам сомнительным). Не вызывает сомнения то, что Наполеон интересовался задачами тако- го рода и предлагал их математикам. Напо- леону посвящен известный английский па- линдром (фраза, читаемая одинаково слева направо и справа налево): ABLE WAS I ERE I SAW ELBA. (Я многое мог прежде, чем увидел Эльбу). Теорема Наполеона. Пусть на сторонах произвольного треугольника АВС вне его по- строены равносторонние треугольники АКВ, ВРС, CQA. Тогда их центры Qt, Qj. и Q3 располагаются в вершинах равностороннего треугольника. Докажите эту теорему. 4. Вычислить площадь каждой фигуры. 2а 119
С помощью циркуля и чертежного угольника Невозможные фигуры из голландского ма- тематического журнала для школьников «Пи фагор» (20
5. Чему равна (в единицах длины, кото- рые выбраны по осям координат) длина сто- роны квадрата, равновеликого четырехуголь- нику с вершинами в точках (1. 0), (17, 0). (13, 12), (0, 7)? 6. От куба отсечены 8 частей, имеющих форму треугольных пирамид (тетраэдров). Какую долю от объема всего куба состав- ляет объем оставшейся части VOt , ? 7. Периметр прямоугольника равен 40 м. Его длина х и ширина у отличаются не менее чем на 2 м. 12)
Чему могут быть равны наименьшая дли- на и наибольшая ширина такого прямоуголь- ника? 8. Дан угол 63°. С помощью циркуля и линейки разделить его: а) на 3 равные части; б) на 7 равных частей. 9. На каждой стороне квадрата (длиной а) вне его построено по равнобедренному тре- угольнику, равновеликому квадрату. Вычислить расстояние между дальними вершинами треугольников, построенных на противоположных сторонах квадрата (вся фигура напоминает 4-конечную звезду). 10. Даны два правильных многоугольника. Число сторон второго многоугольника вдвое больше числа сторон первого. Каждый внут- ренний угол первого многоугольника на 10° меньше любого внутреннего угла второго. Найти число сторон каждого из двух мно- гоугольников и их внутренние углы. 11. Задача на сообразительность из пере- дачи венгерского телевидения. Возьмем вы- пуклый п-угольрик. Предположим, что ника- кие три из его диагоналей не проходят через одну точку. В скольких точках пересекаются диагона- ли нашего п-угольника? (Вершины п-уголь- ника и расположенные вне его точки пересе- чения прямых, на которых лежат диагонали, в число точек пересечения диагоналей не входят.) 122
12. Доказать, что полупериметр произ- вольного треугольника всегда больше любой из его сторон. 13. Одну из сторон прямоугольника уве- личили на 25%. На сколько процентов следует уменьшить другую сторону, чтобы площадь прямоуголь- ника осталась неизменной?
ИГРА числами
Хорошая математическая шутка всегда лучше целой дюжины посредственных математических работ. Дж. И. Литлвуд 1. От башни нуж- но пройти в правый нижний угол за 10 шагов, набрав по дороге сумму в 60 очков (суммируются очки, проставленные на тех полях, по ко- торым проходит маршрут). Кто сумеет бы- стрее других найти дорогу? 2. Равенство двух дробей, в записи кото- рого использованы все 10 цифр, а сами дро- би принимают наименьшие из возможных значений, имеет вид 1 = 2 4865 9730 Как выглядит равенство двух дробей, в за- записи которого также использованы все 10 цифр, если дроби принимают наибольшее из возможных значений? 3. В комплект домино входит 28 костей. Из 18 костей составлен магический квадрат с суммой очков по всем горизонталям, верти- калям и диагоналям (магической постоян- ной), равной 13. 125
Из 8 костей домино можно составить ма- лый магический квадрат с магической посто- янной, равной 5, Как это сделать? 4. В каждом из секторов А, В, С и D «ми- шени» числа вписаны в кольца по определен- ному закону. Тот, кто отгадает его, сможет вписать числа в «яблочко» вместо вопросительных знаков 126
5. Упростите дроби в следующих числовых последовательностях. Если удастся, попытай- тесь найти общий член каждой последова- тельности. 2 . 3 . 4 5 , 2 ’ 2+4 * 2+4+6 ’ 2+4+6+а ’ 2 . 2+4 . 2+4+6 . 2+4+6+в . 3 * 3+5 ’ 3+5+7 * 3+5+7+9 ’ 1 . 1+2 . 1+2+3 . 1+2+3+4 . 1 ’ 1+3 ’ 1+3+5 ’ 1+3+5+7 ’ 6. Перед вами лабиринт. Темные кольца— стенки, светлые — коридоры между ними. Проходы в стенках проложены только в тех местах, где стоят числа («номера» проходов). Сумеете ли вы пробраться внутрь лаби- ринта, набрав по дороге сумму в 500 очков? (Стоя снаружи, вы имеете 0 очков. Минуя очередной проход, вы прибавляете его «но- мер».) 127
Числовые узоры 5 5 5 5 5 5 52 2 5 2 5 2 5 2 5 2 5 2 5 25252525 2525252525 252525252525 25252525252525 252525252525 2525252525 25252525 2 5 2 5 2 5 2 5 2 5 2 5 1-1 = 1 11-11 = 12 1 111-111 = 1 2 3 2 1 1111-1111 = 1 2 3 4 3 2 1 11111-11111 =1 2 3 4 5 4 3 2 1 30864191358025 1 + 3 = 22 1 + 3 + 5 = З2 1+3+5+7=42 1+3+5+7+9=52 1+ 3 + 5 + 7 + 9 + 11= 62 1+3 + 5 + 7+9 + 11 +13 = 72 7 4 4 9 4 72 9 4 9 9 6 6 62 3 6 3 6 3 6 3 6 3 6 3 6 3 6 3 6 3 6 2 2 5 2 5 2 5 2 5 2 5 2 5 2 5 2 5 2 5 5 2 5 2 5 2 5 52 5 2 5 5 2 5 2 5 5 5 9 2 9 4 4 3 5 5 6 3 0 8 5 8 0 2 5 7. Разделить изображенную на рисунке двухрядную полоску на четыре конгруэнтные части так, чтобы сумма чисел в клетках каж- дой части была равна 34 1 9 16 7 12 5 4 3 8 15 10 2 13 6 11 14 128
Арифметические курьезы Заметив закономерность в каждой из по следовательностей чисел, продолжите их. 72=49 672=4 489 6672=444889 1 23456789- 9=111 111 111 12345678 9-18=222 222 222 1 2 3 4 5 6 7 8 9-27=333 333 333 62=36 762=5776 3762=141376 (101-65)- 36=1296 (65-361-101=2929 (101-36)- 65 = 4225 42:3=4-3+2 85 — 63=8+5+6+3 4-23=34 —2 1-9+2= II 12-9+3= 111 123-9+4=1111 9-9+7= 88 98-9+6= 888 987-9+5= 8888 1^121 = 12—1 У'в£=ъ+У1~ з,---- V 1331=3+1+3+3+1 (533 — 456=77) (552 — 474=78) 773=456 533 783=474 552 151+264= (13+53+13) + (23+63+43) 1233=122+332 8833=882+332 8. Вычислите и вы удивитесь! Xi =900 991-863 247 х2=803 -202 -137 х3=689 976: 888 х4= (379+888) — (477+124) 9 Ф-73 129
х5= (2997-729) : (81-81) х6—412-|-432-+-452 <б.5)-И5-5) + (5:5) 7 ^5 ..5) + (5+5) —(5:5) 9. Венгерский эстрадный вычислитель Па- таки поставил трем своим партнерам из зри- телей А, В и С следующую задачу. Партнер. А должен был задумать одно произвольное четное, одно произвольное не- четное число и сообщить одно из них партне- ру В, а другое — партнеру С. Партнер В дол- жен был умножить свое число на 2, а партнер С свое число должен был умножить на 3. По- лученные произведения партнеры В и С долж- ны были сложить и назвать вслух сумму. Патаки по сумме сразу же назвал, кому из двух партнеров В и С партнер А сообщил четное и кому нечетное число. На чем основан этот фокус? 10. Числа 12 и 60 обладают интересным свойством: их произведение в 10 раз больше нх суммы. 12-60=720, 124 60= 72. Существуют ли другие пары натуральных чисел, обладающих тем же свойством? 11. Над двумя различными натуральными числами проделывают четыре операции: а) находят их сумму; б) из большего числа вычитают меньшее; в) находят произведение двух исходных чисел; г) большее число делят на меньшее. Сумма результатов всех четырех операций оказывается равной 243. 130
Что это за числа? (2. С помощью знаков арифметических действий и скобок представить каждое из чисел 1, 2, 10 с помощью четырех семерок, например (7-|-7*7) : 7=8. 9*
огонь математических олимпиад
С любознательности начинается позна- ние мира. Именно она составляет наибо- лее характерную и значительную осо- бенность юности, когда формируется личность и знания усваиваются особен- но быстро и прочно. Без любознатель- ности, по моему мнению, человек не мо- жет развиваться нормально. Л. Д. Ландау С 1968 г. проводятся международные ма- тематические олимпиады. Ежегодно в этих соревнованиях принимают участие по 8 луч- ших школьников более чем из 20 стран. Не- которые из приведенных ниже занимательных задач заимствованы из задач, предлагавших- ся на олимпиадах. 1. Перед началом бегов на ипподроме че- тыре знатока из числа зрителей обсуждали шансы фаворитов Л, В и С. (1): «Заезд выиграет А или С». (2): «Если Д будет вторым, то выигры- вает В». (3): «Если А придет третьим, то С не вы- играет». (4): «Вторым придет А или В». После заезда выяснилось, что три фаво- рита Л, В и С действительно заняли первые три места и что все четыре утверждения зна- токов Оказались истинными. Как фавориты поделили между собой три первых места? 2. Два школьника Эпсилон и Дзета бесе- дуют между собой на досуге. 133
Эпсилон: Я могу представить число 30 в виде арифметического выражения, содер- жащего только 3 пятерки и знаки основных действий. Дзета (после короткого размышления): Ничего удивительного! При любом натураль- ном //>2 число 30 можно представить в виде арифметического выражения, содержащего ровно п пятерок и знаки основных действий. Как Дзета может доказать свое утвержде- ние? 3. Из квадратного куска картона со сто- роной 8 дм нужно вырезать 9 неконгруэнтных разверток куба. Каждая развертка должна складываться в куб с ребром длиной 1 дм. Доказать, что задача разрешима, и рас- кроить лист картона, начертив на нем 9 не- конгруэнтных разверток куба нужного раз- мера. 4. Фамилия великого математика содер- жит пять букв. Если буквы алфавита А, Б, В, ...» Ю, Я (без Е) перенумеровать по поряд- ку идущими подряд числами от 1 до 32 и вместо букв, входящих в фамилию матема- тика, подставить их «номера», то окажется, что сумма чисел, соответствующих (1) первой и второй буквам, равна 40; (2) первой и третьей буквам, равна 42; (3) первой и четвертой буквам, равна 36; (4) первой и пятой буквам, равна 47; (5) всем пяти буквам, равна 75. Назовите фамилию великого математика. 5. Обсуждали высказывания, которые на- чинаются словами: «Если а и b — два отлич- ных от нуля действительных числа, таких, что а^>Ь и |а|<|/?|, то...» 134
А предложил закончить словами «...число а отрицательно». В предложил закончить словами «...а и b — отрицательные числа». С предложил закончить словами «...число b отрицательно». D предложил закончить словами «...ни число а, ни число b не должно быть отрица- тельным». Какие из получившихся полных высказы- ваний истинны и какие ложны? 6. На рисунке вы видите четыре концен- трических круга. Примем площадь внутрен- .него круга за единицу. Кольцо, заключенное между границами внутреннего и следующего круга, разделено на две конгруэнтные части, помеченные номерами 2 и 3. Следующее коль- цо разделено на четыре конгруэнтные части, получившие номера 4, 5, б и 7. Наконец, по- следнее кольцо разделено на 8 конгруэнтных частей с номерами 8, 9, ..., 14, 15. Как следует выбрать радиусы четырех кругов, чтобы все 15 частей были равновели- кими? 135
7. Вот треугольник не простой, В нем угол АС В — прямой. А теперь условий узы: с—длина гипотенузы, р — величина не меньшей нужности — Радиус вписанной окружности. Задача, посильная для школьника: Выразить через них площадь треуголь- ника. 8. Два игрока А и В по очереди берут из большого короба, вмещающего ровно 150 спи- чек, не менее 1 и не более 10 спичек. Кто из игроков может обеспечить себе вы- игрыш? Какая стратегия ведет к выигрышу? Тот, кто хоть раз пережил радость твор- ческого свершения, не пожалеет никаких усилий, чтобы пережить ее снова. Ника- кие трудности не остановят его. Его стремление, старание и упорство в пре- одолении препятствий будут возрастать с каждым новым успехом. _А. Я. Хинчин 9. Дано конечное множество черных и голубых точек. Некоторые точки соединены отрезками прямых. Точка множества называ- ется «необыкновенной», если более половины исходящих из нее отрезков заканчиваются в точках другого цвета, чем она. Если в дан- ном множестве точек имеются необыкновен- ные точки, то, выбрав любую из них, пере- красим ее в другой цвет. То же проделаем и 136
с полученным множеством точек, если в нем существуют необыкновенные точки, и т. д. Доказать, что любое конечное множество черных и голубых точек при любом выборе необыкновенных точек, подлежащий перекра- шиванию в другой цвет, после конечного числа перекрашиваний перейдет в множество, не содержащее необыкновенных точек. 10. По преданию, основательница чешско- го государства принцесса Либуша обещала отдать свою руку тому из трех женихов, кто сумеет решить следующую задачу: «Если бы я дала первому жениху половину слив из этой корзины и еще одну сливу, второму жениху половину оставшихся слив и еще одну сливу, а оставшиеся сливы поделила пополам и. по- ловину их и еще три сливы дала бы третьему жениху, то корзина опустела бы». Сколько слив в корзине? 11. Гюнтер рассказывает: «Шестизначный номер телефона своей школы я запоминаю следующим образом. Записываю двузначный номер дома, в котором живу, к нему припи- сываю справа сумму его цифр, а затем при- писываю справа сумму двух последних цифр до тех пор, пока не получится шестизначное число. Оно-то и будет номером телефона моей школы. Должен еще сказать, что в номере .теле- фона моей школы неу ни одной единицы, а номер дома, где я живу, делится на 3». Назовите номер дома, в котором живет Гюнтер, и номер телефона его школы. 12. В одном старинном учебнике арифме- 137
тики упоминается о следующей торговой сделке. Крестьянин хотел купить у торговца не- сколько голов скота. Торговец запросил за всех одинаковую цену. Крестьянину удалось сбить цену на столько же процентов, сколько грошей он должен был бы уплатить за каж- дую голову скота первоначально. Торговцу скотом за каждую голову кре- стьянин уплатил 21 грош. При первоначаль- ной цене денег хватило бы на покупку трех голов скота. После скидки крестьянин купил больше скота, израсходовав на покупку все евои деньги. Сколько голов скота купил крестьянин?

Каждая задача, которую я решал, ста- новилась правилом, служившим впослед- ствии для решения других задач. Рене Декарт 1. Монголия^ В шахматном турнире участ- вовало 10 шахматистов. Каждый сыграл с каждым по одной партии. Все участники тур- нира набрали различное число очков. Шах- матисты, занявшие первое и второе места, не проиграли ни одной встречи и набрали на 10 очков больше, чем шахматист, занявший третье место. Шахматист, занявший четвертое место, набрал столько же очков, сколько на- брали вместе шахматисты, занявшие четыре последних места. Сколько очков набрали шахматисты, за- нявшие с первого по шестое места? (За вы- игранную партию шахматист* получает 1 очко, за партию, закончившуюся вничью, оба парт- нера получают по ^2 очка. Шахматист, про- игравший партию, получает 0 очков). 2. Исландия. Неподалеку от деревни рас- положился летний лагерь спортклуба. В пер- вый день 28 обитателей лагеря приняли уча- стие в соревнованиях по прыжкам в длину, прыжкам в высоту и прыжкам с шестом. Каждый из 28 спортсменов выступил по край- ней мере в двух видах программы. 8 спорт- сменов состязались в прыжках в длину. В прыжках в длину и в прыжках в высоту участников оказалось на 3 больше, чем в прыжках в высоту и в прыжках с шестом. Спортсменов, выступивших в двух номерах программы—и в прыжках в длину, и в ярыж- но
ках в высоту, было столько же, сколько пры- гунов с шестом. Сколько спортсменов выступило во всех трех номерах программы? Попробуйте решить задачу с помощью диаграмм Венна. 3. Польша. Между пятью городами, из которых никакие три не лежат на одной пря- мой, требуется проложить сеть железных до- рог из четырех прямолинейных отрезков. Же- лезнодорожные пути могут пересекаться: в местах пересечений будут построены путе- проводы. Сколько существует вариантов прокладки железнодорожной сети? 4. Англия. Шесть фигурок расположены следующим образом: Фигурки могут перепрыгивать через одну фигурку, продвигаться вперед, но не могут поворачиваться, ходить назад или отклонять- ся в сторону. За какое минимальное число прыжков или шагов фигурки можно перестроить в следую- щем порядке? 141
Эти выразительные математические тер- мины построены по образцу и подобию тех, которые предложил своим читателям голланд- ский математический журнал для школьни- ков «Пифагор». биение сзИнипс або/га огпраснсение ' трОпеция корень порань betmop гХпербо/id 5. Швейцария. В «Солитер», как говорит само название (от франц, solitaire — одино- кий, уединенный, отшельник) играют в оди- ночку. Доска для этой игры состоит из 37 осо- 142
бым образом расположенных квадратных полей. Перед началом игры на всех полях, кроме одного, выбираемого «отшельником», расстав- ляются фишки. Каждый ход состоит в том, что играющий переставляет одну фишку по горизонтали или вертикали через занятое поле на свободное и снимает с доски ту фиш- ку, через которую перепрыгнула фишка, со- вершившая очередной ход. (Если поля 02 и 03 заняты, а роле 04 свободно, то фишку можно переставить с поля 02 на поле 04, сняв с доски фишку, стоявшую на поле 03.) После каждого хода на доске становится на одну фишку меньше. Цель игры состоит в том, что- бы оставить на доске только одну фишку. 6. Канада. Решите эти два весьма остро- умных криптарифма: , MIX J.ALORS Т FUN TaLORS t* AND + NOUS ---MATH + NOUS L’AVONS 7. Танзания. Каникулы Мбонго продолжа- лись [ дней. По его наблюдениям: (1) дождь шел 7 раз либо с утра, либо к вечеру; (2) если дождь шел к вечеру, то с утра стояла солнечная погода; (3) 5 раз к вечеру устанавливалась сол- нечная погода; (4) 6 раз с утра стояла солнечная погода. Сколько дней продолжались каникулы Мбонго? 143
Сведения не подтвердились На лекциях по теории групп Феликс Клейн (1849—1925) имел обыкновение рассказывать своим слушателям следую- щую историю. На знаменитом Математическом кон- грессе, состоявшемся в Париже в 1900 г., во время торжественного заседания бы- ли помянуты все выдающиеся матема- тики, скончавшиеся за последние 10 лет, предшествовавших конгрессу. Среди про- чих председательствующий назвал спе- циалиста по теории групп профессора Политехнической школы Камилла Жор- дана, сообщив, что тот родился в 1838 г. и умер 7 ноября 1898 г. Неожиданно в задних рядах поднялась высокая фигура и сообщила ученому собранию, что по крайней мере дата смерти названа явно неверно, поскольку Жордан (а это был он) еще жив. (Камилл Жордан умер 20 января 1922 г. в Милане). 8. Испания. 2=3? «Доказать* это «равенство» можно сле- дующим образом: 4—10=9—15, 4-10+ 2L=9--15+ Л5., 4 4 144
2=3. Где ошибка? 9. Румыния. В одном доме жило несколь- ко супружеских пар с детьми. О них извест- но, что всех детей было больше, чем взрослых, а взрослых — больше, чем мальчиков. В свою очередь, мальчиков было больше, чем дево- чек, а девочек — больше, чем семей. В каждой семье был по крайней мере один ребенок, а число детей во всех семьях было различным. У каждой девочки был по крайней мере один брат и не более одной сестры. В одной семье детей было больше, чем во всех остальных семьях вместе. Сколько семей жило в доме? Сколько де- вочек было в каждой семье? Трудная проблема Во время одной из лекций знаменитому берлинскому математику Ф. Э. Куммеру (1810—1893) понадобилось решить труд- ную задачу: вычислить 7-9. Он обра- тился за помощью к слушавшим его лек- ции студентам. Один назвал число 62, другой — число 65. — Господа!— возразил профессор Кум- мер.— Число 7-9 не может быть одновре- менно равно 62 и 65. Что-нибудь одно! 10. ГДР. Ганс попросил своего друга Уве: — Задумай какое-нибудь отличное от нуля целое положительное число, умножь его на 5 н прибавь к найденному произведению 2. За- тем умножь полученную сумму на 4, прибавь (0 Ф-73 145
к новому произведению 3, а сумму умножь на 5. Если ты назовешь мне результат, то я тут же скажу тебе, какое число ты задумал. Объясните, на чем основан арифметиче- ский фокус, показанный Гансом своему прия- телю Уве.
Часть 2 РЕШЕНИЯ 148 153 160 166 173 180 189 195 204 213 220 222 230 236 245 253 256 263 10* Пестрые картинки из разных стран " Античные этюды 17 Школьные истории 25 Старое и новое из практики 32 Минуту на размышление 39 Арифметика с птичьего полета 47 Занимательная геометрия 55 Тренировка по современной матема- тике 63 Математика на каждом шагу 71 Слово знаменитым математикам 79 Наша секция игр 36 Путь, время, скорость 94 Беседы на естественнонаучные темы 101 Напряженное расписание 108 Вокруг циркуля и линейки Н6 Игра с числами 124 Огонь математических олимпиад 132 По разным странам 139
ПЕСТРЫЕ КАРТИНКИ ИЗ РАЗНЫХ СТРАН 1. Пусть х, у, z и и — число бутылок лимо- нада, которое выпили соответственно жены месье Пона, Дюбуа, Пейзана и Фонтена. Все- го жены выпили *+y+z+w=10 бутылок лимонада. Их мужья выпили x4-2«/-J-3z4-4m бутылок лимонада, а четыре супружеские па- ры вместе опустошили 2x4-3//4-4z+5m=32 бутылки лимонада. Подставляя^ и — 10—х—у—z, получаем 18=3 х 2у -J- z. Числа х и г должны быть либо оба четными, либо оба нечетными. Значения х=1 и х=2 отпадают, так как каждое из чисел у и z не превосходит 4. При х—4 мы получили бы г—2 и у=2, что невозможно, так как х, у, г пи — различные числа. Следовательно, зада- ча допускает единственное решение: х=3, z=l, у=4, и=2\ х=3 (Колетта Пон), у=4 (Анетта Дюбуа), г—1 (Жанна Пейзан), и=2 (Жаклин Фон- тен). 2. Пусть х-- число стоящих, у—число ле- жащих молодых буйволов иг — число старых буйволов. Тогда х-НН-z=100, (1) 5х+3у+ 4- = 10°, (2) О </=25- 4 148
Так как х и у — натуральные числа, послед- нее равенство выполняется только при х=0, 4, 8, 12. Задача допускает поэтому следующие четыре решения: х у z О 25 4 18 8 11 12 4 75 78 81 84 4. Пусть а и b различные цифры. Тогда из них можно составить следующие надписи на километровых столбах: (aab) или (9 — а, 9 — а, 9 — &); (aba) или (9 — а, 9— ft, 9“ а)\ (baa) или (9 — Ь, 9—а, 9 — а); (ааа) или (9~а> 9 — а, 9—а); (bbb) или (9 — Ьч 9 -Ь, 9 — Ь). Так как по условию задачи различными должны быть только 2 цифры, то всюду дол- жно выполняться равенство Ь — 9 — а. Пере- числить все 40 возможных случаев после того, как установлено это соотношение, не пред- ставляет труда. 149
5. Хитрый крот устроил запасную кладо- вую между камерами 10 и II. 6. Так как сумма последних цифр 2+3+ +3+4=12 оканчивается на 2 и не существу- ет квадрата натурального числа, который бы оканчивался на 2, речь идет не о четырех, а лишь о трех рыболовах, т. е. сын одного из любителей рыбной ловли одновременно явля- ется отцом другого^ (2+3+4=9). Николай не может быть сыном Петра, так как улов Ни- колая оканчивается на 2, а не на 4, как того требуют условия задачи. Следовательно, Петр сын Николая. 7. Результаты взвешивания уловов а, Ь, с и d удовлетворяют соотношениям c<zd (1), a+b=c4-d (2), a+d<6+c. (3) Из (2) и (3) при сложении получаем не- равенство 2а4-&+^<Ь+2с+«Л откуда 2а<2с и, следовательно, а<с. Из (1) и неравенства а<с заключаем, что a<c<d. Наконец, из (2) и неравенства а<с полу- 150
чаем d<zb. Таким образом, выполняется це- почка неравенства а <£<</<&. Следовательно, самый большой (по весу) улов у рыбака В; за ним следуют D, С и А. 8. Ромб является трапецией и параллело- граммом, но не является квадратом. Следова- тельно, четырехугольник, начерченный учите- лем на доске, имеет форму ромба. 9. Задача допускает 13 решений, например: 10. 11. Пусть а — число бокалов, b — чашек, с — кувшинов и d — бутылок. Тогда предметы, находящиеся в равновесии на чашах весов на рисунке, позволяют записать следующие урав- нения: . 2а+Ь=2с, 5b=;C+2d, 5a=3c-f-2rf, 151
из которых следует, что 3c=6d, 3c=5&4-2d, 3c=2a+b+2d, 3c=a+3b+2d. Итак, 3 кувшина можно уравновесить ли- бо 6 бутылками, либо 5 чашками и 2 бутыл- ками, либо 2 бокалами, 1 чашкой и 2 бутыл- ками, либо 1 бокалом, 3 чашками и 2 бутыл- ками. 12. 72—42— 62 = 7—4—6, 92—62—72 = 9—6—7. Существует ровно 4 таких трехзначных числа: 976, 967, 764, 746. 13. Задача легко решается, если сначала построить треугольник ACD (I), площадь ко- торого составляет. ’А от площади треугольни- ка АВС: для этого достаточно выбрать точку D так, чтобы CD= (*А) СВ. Продолжая дей- ствовать в том же ду- с хе, построим треуголь- никЛОЕ (II), площадь /// которого составляет '/< //\\ //'ы v\ оставшейся части ис- А—jf“--------q---- ходного треугольни- ка — треугольника A.BD-. для этого достаточ- но выбрать точку Е так, чтобы ДЕ=(*/4) АВ* Затем достаточно выбрать точку F так, чтобы DF—(4$)DB и, наконец, точку G так, чтобы EG=(V2)EB. 14. Обозначим три высказывания первыми буквами имен их авторов и индексами 1, 2, 3, соответствующими номерам, под которыми 152
высказывания приведены в задаче: Аь А2, А3, Б.ь Б2, Бз, Вь В2, В3, Мь М2, Мз. Начнем с анализа высказываний Вольфганга. Высказы- вания Bi и Вз утверждают одно и то же, по- этому они либо оба истинны, либо оба ложны. Но по условию задачи они не могут быть оба ложными. Следовательно, они оба истинны и ложно высказывание В2. Так как В3 истинно, высказывание Б3 ложно, а это означает, что Б! и Б2 истинны. Так как Б2 истинно, выска- зывание Мз ложно. Следовательно, высказы- вания Mi и М2 истинны. Но если М2 истинно, то А| ложно, А2 и Аз истинны. Итак, окно разбила Ангелика. АНТИЧНЫЕ ЭТЮДЫ 1. а) Так как—- —означает 1, можно предположить, что сплошная верхняя линия соответствует 1, а линии, состоящие из двух разделенных промежутком отрезков, всегда соответствуют 0. Поскольку знаком .......... Фу Ши обозначил число 3, средняя сплошная линия должна соотвеугтвовать числу 2 (1+2+ +0=3). Знаком ---------— Фу Ши обозначил число 6, поэтому нижняя сплошная черта должна соответствовать числу 4 (0+2+4 = 6). Таким образом, знак zzz .zll1 может означать только число 4 (0+0+4 = 4). б) С помощью трех линий, сплошных или состоящих из двух отрезков, разделенных промежутком, можно составить знаки, соот- ветствующие в системе обозначений Фу Ши числам 0, 1, 2, 3, 4, 5, 6, 7О 153
2. Пусть ширина составляет х ладоней, длина — у ладоней. Тогда 7+У=7, (1) х+!/=Ю, (2) х=10 — у. (2') Подставляя (2') в (1), получаем +//==7, 4 y=f>. Затем из (1) находим —т~ +6=7, х = 4. 4 3. Пусть х — число пчел в рое. Тогда х= -^- + ^-«+2. (1) Z У /X 2 через у, преобразуем уравне- ние (1) (так как у2—х!2, или х=2у2) к виду У+ ~у2+2=-2у\ (2) 2у2 — 9у — 18=0, откуда „ 3 t/! = 6, 1j2=- -2~- Этим значениям у соответствуют следующие значения х: х{ = 72, Х2=4,5. Так как число пчел в рое . может быть только натуральным числом, то ____ т/ 21 + -72+2= 72. Г 2 9 Итак, в рое было 72 пчелы. 154
4. По теореме Пифагора Х2 + 52==(Х_|_|)2< х2-|-25—x’+2x-j-l, х=12. Глубина пруда—12 шагов. 5. Пусть х — число учеников Пифагора. Тогда 2_х+ -1_х+3=х, откуда х=28. Итак, у Пифагора было 28 учеников. 6. 7. Пусть мул нес х, а осел у мер зерна. Тогда х+1=2 (у- 1), (1) х—1=у4-1. (2) Подставляя х—у+2 в (1), получаем 155
jH-2+1 = 2 (0- 1), 0=5. Следовательно, x=7, т. e. мул нес 7 мер зер- на, а осел 5 мер. 8. а) Пусть х — число голов скота во всем стаде. Тогда откуда после эквивалентных преобразований 2 — х=70, 2х—630 находим: х==315. Итак, во всем стаде было 315 голов скота. /2 1 2X1 б) I х + —х-|- —х-|- —— х • — =10, \ 3 3 9 ) 3 / 9 6 3 2 X1 х= 10JL -=13,5. 20 9. Площадь луночек нетрудно вычислить следующим образом: AfI+Af2= - -2- о о Z О = (а2+62—с2)+ -у>- Но так как а24-&2=с2, л1,+ль= -|L что и требовалось доказать. 156
10. Пусть 1 сноп хорошего урожая дает х доу зерна, среднего — у доу и плохого — z доу .зерна. Тогда 3x-|-2i/4- z==36, 2x-|-3y-|- z=34, x-j-2y+3z=26, откуда х=9*/4, У=4’Д, z=23/t. Итак, 1 сноп хорошего урожая дает 9‘Д доу зерна, 1 сноп среднего урожая — 4*/< доу и 1 сноп, плохого урожая — 23/< доу. 11. Пусть S — площадь двух «сапожных ножей», Г| и Г2— радиусы двух внутренних ок- ружностей. Тогда S—пг2—пг21—ЛГ22, (1) 2г=2г,4-2г2. (2) К, этим двум соотношениям добавим третье (высота, опущенная из вершины прямого уг- ла на гипотенузу, есть среднее пропорциональ- ное отрезков, на которые ее основание делит гипотенузу). Соотйошения (2) и (3) преобра- зуем к виду (Г14-Г2)2 = г2, (2') 157
2rir,== -7- ' <3'> О Вычитая, получаем z2 п24-г22=г2— — О и, подставляя в (1), находим 12. Если все 4 источника заполняют бас- сейн за х дней, то 12х 6х , 4х , Зх _ 12 12 12 12 12 12 ’ 12x+6x+4x-J-3x= 12, 25х=12, Следовательно, чтобы заполнить бассейны из четырех источников, требуется 12/е5, т. е. чуть меньше половины дня. 13. Пусть доля сына составляет х, дочери у, доля матери г динариев. Тогда х+у+г=3500, х—2z, у— — . следовательно, х=2000, у=500, z=1000. Таким образом, вдова должна получить 1000 динариев, сын — 2000 динариев и дочь— 500 динариев. 158
14. Пусть х—число, которое требуется найти. Тогда 200x=t/2, (1) 5х=у. (2) Подставляя (2) в (1), получаем 200х=25х2, 200 = 25х, х =8. Итак, третье число равно 8. Проверка: 5*8= =40; 200-8=1600 и 1600 =40. 15. Пусть х — число голубей, севших на де- рево, а у — число голубей, расположившихся под деревом. Тогда и, кроме того, х—1=у4-1, т. е. x=z/4-2. Подставляя x=t/+2 в первое уравнение, получаем (у—О •3=</+2-|-1/, Зу—3-2у-\-2, У—5- Следовательно, х=у+2—7. Итак, 7 голубей сели на дерево, а 5 голу- бей расположились под деревом. 16. Если х — число яблок, собранных жен- щиной в саду, то первому стражнику доста- лось х/2 яблок, второй получил х/4 яблок, третий — х/8 яблок и четвертый — х/16 яблок. Так как х/16=10, то х= 160. Следовательно, женщина собрала в саду 160 яблок. 159
ШКОЛЬНЫЕ ИСТОРИИ 1. Наименьшее отличное от нуля натураль- ное число, делящееся на 3, 4, 6 и 8, равно 24. Следующее такое число (равное 48) больше 30. Таким образом, в классе всего 24 учени- ка. Они-то и писали контрольную по матема- тике. Так как (*/з) -24=8, (»/4) -24=6, ('/6) -24= =4, (*/8)-24=3 и 84-64-4+3=21, контроль- ную написали с ошибками 21 человек. Следо- вательно, 3 человека решили все задачи пра- вильно. 2. Если утверждение Вольфганга истинно, то утверждение Карин ложно. Следовательно, число отлично от 9. Это означает, что если утверждение Петера истинно, то простое чис- ло должно быть двойкой, и утверждение Рос- виты ложно. Все другие варианты приводят к противоречию. Итак, лучший математик из 5-го «А» должен был назвать число 2. 3. Пусть х — число деталей по 10 марок за штуку, у — число деталей по 3 марки за шту- ку и г — число деталей по 50 пфеннигов за штуку. Тогда 10x+3y+0,5z=29, (1) х+у+г=29, (2) Кх<29, (3) Ку <29, (4) Kz<29 (5) и, следовательно, 10x+3y+0,5z=x+y+z, 9x-j-2y=0,5z, или, что то же, 18х+4у=z. 160
Из (1) следует, что х— 1. Но тогда 18х+4у= = г, откуда #=1, при этом г=22 или У~2 (г=26). Условию (2) удовлетворяет только набор значений х=1, у—2, г=26. Для школьной мастерской купили 1 де* таль по 10 марок за штуку, 2 детали по 3 мар- ки за штуку и 26 деталей по 50 пфеннигов за штуку. 4. Кроме нуля, существует 9 однозначных чисел. Следовательно, число цифр, необходи- мых для нумерации страниц однозначными номерами, равно 9. Двузначных чисел всего 90. Следовательно, число цифр, необходимых для нумерации страниц с двузначными номе- рами, равно 90-2=180. Трехзначных чисел 900. Чтобы записать их, потребуется 900-3 цифр. Четырехзначных чисел всего 9000. Для их записи потребуется 9000-4 цифр. Пусть х— число страниц учебника с четырехзначны- ми номерами. Тогда 9+90 • 2+900 • 3+х - 4= 6869, х=995. Число страниц равно числу всех страниц с од- но-, двух-, трех- и четырехзначными номера- ми, т. е. 9+90+900+995= 1994. Итак, в учебнике 1994 страницы. 5. Требуется найти все пары натуральных чисел (a, b) с aZ>b, для которых а2—62=я+&. По формуле разложения разности квадра- тов а2—Ь2— (а-~-Ь) (а—Ь), поэтому требуемое равенство эквивалентно равенству (а+&)Х X {а—Ь} — (а+&). Так как аиЬ — натураль- ные числа и а>Ь, то а+Ь^0. Разделив на а+& обе части последнего равенства, получим, 11 Ф-73 161
что требуемое свойство чисел а и b эквива- лентно равенству а—b= I, или а—1 = Ь. Итак, требуемым свойством обладают все пары (а, &) натуральных чисел, в которых а на едини- цу больше b (Ь и а — последовательные нату- ральные числа). 6. За 3 шага Рози продвигается вперед на 50 см. Следовательно, сделав 2-3-29=174 ша- га, она окажется на расстоянии 29 м от стар- та. Сделав еще 2 шага, Рози достигнет вто- рого флагштока и закончит упражнение. Итак, двигаясь от одного флагштока до другого, Рози успевает сделать 176 шагов. 7- «>4- 2 _ 3 1 6 а) Деление Ь) -п- J5_= 17 b) Умножение 14 21 42 4+ 1 _ 4 3 4 с) Вычитание d| т- JL = 4 3 8 4) Сложение 8. Пусть х — число задач, решенных сыном правильно (за них сын получил от отца 10х пфеннигов), а у — число неправильно решен- ных задач (за них сыну пришлось вернуть от- цу 5у пфеннигов). Тогда 10х—5z/=80, х+ у=20, и х=12, z/=8. Сын правильно решил 12 задач и допус- тил ошибки при решении 8 задач. 9. Определение, .скрытое в клетках табли- цы, гласит: выпуклый четырехугольник, две противоположные стороны которого парал- лельны, называется трапецией. 162
10. На седьмой день пути оба подмастерья проходят одинаковое расстояние — по 7 миль каждый. В последующие дни второй подмас- терье должен наверстать то расстояние, на ко- торое он отстал от первого подмастерья за первые 6 дней пути. Возникает уравнение: (7-6) + (7-5) 4- (7-4) + (7-3) + (7—2) + + (7-1) 4- (7-f-1) + (7+2) + (7+3) (7+4) + (7+5) + (7+6)=7х, 91=7х, х=13. Итак, подмастерья встретятся к исходу 13-го дня. 11. Обозначим число цветных карандашей у каждой из девочек начальной буквой ее имени. Из условий задачи следует, что У=2Р, С=Р—13, У+Р+С<50 и У+Р+С —про- стое число. Так как 11 = 2+9=3+8=4+7=5+6, простыми числами, меньшими 50. с суммой цифр, равной 11, могут быть только 29 и 47. Подставляя каждое из этих чисел, получаем: 2Р+Р+Р—13=29, 4Р=42, и 2Р+Р+Р—13=47, 4Р=60. У трей девочек всего было 47 цветных ка- рандашей, причем Р=15, У=30, С = 2, т. е. у Уты было 30 карандашей, у Регины 15 и у Сабины 2 карандаша. 12. «Тот, кто знает счет», без труда соста- вит уравнение _|_+150= -|-х+50, х=160. Число, которое требуется найти, равно 160. 11* 163
13. 1. Учитель химии живет в одном доме с учителем математики =^*Х=/=М. 2. А младше В и С=>А<В, С. 3. Учитель математики играет в шахматы с С^МУ=С. 4. В старше учителя физики, который, в свою очередь, старше учителя биологии ->В>Ф>Б. 5. Старший из учителей живет дальше всех от школы ^старший =/=Х. Из 3: учитель математики не С. Из 2 и 4: учитель физики не А. Из 4 и 5: В не преподает химию и мате- матику, так как он старше двух других учи- телей по возрасту ^А преподает математи- ку =>/1 не учитель химии. Из 2 и 4: А младше своих коллег =>А пре- подает биологию. Из 2 и 4: А<С<В=>С преподает физику. Так как В старше своих коллег и не пре- подает химию и математику, В учитель немец- кого языка и истории =>С учитель химии. М Ф X Б Н И д х - ~ X - - В - - • - - х X С - X X - - - 14. Пусть х — число скамей и у — число спортсменов в гимнастическом зале. Тогда 6(х-1)+3=у, (I) 5x4-4=у, (2) * => означает «следовательно». 164
откуда 6х—6+3==5х4~4 и, следовательно, х=7. Подставляя в (2), на- ходим £/==39. Итак, в гимнастическом зале занимаются 39 спортсменов и расставлено 7 скамей. 15; Условия задачи позволяют составить уравнение (х — число победителей олимпиа- ды): х=64-10—1 = 15. Призовые места в математической олим- пиаде заняли 15 человек. Задача допускает обобщение: если бы школьник, набравший полное число очков, стоял на т-м месте слева и на n-м месте справа, то число победителей олимпиады бы- ло бы равно 1. 16. Пусть V—объем 2 г серебра. Тогда Пусть х — длина изготовленной из 2 г серебра проволоки (в см). Диаметр проволоки состав- ляет 0,002 мм, ее радиус — вдвое меньше, т. е. равен 0,001 мм = 0,0001 см, и 0,00012лх=0,1905, откуда «о,обоб1пя=-бобоооо 3,14 Итак, из 2 г серебра получается около 6 060 000 см = 60 600 м = 60,6 км круглой про- волоки диаметром 0,002 мм. <65
СТАРОЕ И НОВОЕ ИЗ ПРАКТИКИ 1. Предположим, что на отливку царской короны пошло х мин золота, у мин меди, г мин свинца и f мин железа. Тогда х+у+г+/=60, (1) х+у= .60=40, (2) О x+z= — -60=45, (3) 4 х-Н= —-60=36. (4) 5 Складывая (2), (3) и (4), получаем 3x+zH-z+f=121, (5) а вычитая из (5) уравнение (1), находим, что 2х=61, т. е. х=30,5. Подставляя найденное значение х в (2), (3) и (4), вычисляем зна- чения у, z и f: (/=40—30,5=9,5, 2—45—30,5=14,5, f== 36—30,5= 5,5. Итак, на отливку царской короны израсходо- вано 30,5 мин золота, 9,5 мин меди, 14,5 мин свинца и 5,5 мин железа. 2. Воспользуемся для вычислений горизон- тальной системой координат. Радиус Земли пренебрежимо мал по сравнению с расстоя- нием от Земли до Солнца, и солнечные лучи вполне допустимо считать параллельными. Итак, наблюдатель находится в центре Зем- ли М и видит, что отклонение а=7,5° направ- ления на Солнце от отвесной линии в Алек- сандрии равно центральному углу р между 166
Александрией и Сиенной (как соответствен- ные углы). а) Окружность Земли и вычислим из про- порции р : 360=5000 : и. _ 360-5000 _ 360-5000 ~о40000 ₽ 7,5 Солнечные лучи М — Центр Земли А — Александрия В - Сиенна а-р Дуга АВ = 5000 егил .стадий Итак, окружность Зем- ли составляет 240 000 египетских стадий. б) 240 000 0,18472«а «44 333. Окружность Земли составляет 44 333 км. в) Полученная Эра- тосфеном величина отличается от принятого ныне значения на 4333 км. 3. Пусть х — число дней, отработанных работником. Тогда 48х—12(30—х) =0, откуда х=6. Итак, из 30 дней отработанными оказа- лись лишь 6. 4. Фабрика выпустила: за январь х=х-|-0-10, за февраль х-|-10—х-{-1 • 10, за март х-(-20=х4-2-10, за декабрь х+110=х-4-1Ы0. Итого за год 1920= 12х-Н04-20+...+110, х=105, за июнь x-f-5> 10= 155, за декабрь x-f-11 • 10=215. В июне фабрика выпустила 155, а в де- кабре 215 столов. 167
5. ЗГ, ЗБ. ЗК, 34, ЗЖ, ЗС, ЗФ, ГБ, ГК, гч, ГЖ, ГС, ГФ, БК, БЧ, БЖ, БС, БФ, КЧ, КЖ, КС, КФ, чж, ЧС, ЧФ, же, ЖФ, СФ. Число различных маркировок двухжиль- ных шнуров с жилами в изоляции различного цвета равно 7+6+5-1-4+3-1-24-1 = 28. Электрик может скрутить шнуры 28 различ- ных маркировок. 6. Пусть х — число ламп мощностью 40 Вт, у— число ламп мощностью 60 Вт и z— число ламп мощностью 75 Вт в комплекте. Тогда 40x+60i/+75z= 1800, (1) х+//+г=32. (2) Умножая второе уравнение на 40 и вычи- тая из (1), получаем 20//+35г=520, откуда </ = 26- £. Это уравнение допускает целые положитель- ные решения у только при z=0, 4, 8, 12. Следовательно, заявки на комплекты ламп можно удовлетворить следующими тремя способами: (75 Вт) г 0 4 8 12 (60 Вт) у 26 19 12 5 (40 Вт) х 6 9 12 15 7. Укладка деталей в футляр производи- лась в следующей последовательности: 2—7— —5—6—1--3-4. 168
8. Удвоение диаметра (при неизменной длине) приводит к учетверению веса: после удвоения диаметра балка весила бы 1200 Н. При уменьшении длины балки вдвое вес ее уменьшился бы вдвое, т. е., став вдвое толще и вдвое короче, балка весила бы 600 Н. Итак, толстая короткая балка круглого сечения ве- сит вдвое больше более длинной и тонкой. 9. Пусть х — число посетителей, которое необходимо для заполнения цистерны при условии, что каждый, кто открывает калитку в сад перед домом Эдисона, накачивает в цистерну 20 л воды. Тогда 20х=25 (х—12) и х—60. Итак, при «норме» 20 л цистерну запол- няют 60 посетителей. Следовательно, цистер- на вмешает 1200 л. 10. Изображенным на рисунке валиком маляр «накатал» 6 рисунок. 11. Пусть х — грузоподъемность (в т) меньшей машины, у — грузоподъемность боль- шей машины. Тогда 31x+27i/= 143, (1) 1,5х=у, (2) откуда х=2, у—3. Итак, машина с меньшей грузоподъем- ностью перевозит за 1 рейс 2 т сыра, а ма- шина с большей грузоподъемностью — 3 т сыра. 12. По проекции на две взаимно перпен- дикулярные плоскости (вид спереди и вид сзади) тело, вообще говоря, можно восста- новить неоднозначно, т. е. весьма многими способами. В частности, это относится к на- 169
шей задаче. Приведем лишь три тела, имею- щие те виды спереди и сверху, которые при- ведены в условиях задачи. Прямоугольный параллелепипед 13. Как видно из схемы, путешественнику достаточно нанять 2 носильщиков (П — путе- шественник, Н| — первый носильщик, Н2 — второй носильщик, V* — запас пищи и воды на k дней для 1 человека, k—О, I, 2, 3, 4). п Н1 н2 1 -й день 2-й день 14. Где бы ни начинался маршрут, обойти все 17 мостов, побывав на каждом из них не более 1 раза, невозможно. На приводимом ниже маршруте по крайней мере 2 моста придется пройти дважды. 170
15. Пусть а первоначальный объем дре- весины. По истечении года с благоприятными условиями объем древесины составит , 5-а 105а _ 21а 100 100 20 По истечении следующего года с неблаго- приятными условиями объем древесины со- ставив 21а , 3 20 100 21а _ 2163а 20 2000 1,0815а. Итак, за 2 года объем древесины возрас- тет на 8,15%. 16. Цена Р складывается из платы, взи- маемой за пользование архивным докумен- том в размере х марок, и стоимости изготов- ления копий в размере $ марок за штуку. Если заказчику требуется п копий, то их из- готовление обойдется ему в P=x-f-ns марок. По условиям задачи при п—З заказчик 171
уплачивает P—6—x-}-3s марок, при n=b стоимость заказа составляет P==9=x4-5s марок. Следовательно, 6—3s—9—5s, откуда 2s=3, или s—1,5. Но тогда х=6— -3-1,5= 1,5. Цена изготовления 9 копий, приведенная в условиях задачи, служит для проверки и подтверждает правильность полученного ре- шения. Итак, изготовление п копий одного архивного документа обойдется заказчику в Р— 1,5-f-n-1,5 марок. 17. Пусть х — число страниц в книге, на- бранной более крупным кеглем. Тогда х-32-51 = 152-45-68, Таким образом, при наборе более круп- ным кеглем объем книги составит 285 стра- ниц. 18. 1 кг нектара содержит 700 г воды и 300 г твердого вещества, 1 кг меда содержит 170 г воды и 830т твердого вещества. Состав- ляем пропорцию: 300:830=1 : х, откуда х « 2,77. Итак, для получения 1 кг меда пчелам необходимо собрать 2,77 кг нектара. 19. Докажем, что все шарики умещаются в кубе с длиной ребра 10 см. Действительно, один слой содержит 100X100=10 000 шари- ков. Так как размеры куба позволяют по- местить в нем 100 слоев, куб с длиной ребра 10 см действительно вмещает 10 000X100= 172
= 1000 000 шариков. Масса шариков меньше массы куба, изготовленного из того же мате- риала, так как шарики не заполняют про- странство куба целиком. Масса стального куба с длиной ребра 10 см составляет около 7,8 кг, масса шариков меньше и составляет около 4 кг. Такую массу без труда может поднять один человек. МИНУТУ НА РАЗМЫШЛЕНИЕ 1. Парис мог рассуждать следующим образом. а) Предположим, что Афина изрекла исти- ну. Тогда Афина — прекраснейшая из богинь, и, по предположению, утверждение (4) лож- но. Мы приходим к противоречию, так как Гера не может быть прекраснейшей из бо- гинь, ксль скоро прекраснейшая из богинь Афина. Итак, исходное • предположение ложно. б) Предположим, что истину изрекла Ге- ра. Тогда Гера —прекраснейшая из богинь и, по предположению, утверждение (2) ложно. Мы снова приходим к противоречию, так как Афродита не может быть прекраснейшей из богинь, кодь скоро прекраснейшая из богинь Гера. Следовательно, и зто исходное предпо- ложение ложно. в) Предположим, что Афродита изрекла истину. Тогда Афродита — прекраснейшая из богинь. Отрицания утверждений (2), (3) и (5) истинны и подтверждают, что Афродита прекраснейшая из богинь. Итак, по решению, вынесенному «судом Париса», прекраснейшая из богинь Афродита. 173
2. Число, стоящее на месте каждой бук- вы, показывает, сколькими способами можно добраться до нес. спускаясь с вершины кон- фигурации 1 2 3 2 1 1 1 3 6 7 6 3 1 1 1 2907 3139 2907 1 2 1 8957 1 3 3 1 1 4 6 4 1 1 5 10 10 5 1 1 6 15 20 15 6 1 1 7 21 35 35 21 7 1 1 8 28 56 70 56 28 8 1 g 36 84 126 126 84 36 9 29-512 Итак, слово «абракадабра» можно прочи- тать 8957, а слово «математика» 512 спосо- бами. 3. 180 мин отделяют отправление поезда (6 ч) от 3 ч. Число минут, оставшихся до от- правления поезда, мы найдем, разложив раз- ность 180—50=130 в сумму двух чисел, одно из которых в 4 раза больше другого, т. е., взяв 'А от 130. Следовательно, до отправле- ния поезда оставалось 26 мин. Действитель- но, за 50 мин перед разговором до 6 ч оста- валось 50+26=76 мин, т. е. с 3 ч прошло 180—76=104 мин, что вчетверо больше 26 мин, остававшихся в момент диалога до отправления поезда. Следовательно, диалог происходил в 5 ч 34 мин. 174
4. Стол должен быть накрыт по крайней мере на 7 персон: для одного дедушки и од- ной бабушки, образующих супружескую чету, их сына, его жены и их троих детей (внуков родителей сына): одного сына и двух доче- рей. 5. Локомотив должен маневрировать сле- дующим образом: двигаясь вперед, миновать стрелку 6, после чего, двигаясь назад, про- ехать стрелки 6, 8, 5, 3, 4 и 7, а затем, дви- гаясь снова вперед, миновать стрелки 7 и I. 6. Девушки сидят в следующем порядке: К-Д-А-Ж-Б. Следовательно, лучших подруг Жанетты зовут Анеттой и Бабеттой. 7. х(х-Н)—х=х2. Чтобы найти задуманное число, достаточно 16 512 4 8 32 128 266 2 64 3 36 2 4 6 9 18 1 12 9. а) Крест из 4 черточек от картинки к картинке в верхнем ряду (слева направо) поворачивается по часовой стрелке на 45°. Левая точка (на первой картинке) остается при этом на своем месте, а правая при каж- 175
дом повороте перемещается на одну черточку по часовой стрелке и на четвертой картинке совпадает с другой точкой. Итак, пустой квад- рат следует заполнить картинкой А из ниж- него ряда. б) Пустой квадрат следует заполнить кар- тинкой А, так как при движении слева напра- во малый круг остается на месте, а средний каждый раз поворачивается на 90° по часо- вой стрелке. в) Темные треугольники в правом верх- нем и в левом нижнем углах остаются непод- вижными. Треугольник в левом верхнем углу при движении слева направо поворачивается каждый раз на 90°, а треугольник в правом нижнем углу поворачивается каждый'раз на 180°. Следовательно, в пустой квадрат сле- дует поместить картинку В. . г) Переход от картинки к картинке в верх- нем ряду можно осуществить, поворачивая куб каждый раз на 90° вокруг оси, проходя- щей через центры его правой и левой граней. Следовательно, пустое место на правом конце верхнего ряда следует заполнить картин- кой В. 10. Номер автомашины учителя математи- ки выражается четырехзначным числом вида abba, причем 2(а+&) = 10а4-Ь, Ь=8а. Следовательно, а может быть только циф- рой 1, а b — только цифрой 8. Итак, автомашина учителя математики имеет номер 1881. 11. Пусть а, Ь, с—3 различные цифры, каждая из которых больше нуля. Перестав- 176
ляя их всеми возможными способами, мы по- лучим следующие трехзначные числа: 100<z-H0&4-c, 100с4-10а4-&, 100&-Ы Oc-f-ct, 100&+Юа4-с, ЮОа-МОс-Н, 100с4-Ю&+а. Сумма всех 6 чисел представима в виде: з= 100(2а+2&+2с)4-10(2а+26+2с)4- + (2а4-26-}-2с) = 111 (2а+26+2с) = = 222(a+f»+c) = 222Q, - где Q — сумма цифр любого из 6 трехзнач- пых чисел, Моника умножила сумму цифр названно- го Марией-Луизой трехзначного числа на 222. 12. Разместить зверей по их клеткам слу- житель зоопарка может за 14 переселений, производимых по следующей схеме (см. с. 178, номера клеток идут слева направо). 13. Пусть Р — рыбка, Ш — шарик, К — колокольчик, Кр — коромысло. Тогда Ш=2Кр, (1) Р+Ш=К, (2) 2Р+К=Р+Ш-ЬК+Кр, (3) или Р=Ш4-Кр. (4) Из (1) и (4) следует, что Р=3 Кр, (5) а из (1), (2) и (5)—что К- 5 Кр. (6) Если х — груз, удерживающий систему в равновесии вместо вопросительного знака, то х=3 Кр+Ш=5 Кр- Как видно из (6) и (2), равновесие систе- мы не нарушится, если вопросительный знак заменить двумя предметами: одним шариком и одной рыбкой. 14. В отдельных бросаниях выпало 2, 2, 3, 4 и 5 очков (24-2+4=8, 8—5—3=0). 12 ф-га 177
№ п/п 1-я клетка 2-я клетка 3-я клетка I Пантера Крокодил Осел 2 Пантера Крокодил —* 3 Пантера Крокодил Лев 4 — Крокодил Лев 5 Крокодил — Лев 6 Крокодил Лев —- 7 Крокодил Лев Осел 8 Крокодил Лев Осел 9 Крокодил Лев Осел 10 Лев Осел 11 Лев — Осел 12 Лев Осел .— 13 Лев Осел Волк 14 Лев Осел Волк
4-я клетка 5-я клетка Общий вольер Волк Лев Осел Волк Лев Осел Волк • Осел Волк Пантера Осел Волк Пантера Осел Волк Пантера — Волк Пантера Волк Пантера Пантера Волк — •— Волк Пантера Крокодил Волк Пантера Крокодил Волк Пантера Крокодил — Пантера Крокодил Крокодил Пантера —
15. Названия профессий, требующих осно- вательной математической подготовки: (1) радиоинженер; (2) мостостроитель; (3) мо- реплаватель; (4) программист. 16. Пусть х — число купленных лошадей, у — число купленных быков. Тогда 31x4-21#= 1770, откуда 210= 1770—31х= 17644-6—21х-10х Из последнего равенства следует, что число 10х—6, а следсвательно, и 5х—3 должно де- литься на 21. Полагая 5х—3=21z, получаем 0=84—х—2z, 21£+3=4z-F-£+1 . Так как х — целое число, числитель z-f-З дол- жен делиться на 5, т. е. z=5u—3 и х=4(5«—3)4-«=21ы—12, 0=84—21«4-12—10и4-6= 102—31ы. Но 0 — положительное число, а поскольку z=5u—3 не может быть равно нулю, то воз- можны только 3 случая: ы=1, и=2 и ы=3. Итак, задача Эйлера допускает следую- щие 3 решения: 1) и=1; х=9, 0=71; 2) и—2; х=30, 0=40; 3) н=3; Х=51, 0=9. Нетрудно проверить, что во всех трех слу- чаях 31x4-210=1770. 12* 179
АРИФМЕТИКА С ПТИЧЬЕГО ПОЛЕТА 1. Пусть х—первое число. Тогда второе число равно 19—х. По условиям задачи х24-(19—х)2==205, т. е. х2— 19х-{-78=0. Это квадратное уравнение имеет корни Xi = 13 и х2=6. Итак, в задаче Штифеля речь идет о чис- лах 13 и 6. 2. а) Если х>8, то х-}-3>11, и поэтому заведомо выполняется неравенство х4-3>10. б) Если 60х=50у, то x=5k и у—4k, где k—натуральное число. Следовательно, х<у. в) Если 5х>10, то х>2, а поскольку У>х, то </>3. г) Так как х>у, должно выполняться ра- венство x—y+k, где k— некоторое отличное от нуля целое положительное число. Но y-r2<.y+k+5, поэтому »/4-2<х+5. д) Гак как х>у, то x=y+k, где k— не- которое отличное от нуля целое положитель- ное число. Поскольку 60<75 и y<.y+k, спра- ведливо неравенство 60— (y+k)<75-y или 60—х<75—у. е) Если у<5, то 3«/<15 и тем более 3</< 17. 3. Пусть а и b — два числа, удовлетворяю- щих условиям задачи. Не ограничивая общ- ности, будем считать, что а>Ь. Тогда | аЬ = &+4, (!) 2J±=a-S. (2) (КО
Из (2) следует, что а-^-Ь=2а—12, пли Ь=а—12. Подставляя это соотношение в (1), получаем }а (а—12) —а—12+4, а2— 12а=(а—8)2, а2— 12а—а2— 16а 4- 64, а—16, откуда 6=4. Как показывает проверка, числа 16 и 4 действительно удовлетворяют условиям задачи. 4. Выпишем 10 наименьших натуральных чисел, дающих при делении на 6 остаток 4: 4, 10, 16, 22, 28, 34, 40, 46, 51, 58. Лишь 2 из них, а именно числа 28 и 58 при делении на 5 дают остаток 3. Из 2 чисел 28 и 58 только число 58 при делении на 4 дает остаток 2. Это же число при делении на 3 дает остаток 1. Итак, число 58 обладает всеми требуемы- ми свойствами. Одновременно доказано, что 58 — наименьшее из чисел, удовлетворяющих условиям задачи. Все остальные числа, обла- дающие требуемыми свойствами, представи- мы в виде 58+606, где k — произвольное на- туральное чисдо. 5. Пусть п — первое из двух чисел. Тогда второе число равно 90—п и 25п , 75(90—п) _,л -Too" +“iob-------3 • откуда п 3(90—n) 4 4 181
«+3(90—«) = 120, «+270—3«= 120, 2«=150, «=75. Итак, условиям задачи удовлетворяют числа 75 и 15. 6. Неравенство а(с—b)/(b—а)>0 выпол- няется в том и только том случае, если либо с>Ь и Ь>а, либо c<zb и b<Za, т. е. если либо а<.Ь<с, либо а>Ь>с. При а<_Ь<.с, т. е. при а=13, 6= 15, с=20 выражение а(с—b)/(b—а) принимает нецелое значение: 13(20—15) _ 13-5 15-3 2 При а>Ь>с, т. е. при а=20, 6=15, с=13, получаем 20(13—5) _ 20(—2) _8 15—20 —5 Итак, приведенное в условиях задачи выра- жение принимает целое положительное зна- чение при а—20, 6=15, с=13. 7. Подставляя а=с-^-7, получаем (с+7) (с+9)+с(с-2)-(2с+14)с= =С2+ 16с+63+с2—2с—2с2— 14с=63. 8. а) ЛиВ={4, 5, 6, 7, 8}, б) ЛПВ={4, 6}, в) 5={3, 7, 9}, г) (7=0, д) Д\В={7}. 182
Магический квадрат Пусть Т — сумма чисел, стоящих в каж- дой строке, в каждом столбце и на каждой диагонали. Тогда T—x-\-y-\-2z——2x+2y+5z= =x+y+4z—\G—2z-\-“iy—\b=lz—x. Мы получаем только 5 из 8. возможных уравнений и 3 тождества. Из четырех пер- вых уравнений находим х=8, t/=9, z=5, Г=27. Полученные значения удовлетворяют пятому уравнению. Следовательно, в клетках магического квадрата стоят числа 8 7 12 13 9 5 6 11 10 Сумма четных чисел в 4 угловых клетках 84-6+12-|-10=36=4-9 в 4 раза больше числа, стоящего в центральной клетке. Кросснамбер 9. a) af(6, 7, 8, .... 58, 59}, б) х?{0, 1, 2... п, п+1, ...}, в) уе{0, 1, 2, .... п, п+1, ...}, 183
г) z?(5}, д) &<?{0, 1, 2}. . 10. 10fl+b=3(a4-&), где 1Са<9, 0^6<:9, откуда 7 a—2b, a=— . 7 Итак, а принимает целое значение лишь при 6=7 (а=2). Условиям задачи удовлетворяет единствен- ное двузначное число: 27. 11. а) Пусть а=1. Тогда равенство 2а-{-6Ь—аа^-Ьа переходит в равенство 2+66=1+6, откуда ь=— —. 5 Поскольку 6 не является ни одним из чисел 0, 1, 2, 9, то случай д=,1 необходимо исключить. б) Пусть а=2. Тогда приведенное в усло- виях задачи равенство переходит в такое: 4+66=4+62 или 6(6—6) =0. Так как 6>0, то из этого квадратного урав- нения следует, что 6=6. Итак, одно решение задачи найдено: а—2, 6=6. в) Пусть а>3. Преобразуем исходное ра- венство 2а+66=а°+6а к виду а(а°~’—2) = 6(6—6е-1). Так как а^З, выполняется неравенство а(а°-1—2)>3(32—2) =21. С другой стороны, 6(6—6°-') = 5<21 при 6=1, 6(6—6°-')=4<21 при 6=2, 184
6(6—6“~')=0<21 при 6=3. Следовательно, при а>=3 и 6=1, 2, 3, ... ра- венство 2а 4-66=а“ 4-6“ не выполняется. Итак, задача допускает единственное ре- шение, которое дано в случае «б». 12. Число, предшествующее натуральному числу п, равно п—1. Число, следующее за п, равно n-4-l. По условиям задачи (п—1)Х X («4-1) =2208, или п2=2209. Следовательно, задача допускает един- ственное решение в натуральных числах: п=47. Проверка: 46-48=2208. 13. Умножив обе части уравнения на ху, получим после несложных преобразований: </4-х4-1=х//, ху—х—у-\-\, х(у—1)=//4-1, х= У+1 у— 1 Итак, пары натуральных чисел (х, у), удов- летворяющие уравнению, получаются только при у\—2 и у2—3, что соответствует Xj=3 и х2=2. 14. Сумма чисел от 1 до 9 равна 45. Она разделена на 3 равные части. Следовательно, каждая из 3 сумм, приведенных в условиях задачи, равна 15. Итак, задача допускает следующие реше- ния: 185
1+5+9=24-6+7=34-4+8, 1+6+8=2+4'+9=3+5+7. 15. Число 0 меньше каждого из 20 осталь- ных допустимых значений 1, 2, 20 пере- менных а и Ь. Следовательно, полагая а=0 и придавая b любое из 20 остальных значе- ний, можно составить 20 неравенств a<Zb. При а—1 переменная b может принимать только значения 2, 3, 20, т. е. всего 19 раз- личных значений. Те же рассуждения приме- нимы и при значениях а, равных 2, 3 и т. д. Наконец, при а=19 переменная b может при- нимать только одно значение: Ь=20. Так как 20+19+...+3+2+1 = 210, неравенство a<Zb можно составить 210 способами. 16. Предположим, что неравенству удовлетворяет какая-то пара чисел (а, 6). Так как а и Ь — натуральные числа и а>Ь, справедливо неравенство а—&^1. Умножая обе части исходного неравенства на а—Ь, по- лучаем а+Ь>аЬ(а—b) ^ab, поскольку а—&:>!. Из неравенств a-\-b>ab и а>Ь следует, что 2а>аЪ, откуда 2>Ь, т. е. Ь=1. При Ь— 1 исходное неравенство преобразуется к виду (а+1)/(а—1)>а, откуда а+1>а2—а, или а2—2а—КО. Прибавляя к обеим частям последнего неравенства по 2, получаем а2—2а+1<2, т. е. (а—1)2<2. Последнему неравенству удовлетворяет только одно натуральное число: а=2. Как по- казывает проверка, пара чисел а—2, 6=1 действительно удовлетворяет исходному нера- венству. 186
Другой вариант решения задачи. Преобразуем исходное неравенство: ab< a+b — a~b+2b —1-| 26 а—Ь а—Ь а—Ь Так как а>&, выполняется неравенство или ab<2b+l. (1) Если существует какая-то пара чисел, удовлетворяющих исходному неравенству, то эти же числа должны удовлетворять неравен- ству (1), которое можно представить в виде Ь(а—2) 1. По предположению, Ъ<а, поэтому нату- ральное число Ь может быть равно только 1. Число а может принимать значения 1, 2 и 3. Как показывает проверка, исходному нера- венству удовлетворяет единственная пара чисел: а—2, Ь=1. 17. а) —> — ; 2х 2 х<1, например, х=0,5; х=0,25. б) />1, папример, 1=1,5; t—2. в) ; 2г4 2 z4=l, например, z=l, z——1. 18. x2-f-x2-{-2x-}-l-|-x2-|-4x-+-4> >x24-6x-f-94-x24-8x+ 164-х24-10х4-25, 187
откуда 19. Раскроем скобки и, перенеся все чле- ны в левую часть уравнения, приведем подоб- ные члены. После этого разложим получив- шееся выражение на линейные множители: (х2+х+1) (2х24-2х—3)=—3(1—х—х2); 2x4-j-2x3—3x24-2x3-f-2x2- -Зх+2х2+2х—3= ==—3-f-3x-f-3x2, 2х4-|-4х3 — 2х2—4х=О, х4+2х3—х2—2х=0, х3(х-{-2)—х(х-|-2) —О, (х+2) (х3——х)=0, (х-1)х(х+1)(х+2)=0, Х] = 1, х2=0, хз=—1, х4=—2. 20. Если t/=0, то х<4, (1) 2х> 10, (2) т. е. система неравенств не имеет решений. Если у— 1, то х<3, (1) 2х>5, и система неравенств также не имеет реше- ний. Если у—2, то х<2 2х>0, п система неравенств допускает единственное решение: х~1. Если же у—3, то х<1, т. е. система допускает единственное решение: х=0. При у^4 в силу неравенства (1) х+у<4: система неравенств не имеет решений. Итак, задача может иметь не более 2 ре- шений. Как показывает проверка, пары чисел 188
(1, 2) „ (О, 3) удовлетворяют условиям задачи. 21. Разрешим второе уравнение y+z— 11 относительно z и подставим z—11—у в пер- вое уравнение: 7х4-5у— (11— */) = 8, или 7x4-6*/= 19. Так как х и у — целые неотрицательные числа, должно выполняться неравенство 0^х^2 (в противном случае 7-3=21 >19). Если х=0, то мы получаем уравнение бу—19, не имеющее решений в целых числах. Следо- вательно, х=0 отпадает. При х= 1 мы полу- чаем у—2 и z=9. При х—2 уравнение также не имеет решений в целых числах. Итак, нсходйая система уравнений может иметь единственное решение: (1, 2, 9). Как показывает проверка, эта тройка чисел дей- ствительно удовлетворяет обоим уравнениям. ЗАНИМАТЕЛЬНАЯ ГЕОМЕТРИЯ 1. Положим кирпич на угол стола так, чтобы 2 его грани совместились с боковыми гранями столешницы. Отметив карандашом на поверхности стола, где проходит грань, перпендикулярная краю стола в точке М, сдвинем кирпич на его длину вдоль края стола так, чтобы грань, располагавшаяся у края стола, совместилась с отметкой М. Затем линейкой измерим по воздуху длину отрезка от угла стола до наиболее далекой вершины («угла») кирпича, лежащей в грани, 189
которая проходит через отметку М перпенди- кулярно поверхности стола. Другое решение: уложить линейку так, чтобы ее обрез совпадал с диагональю верх- ней грани кирпича, и, сдвинув вдоль диаго- нали на длину диагонали, измерить требуе- мое расстояние. 2. (1) а, с, d, f. (2) Ъ, с, f. 3. 190
4. 5. Письмо адресовано Доретте. 6. 7. Номера рисунков 1 2 3 4 5 6 Фигуры ' а d b d c a Номера рисунков 7 8 1 9 10 11 12 Фигуры d b с d e c 191
Требуемся найти < Sp ’Skb = 3 Найти x 1 + 4)2: (i.)2 = = <S₽ + Skb> откуда (x + 8) x = 2, x=8. = 7273-98*, S*96,4 8. Сложим квадрат пополам, отметим ли- нию сгиба АВ, после чего развернем и, сло- жив пополам в другом направлении, отметим линию сгиба СО. Пересечение линий сгиба 192
АВ и CD даст нам точку О. Сложив и отме гив линии сгиба, разогнем каждую из поло- вин, на которые делит квадрат отрезок CD. Веригину Е правильного шестиугольника мы найдем, наложив угол с вершиной К па остальную часть квадрата так, чтобы точка А осталась на месте, а вершина К легла на сгиб, делящий пополам отрезок АО. Найдя верши- ну Е, проведем сгиб АЕ, после чего найти вершины F, G и Н уже не составляет труда. 9. b = 5a; с=6п; d = 4a тав, сколько полей шахматной доски целиком попадает в прямоугольные треугольники АВМ и DCM, возникающие при построении па доске треугольника AMD. Так жак треуголь- 13 Ф-73 193
ники АВМ и DCM конгруэнтны, достаточно подсчитать, сколько полги целиком помеща- ется в одном из них. Введем на шахматной доске прямоуголь- ную систему координат, положительные по- луоси которой х и у совпадают с лучами АВ и AD. За единицу длины по обеим осям вы- берем длину стороны поля. Так как и квад- рат, и треугольник принадлежат к числу вы- пуклых многоугольников, любое поле шахмат- ной доски целиком лежит внутри треугольника АВМ, если в этом треугольнике лежат вер- шины поля. Пусть [х, у] — координаты левой нижней вершины поля. Тогда 3 остальные вершины поля имеют координаты [x-f-1, у], [х, t/H-1], [х-j-l, 1/+1]. Прямой, проходящей через точки А и М, соответствует уравнение */=(1/2)х. Поле шахматной доски попадет внутрь треугольника АВМ, если координаты вершин поля удовлетворяют неравенствам 0<х<7, 0<f/C7, </«£ -±х, y+\sZ-Lx. (1) Таким образом, задача будет решена, если мы найдем все упорядоченные пары [х, у] целых чисел, удовлетворяющих нера- венствам (1). Придавая х целые значения от О до 7 и находя из неравенств (1) допусти- мые значения у, получаем: X 0 1 2 3 У X 4 5 0 6 0 7 У 0; 1 0; 1 0; 1; 2 о to 194
При составлении таблицы третье неравен- ство, следующее из четвертого, можно отбро- сить. С полным основанием мы можем отбро- сить также второе неравенство, так как оно следует из первого и третьего неравенств. Итак, при составлении таблицы достаточно воспользоваться неравенством и первым и четвертым неравенствами (последнее удоб- нее записать в виде у=~-х—I). Мы пслучаем 12 полей, левые нижние вершины которых лежат в точках с координатами [2, 0], [3, 0], [4, 0], [4, 1], [5, 0], [5, 1J, |6, О]. [6, 1], [6, 2J. [7, 0], [7, 1], [7, 2J. Столько же полей целиком умещается в треугольнике DCM. Итак, с треугольником AMD не имеют об- щих внутренних точек 24 поля шахматной доски. ТРЕНИРОВКА ПО СОВРЕМЕННОЙ МАТЕМАТИКЕ 1. а) Число У2 п отличное от него у8 ир- рациональны, произведение же этих чисел У2«У8=4 рационально. Следовательно, утвер- ждение _«а» лоясно. б) У2 и —У2 — два различных иррацио- нальных числа. Их сумма У2-|-(—У2)=0 — рациональное число. Следовательно, утвер- ждение «б» ложно. в) Предположим, что существует рацио- нальное число г и иррациональное число х^ сумма которых г+х рациональна. Тогда суще- ствуют такие целые числа а, Ь=^0, с и d=^0, (3* 195
что r=ajb и r~^x=cid. Следовательно. x=c/d—a/b = (be—ad) /bd, что противоречит предположению об иррациональности числа х. Итак, мы доказали, что утверждение «в» истинно. (То же самое можно доказать ина- че, а именно, сославшись на теоремы о том. что разность двух рациональных чисел ра- циональна.) 2. Пусть п — произвольное натуральное число. Тогда /г-Н —число, следующее за ним. и удвоенное произведение числа п и следую- щего за ним числа равно 2п(п+1). Сумма квадратов числа п и следующего за ним числа равна н2Ч-(n-Н)2, а так как по усло- вию задачи удвоенное произведение чисел на 1 меньше суммы их квадратов, должно вы- полняться равенство 2п(п+1) + 1=/г2+(н+1)2. Раскрывая скобки, получаем тождество 2п2+2п+1 =n2+n2+2n+1, которое выполняется при любом натуральном п, что и требовалось доказать. 3. Нетрудно видеть, что tga=V3, tgp—!/7. По формуле тангенса суммы углов, примени- мой к данному случаю, получаем: g( +Р) 1-tga.tgp j____. JL 3 7 10 20 21 20 2 ’ 21 t. e. tg (a+p) = 1/2. Из таблиц тригонометрических функций 196
находим значения tg £6,5°, tg 26,6° и убежда- емся, что tg26,5°<1/2<tg26,6°. А так как 0°<а<45° и 0°<р<45°, неравенство тан- генсов соответствует неравенству углов 26,5°<а+0<26,60, что и требовалось дока- зать. 4. Поскольку 1-6+3-3=15 и 24—15=9, то для того чтобы средняя оценка по трем остальным предметам была выше 2, сумма оценок по этим предметам должна быть мень- ше 9. Действительно, 24 : 12=2, но 23 : 12<2. Итак, по остальным трем предметам студенту необходимо получить по крайней мере одну двойку (напоминаем еще раз, что в ГДР при- нята пятибалльная система, в которой выс- шей оценкой считается единица). 5. а) В геометрической прогрессии отно- шение последующего члена к предыдущему постоянно (и называется знаменателем про- грессии). Обозначим знаменатель прогрессии через q. Тогда 3 соседних члена прогрессии можно записать, например, в виде — , /и, niq q (такая запись членов геометрической прогрес- сии для нас удобнее более привычной записи а, aq2\ разумеется, обе записи эквивалент- ны: в этом нетрудно убедиться, положив в традиционной записи a=m!q). По условиям задачи — -\-m-\-mq—19, (1) q +m2+wiV=133. (2) <72 197
Подставляя x—q-{-\/q, получаем m(x-f-l) = 19, (Г) /n2(x-f-l) (х—1) = 133, (2') откуда 19 133 х=——1 п соответственно х2= —-+1. , т т2 Приравнивая оба выражения для х2, находим: тг=6их=Д2-, а также <7= —п 0= —. 6 3 2 Итак, условиям задачи удовлетворяют 2 геометрические прогрессии: 4, 6, 9 и 9, 6, 4. б) Запишем условия задачи в виде двух уравнений: 04-098=13, (1) aq-j-aq2—4. (2) Решив их, мы получим две геометрические прогрессии: Vs, 4/s, l6/s, 64/s и 64/5, 16/5, 4/5, 1/5. 6. а) Исходное неравенство 8(2х4-1)/5< <3x4-2 с помощью эквивалентного преобра- зования приводится к виду 16x4-8<15x4-10, или х<2. Следовательно, ему удовлетворяют все действительные числа, которые меньше 2. б) 1. Р1={0,1} 2. Р2={—3; —2; —1; 0} 3. М={0} 7. Вероятность, с которой автобусы, сле- дующие по тому или ино’му маршруту, при- бывают на остановку, зависит от относитель- ной частоты, с которой они курсируют на ли- нии. Например, в течение 30 мин на остановку успевают прибыть 6 автобусов, следующих по маршруту с интервалом 5 мин, 15 авто- бусов, следующих с интервалом 2 мин, 3 ав- 198
тобуса, следующих с интервалом 10 мин, и 2 автобуса, следующих с интервалом 15 мин, т. е. всего 26 автобусов, из которых 15 при- ходится на долю следующих с интервалом 2 мин. Значит, относительная частота авто- бусов, следующих по маршруту с интервалом 2 мин, составляет Ц--0.577, т. е. примерно каждый второй автобус сле- дует по маршруту с интервалом движения 2 мин. Как кошка с собакой а) Так как все функции линейны, их графики имеют вид отрезков прямых и, следователь- но, однозначно определяются концевыми точ- ками. Например, для функции fi с областью определения и аналитическим выра- жением t/=x/2-Ml началом отрезка служит точка с координатами х=4; «/=24-11 = 13, а концом — точка с координатами х=8; 199
(/==4+11 = 15. Графики всех 14 функций об- разуют «портрет» собаки, который вы видите на рисунке. б) Фун ция к- Область определения Аналитическое выражение h 4<t/^6 Х=1 /2 4^t/^6 x=-l /з ~ l^X-^1 (/=| । к / —x+5 при —l^x^O, х1+ь- ( x+5 при O^x^l. 4,5<t/^5 x—0,5 /б 4,5^//^5 x=—0,5 —x+4,5 (в ~ 0,5 <x ^0,5 ,= |л-|+4,5= {ПРИХ+°4^Х^°* при 0s^x^0,5 h —0,25 5^ x 0,25 у=4 h —l:gxsC0 У=— 3x+l h —0,5sgxC0 0=2x-M Ao —0,5sJxC7 y=-0 fu G^x<7 y=—x+7 /12 3^№g6 y=——1 3 /13 0;gysg2 -x=3 /14 l=gx<3 x+5 8. Плоскость а параллельна плоскости р(а||р) в том и только в том случае, если а=р или аПр=0. Прямая g параллельна прямой h. (g||ft) в том и только в том случае, если g=h или gfth=0. Переходим к доказательству теоремы. По предположению, аПв¥=0. Следователь- но, существует прямая g=af|e. Кроме того, РР)8:/=0. Следовательно, существует прямая ft=Pf|e. Так как gee, и Лее, эти прямые ле- жат в одной общей плоскости, а именно, в плоскости е. Так как а||р, необходимо разли- чать следующие случаи. I) а=р=*я=аР!8=рП8—h, т. е. g||ft. 200
2) a(\fi—0=>g(]h= (afle)fl (РПе)== аПеПР Ле= (аП₽) П (8Л8) = 0Ле=0- Следовательно, и в этом случае gllft. в) Тетраэдр (прямой, неправильный). 10. Можно. Для этого карандаши следует уложить в коробку слоями, содержащими по- переменно то 20, то 19 штук. Карандаши верхнего слоя должны располагаться в про- межутках между карандашами нижнего слоя. При таком способе упаковки в коробку вой- дет 5-204-4-19= 100+76= 176 карандашей. При рекомендуемом нами способе упаков- ки в коробку войдет более 160, а именно 176 карандашей. 11. Из условия «а» следует, что если от- дел В не принимает участия в обсуждении проекта, то отдел А также не участвует в об- суждении проекта. Иначе говоря, если отдел А принимает участие, то В также участвует в обсуждении проекта (обратное* утвержде- ние). Из условия «б» мы заключаем, что если отдел А принимает участие в обсуждении про- екта, то отдел С непременно участвует в об- суждении. 201
12. Из (4) следует, что число 2 принадле- жит как множеству А, так и множеству В. Из (5) мы заключаем, что числа 2, 4 и 8 при- надлежат как множеству В, так и мно- жеству С. Таким образом, число 2 принадле- жит всем 3 множествам, в то время как числа 4 и 8 не принадлежат множеству А, ибо в противном случае мы пришли бы к проти- воречию с условием (4), Из (1) следует, что число 1 не принадле- жит ни множеству А, ни множеству В, а из (2) и (3) — что число 1 принадлежит множе- ству С. С помощью аналогичных рассуждений мы заключаем из (1), (2) и (3), что число 3 принадлежит только множеству А, число 5 принадлежит только множеству А, число 6 принадлежит только множеству В, число 7 принадлежит только множеству А. Итак, множества А, В и С содержат сле- дующие элементы: Л={2, 3, 5, 7), В={2; 4, 6, 8}, С={1, 2, 4, 8}. 13. Обозначим длины ребер упаковки 10 спичечных коробков через А, В и С. Пред- положим, что вдоль ребра А упаковки ребро а спичечного коробка укладывается х раз (А = ах), вдоль ребра В упаковки ребро b Спичечного коробка укладывается у раз (В=Ьу) и вдоль ребра С упаковки ребро с спичечного коробка укладывается г раз (С=сг). (}бъем упаковки равен АВС=аЬс хуг. Так как abc — объем одного спичечного коробка, то произведение хуг задает число спичечных коробков в упаковке. Следователь- но, х«/г=10. Так как х, у, г—могут быть только положительными целыми числами, то 202
существует всего 9 возь представленных в следую х | 1 | 1 | 10 | 1 ложь щей 1 1ЫХ таб. 2 вар лице 2 шан ь • 5 тов, 5 У 1 10 1 2 5 1 5 1 2 2 10 1 1 5 2 5 1 2 1 Расход бумаги на одну упаковку равен площади поверхности прямоугольного парал- лелепипеда, составленного из 10 спичечных коробков (припуски на клапаны, необходи- мые для склеивания обертки, не учитывают- ся), т. е. 24 В4-2Д С+2ВС=2 (abxy-^acxz+bcyz). Поскольку расход бумаги требуется свести до минимума, величина W—abxy-Vacxz^bcyz также должна быть минимальной. В 9 перечисленных выше возможных ва- риантах величина W принимает значения 28709, 26414, 17054, 24918, 24 153, 19718, 17 678, 15 833 п, наконец, 14 558 (при х=5. у—29 z=l). Итак, наиболее экономичной по расходу оберточной бумаги оказывается упаковка с х=5, у=5, г=1. Именно ее и используют в торговой сети. 14. в) Пожелание невыполнимо. Действитель- но, если бы проектировщики могли учесть все пожелания, то число дорог было бы равно 7, тогда как оно должно быть четным (каждая дорога считается дважды, так как соединяет 203
б) Первый вариант в) Второй вариант два населенных пункта, а подсчет произво- дится по числу дорог, выходящих из каждого пункта). МАТЕМАТИКА НА КАЖДОМ ШАГУ 1. Условия задачи позволяют записать уравнения (х — стаж одного собеседника, У — другого): у=2х, (1) у—2x3 (х—2), (2) откуда х=4, у=8. Итак, один собеседник состоит в шахмат- ном клубе 8 лет, другой — 4 года. 2. Схема переправы приведена в следую- Южный берег Северный берег 1 т at b (Д, а) 1 о а 2 т a, bt с 2 о а, b 3 т а, Д, Ь, В 3 о а, Д 4 т а, Д, В, С 4 о Д, В, С 5 т Д, В, С, а, b 5 о Д, В, С, а (Д, В, а, Ь) 6 тД, В, С, а, Ь, с Д, а, В, b, С, с Д, В, С, с (В, b. С, с) Д, В, b, С, с, Д, В, С Д, В, С, с С, с в, b, С, с b, С а, Ь, с Ь, с (С, с) 204
щей таблице. Буквы А, В и С означают му- жей, а, b и с — их жен (так что А, а; В, b и С, с — супружеские пары). Число рейсов ука- зано вместе с направлением, в котором лодка пересекает реку: т — туда, о — обратно. В слу- чае, когда имеются 2 варианта, второй ва- риант указан в круглых скобках. 3. Пусть х — число распроданных коври- жек по 17 марок за штуку, у — число распро- данных коврижек по 12 марок за штуку. Тогда 17х4-12у=478, (1) где х и у — целые положительные числа. Раз- решив уравнение (1) относительно у, полу- чаем: </=39-x+ Так как у — целое число, а 5 и 12 — взаимно простые числа, (2—х) должно делиться на 12, т. е. 2—х=12/, где t—целое число, или х=2—12/. Подставляя х в (2), получаем (/=374-17/. По условиям задачи х>10 и t/>0, поэтому /=—1 и, следовательно, х=14, у=20. Итак, было распродано 14 коврижек по 17 марок за штуку и 20 коврижек по 12 ма- рок за штуку. 4. Из 100 коллекционеров 30 не собирают старинные монеты, 25 не интересуются знач- ками, 20 равнодушны к этикеткам от спичеч- ных коробков и 15 без энтузиазма относятся к филателии, т. е. (304-254-204-15) =90 лю- бителей не занимаются по крайней мере од- ним из четырех видов коллекционирования. Следовательно, 10 коллекционеров соби- рают и монеты, и значки, и этикетки, и мар- 205
ки. (На рисунке СМ монеты,' Зн — значки, марки.) 5. Из 50 членов об- щины 10 говорят толь- ко на родном (немец- ком) языке. Остальные 40 членов общины, кроме родного языка, владеют также фран- цузским или итальян- ским языком. Так как означает старинные Э — этикетки и М — 20+35=55, а 55—40= = 15, мы заключаем, что 15 членов общины говорят и по-французски, и по-итальянски. 6. Для любых натуральных чирел а и b из допустимых множеств (размер ботинок и возраст) выполняется тождество [ (2а+39) -50+29]— (1979—Ь) = 100а+&. 7. В первом случае потребуется веревка длиной 4о+8с+46, во втором — веревка дли- ной 4а+4&+4с и в третьем — веревка длиной 6а+4с+6б. Самая короткая веревка потребуется во втором случае, так как 206
4a-j-8c-\-4b >4a-|-4&4-4c и 6a4-4c-|-66>4a+4d4-4c. Самая длинная веревка потребуется в третьем случае, так как (4а4-8с+4Ь) — (4а-(-4б+4с) =4с, (6a+4c+6b) — (4a+4&+4c) = 2а±2Ь, а по условиям задачи a-j-b>2c, вследствие чего 2а-Ь-2Ь>4с. Итак, длины веревок удовлетворяют нера- венствам (2)< (1) < (3). 8. Пусть х — число порций рыбы, приго- товленных искусными поварами, у — число столов. Тогда !Н-1=х, (О 2(у—1)=х, (2) откуда «Н-1=2(|/—1) и у=3, х=4. Итак, повара приготовили 4 порции рыбы, а для гостей было 3 стола. 9. Предположим, что до снижения цен входной билет в цирк стоил х марок (рублей или других денежных единиц), а на пред- ставлении побывало п зрителей. Выручка от продажи билетов составила пх марок. После снижения стоимости входной билет стал стоить 0,7х марок, число посетителей возрос- ло на у человек, а выручка осталась прежней, т. е. ПХ==(я-Н/) -jy X, или пх= 7 7 3 7 207
Зп=7у, </=-~-n«0,43n Итак, число зрителей возросло примерно на 43%. 10. Высота дерева относится к длине от- брасываемой им тени так же, как высота столба к длине своей тени, т. е. х: 10=3:2 х—15. Hiai<. Высота дерева равна 15 м. 11. гППП 12. Так как (*/2) • (2/з) ~Чз< пассажир про- спал ’/з пути, v 208
13. Цифры у каждой деревни указывают продолжительность в мин самого короткого маршрута, ведущего к ней из пункта А. Крат- чайшие маршруты показаны на плане более жирными линиями. Следуя кратчайшим маршрутом, из А в В можно добраться за 60 мин. 14. 15. . . 4 846 б) , 1 054 , I 054 , I 065 , 1 076 а) +4|24 + 9 478 + 9 482 + 9 578 + 9 658 + 9 710 10 532 10 536 10 643 10 734 + 6 784 25 464 в) Приведем одно из решений: , 992 992 1 984 (4 Ф-73 209
г) Приведем два решения: , 286 . 467 ф 923 942 1 209 1 409 (Остальные 30 решений этого криптарифма предлагаем желающим найти самостоятель- но.) 16. Пусть х— число подлежащих распро- странению билетов лотереи. Тогда 5 •№ .§.1.19° = 194 000,.№38 800 45 Итак, чтобы израсходовать намеченную сумму в 87 300 франков на выплату выигры- шей, организаторам лотереи необходимо рас- пространить 38 800 билетов стоимостью по 5 франков. 17. Пусть х — стоимость пригласительного билета для члена садового кооператива, а у— для гостя. Если бы сборы от продажи пригла- сительных билетов должны были составить ровно 420 марок, а стоимость пригласитель- ного билета для члена кооператива была вдвое меньше стоимости пригласительного би- лета для гостя, то мы получили бы систему уравнений 150х+100//= 420, //=2х и, решив ее, нашли бы, что х=1,20 и // = 2,40. Поскольку задача сформулирована не столь четко («расходы, по мнению членов правления, должны составить не менее 420 марок», «стоимость пригласительного билета для гостей должна превышать стоимость при* 210
гласительного билета для членов кооперати- ва, но не более чем в 2 раза»), ее условия можно записать в виде системы двух нера- венств: 150х+100у>420, 2х>у. Существует бесконечно много вариантов цен на пригласительные билеты, удовлетво- ряющих этим неравенствам. Например, если бы расходы должны были составить не более 500 марок, то 250 участников праздника долж- ны были бы в среднем заплатить по 250 ма- рок. Из множества возможных решении прав- ление могло бы отдать предпочтение вариан- ту, при котором входной билет для члена кооператива стоил бы 1,50 марки, а входной билет для гостя — 2 марки. Выручка от про- дажи пригласительных билетов составила бы в этом случае 425 марок. 18. Пусть х, у, z, и, v -число монет до- стоинством 50, 20, 10, 5 пфеннигов и 1 пфен- ниг соответственно. Задача сводится к реше- нию в целых неотрицательных числах урав- нения 50х+20у +10z+5«+v = 100 (1) при условии, что х^2, у^5, z^lO, w^20 п иС 100. Все решения уравнения (1) можно было бы найти, прямым перебором, составив таб- лицу, но такой подход слишком громоздок, так как число решений очень велико. Мы будем исходить из другого. Прежде всего обратим внимание на то, что суммы 5и-[-у и 20//+10Z делятся на 10 и поэтому могут принимать только значения 0, 10, 20, 14* 211
100. Кроме того, х может принимать толь- ко значения 0, 1 или 2. Уравнение 5«+i>=0 допускает единствен- ное решение, удовлетворяющее этим условиям (и=1>=0), а уравнение 5«+v=10— ровно 3 решения (ы=0, ц=10; w=1, и=5; «=2, у=0). В следующей таблице показано, сколько решений, удовлетворяющих всем условиям, допускает уравнение 5«+у=Л при Л=0, 10, 20, ..., 100- и уравнение 20y+10z=B при В= 100, 90, 80, ..., 10, 0. А Число решений В Число решений 0 1 100 6 10 3 90 5 20 5 80 5 30 7 70 4 40 9 60 4 50 11 50 3 60 13 40 3 70 15 30 2 80 17 20 2 90 19 10 1 100 21 0 1 При х=0 число решений уравнения (1) равно сумме произведений чисел, стоящих в одной строке во втором и четвертом столб- цах (так как Л+В= 100, и любое решение уравнения 5и-|-у=Л может сочетаться с лю- бым решением уравнения 20z/+10z=B): 1-6+3-5+5-5+7-4+9-4+11-3+13-3+15-2+ + 17-2+19-1+21-1 = 1-(21 + 19)+2-(17+ + 15)+3- (13+11)+4- (9+7)+5- (5+3) + +6-1 = 40+64+72+74+40+6=286. При х--1 число решений уравнения (I) 212
равно сумме произведений чисел, стоящих во втором столбце в первых шести строках, а в четвертом столбце — в шести последних стро- ках (что соответствует Л+В==50): 1.34-3.3+5-2+7*2-1-9-1 + 1Ь 1=3- (1+3) + +2- (5+7) +1. (9+11) = 12+24+20=56. Наконец, при х=2 число решений уравне- ния (1) равно 1 (так как х=2 соответствует /1+8=0). Итак, уравнение (1) допускает всего 286+56-|-1 = 343 решения: 1 марку монета- ми достоинством в 50, 20, 10, 5 пфеннигов и в 1 пфенниг можно разменять 343 способами. 19. Пусть у — число миль, проходимых за день одним, а х — другим гонцом. Тогда r/=x+2, (1) 12x+12z/=260. (2) Подставляя (1) в (2), получаем 12х+12(х+2) =260, откуда '-’4- Таким образом, один гонец проходил за 5 5 день 9 — миль, а другой— Ну миль. СЛОВО ЗНАМЕНИТЫМ МАТЕМАТИКАМ 1. Такие числа х должны удовлетворять соотношениям №б0«+1, х=7а, где п и а — некоторые целые числа. Итак, 60п-|-1 = 7а, откуда 213
„_..60п-Н 7 ’ а=8«+ 12±1 . 7 Целым положительным значениям а соответ- ствует /1=5, 12, 19, ... При п=5 х=301, при п=12 х=721, при п=19 х=1141 и т. д. Эта задача допускает простое решение, если следовать Бхаскаре. В прошлом веке од- ному математику для «доказательства» пра- вильности результата, полученного Бхаска- рой, понадобилось несколько страниц. 2. Пусть х — число дней. Тогда х—1 iq_ х 6 + 5 ' Число дней равно 85. 3. Пусть х — одно из слагаемых числа 10. Тогда хЧ-(Ю—х)2=58, Xi=7, Х2—3. Итак, слагаемые, о которых идет речь в задаче Аль-Хорезми, равны 7 и 3. 4. С каждым прыжком гончая уменьшает расстояние, отделяющее ее от зайца и перво- начально составляющее 150 футов, на 2 фута: 9—7= 2, 150 : 2= 75. Гончая догонит зайца за 75 прыжков. 5. Если гири при взвешивании разреша- ется ставить только на одну чашу весов, то 214
для взвешивания любой массы ее необходимо представить в виде суммы масс имеющихся гирь. (Разумеется, при взвешивании одной массы любая гиря может быть использована только 1 раз.) Следовательно, если рь р2, рз, Pt и ръ — массы гирь, то массу Q^30 кг лю- бого тела необходимо представить в виде Q = alPl+a2P2-l_O3P3+<l4P4+fl5p5. где коэффициенты аь а2, о3, а4 и а5 равны либо единице (если соответствующая гиря по- ставлена на чашу весов), либо нулю (если гиря не поставлена на чашу весов). В такой постановке задачи нетрудно усмотреть ана- логию с представлением числа Q в двоичной системе счисления. Массы гирь р\, р2, Рз, Pi и рз достаточно выбрать равными 1, 2, 4, 8 и 16 кг. Сумма масс равна 14-24-4+8+16=31, т. е. больше 30. Кроме того, любое число Q, не • превышающее 30, можно представить в виде Р=Ь424+Ьз23+г»222+6121+&о2°, где коэффициенты 64, b3, b2, bi и Ьо, как и требуется, равны либо нулю, либо единице. 6. 7. Пусть х — число цветов лотоса в венке. По условиям задачи 215
-Lx+J-x-f- -l_x+-L-x+6=x, 3 □ о 4 откуда х= 120. Итак, венок сплетен из 120 цветов лотоса. 8. Пусть х — сумма денег, внесенная на покупку дома третьим подмастерьем. По условиям задачи 12х-|-4х-|-х=204, Откуда х=12. Итак, на покупку дома первый подмастерье дал 144 гульдена, второй внес 48 гульденов, а третий— 12 гульденов. 9. 10. Пусть х — стоимость 1 яблока, а у — стоимость I груши в динарах. Тогда 9х—у =13, (1) 15у—х=6. (2) Разрешив (2) относительно х, подставим полученное выражение х=15у—6 в (1): 134у=67, откуда у=0,5. Следовательно, х=1,5. Итак, 1 яблоко стоит 1,5 динара, 1 гру- ша — 0,5 динара. 216
Н. Из' произвольной точки А заданной окружности радиусом АО сделаем подряд 3 засечки так, что AB — BC=CD — AO. Во- круг точек А и D как вокруг центров опишем 2 окружности радиусом AC—BD, Пусть Е и Е' — точки пересечения этих окружностей, а F и F' — точки пересечения заданной окруж- ности с окружностью радиуса О£, проведен- ного из точки А как из центра. Тогда AF — сторона квадрата, вписанного в данную окружность. Доказательство. В прямоугольном треугольнике A DC по построению AD = 2CD и, следовательно, AC2=3CD2. Отсюда мы заключаем, что в прямоугольном треугольни- ке АЕО ОЕ2=2ЛО2, т. е. OE=AF=AO]/2— сторона квадрата, вписанного в данную окружность. 12. Пусть у — доля, которую составляет прирост травы за 1 неделю от первоначаль- ного ее количества на лугу площадью в 1 га. Тогда прирост травы на первом лугу за I не- делю составит ЗУз^ долей, а за 4 недели — 217
48 \ 3 3 Аналогичным образом травы съела за 1 неделю 3’/з^-4 долей от того количества травы, кото- рое первоначально было на лугу площадью в I га, что эквивалентно увеличению перво- начальной площади первого луга до (3'/з+ +40/з.У) га. Коровы съели столько травы, сколь- ко ее было бы в первый день на лугу пло- щадью (3*/з4~40/зУ) га. За 1 неделю 12 коров съели ’/< травы, а 1 корова за 1 неделю съела ’Ль всей травы, т. е. столько травы, сколько было первоначально на лугу площадью 1 1 . 40 \ IO-НО// —13 — + — v I — ——— га. 144 ВЫЧИСЛИМ, СКОЛЬКО 1 корова на втором лугу: прирост травы на 1 га луга за 1 неделю: у, прирост травы на 1 га луга за 9 недель: 9 у, ' ' прирост травы на 10 га луга за 9 недель: 90 у. Итак, для того чтобы прокормить 21 ко- рову в течение 9 недель, хватило бы травы, которая первоначально была на лугу пло- щадью (104-90у) га, а для того чтобы про- кормить 1 корову в течение I недели, хвати- ло бы травы, которая была на лугу площадью Ю+90у Ю+900 9-21 189 Так как, по предположению, все коровы в день съедают одно и то же количество-тра- вы, мы получаем уравнение 10+400 10+900’ 144 189 и, решая его, находим у=х1%. 218
Зная недельный прирост травы на 1 га луга, определим площадь пастбища, способ- ного прокормить I корову в течение 1 недели: 10+40у _ 5 144 54 Теперь у нас уже все готово, чтобы отве- тить на вопрос задачи. Пусть х — число коров, которых можно пасти на третьем лугу в течение 18 недель. Тогда И+М-18'ТГ_ 5 18х 54 ’ откуда х=36. Итак, на третьем лугу в течение-18 недель можно пасти 36 коров. 14. 4=2+2 10=5+5 6=3+3 ....... 8=5+3 46=41+5 48=43+5 15. 13+123= 1 + 1728= 1729, 93+103 = 729+1000= 1729. 16. От 16 июля 1799 г. интересующую нас дату отделяют 2770 дней, так как 8113— —5343=2700 дней. Из них на 1799 г. прихо- 219
дится 197 дней (а именно: 16 дней в июле» 3-31 дней в январе, марте и мае, 2-30 дней в апреле и июне, 28 дней в феврале). На про- межуток времени с 1792 по 1798 г. приходит- ся 2557 дней (5 лет продолжительностью по 365 дней и 2 високосных года по 366 дней). Остается 16 дней (2770—197—2557=16). От- считав их назад от 31 декабря 1791 г., мы узнаем, что интересующая нас дата прихо- дится на 15 декабря 1791 г. НАША СЕКЦИЯ ИГР 2. Наибольшая сум- ма очков достигается на видимых гранях тетракуба слева (она равна 67 очкам), наи- меньшая — на видимых гранях «линейного» тетракуба справа (она равна 36 очкам). 8. Полицейским нужно стоять на пере- крестках 3, 12, 14, 23, 25 и 34. Действительно, полицейские должны держать под наблюде- 220
нием все дорожки городского парка: 6 по го- ризонтали (г), 6 по вертикали (н), 6 по диа- гонали в одном (д) и 6 по диагонали в дру- гом направлении (е). По условиям задачи один полицейский стоит на перекрестке 34, что приводит к единственному варианту рас- становки 5 остальных полицейских: 3 : в3, гь дь е5; 12 : в6, г2, д4, е6; 14: в2, г3, &2, е3; 23 : в5, г4, д5, е4; 25 : в,, г5, д3, et; 34 : в4, гв, дв, е2. 9. 10. Пусть х — число очков, выпавших на первой игральной кости, у — число очков, вы- павших на второй кости, и z — число очков, выпавших на третьей кости. Тогда [(2x4-5) -5+10+у] • 104-z=s, или 100x4- lOy+z=s—350. Пример. Предположим, что на костях выпало 2, 3 и 6 очков. Проделываем вы- числения: 2-2=4; 44-5=9; 9-5=45; 454-10=55; 554-3=58; 58-10=580; 5804-6= 586; 586—350=236. Итак, на первой кости выпало 2 очка, на второй — 3 и на третьей — 6 очков. 221
II. эоя ©же-ээ э ли® я иэ-и и ф-м ий фф Ф<ЕЭП<11ФФЭНЕФ э И В И $м э э с ИЯИ ИИ> ВИ ЯС©Ф Ф ® Ф Ф И В1Э Ф Ф ® я и Э ФЭК Ф Э BL Э И ф-Я Ф ВЭНфффИфВЭЯЯ ЭФЭФЯ>ИФФЯЭФ в эеэ'ф я в в. ©Фи л СЯЭ Ф В Э Я Ф И Э Я Ф ИФЭИФФЯЭ ЯЯФЭ 12. При бросании 1 красной it 1 белой игральной кости общее число исходов равно 6-6=36. Рассмотрим теперь бросание двух неотличимых костей. Если на костях выпа- дает по одинаковому числу очков, то безраз- лично, отличимы ли кости или не отличимы. По одинаковому числу очков на каждой из двух костей выпадает в 6 случаях. В осталь- ных 30 случаях из-за неотличимости костей мы отождествляем исходы бросаний попарно, так как не можем сказать, на какой именно кости выпало одно из двух неравных чисел и на какой другое. Следовательно, вместо 30 в случае одинаковых костей мы насчитываем лишь 15 различных исходов. Итак, при двух, одинаковых костях общее число исходов бросаний равно 21. ПУТЬ, ВРЕМЯ, СКОРОСТЬ 1. Воспользуемся для вычислений форму- лой v—s/t 222
у, =s= км/мин, ^2— 117^.^ км/мин, 123 421 «1« 2,976 км/мин, Oi«178,5 км/ч. о2~ 2,795 км/мин, и2« 167,7 км/ч. Итак, на участке Токио — Нагоя «Хикари» развивает среднюю скорость 178,5 км/ч, а на участке от Токио до Яваты — 167,7 км/ч. 2. В обоих случаях вычисления будем про- изводить по формуле s—vt, разрешив ее от- носительно t. Милан: s1 = o1fb Хаммерфест: s2==v2<2, 6 = -^-, 6= —, t>l v2 А «0,0735 с, , 2900 „ <2= С, 300 000 /2« 0,0097 с. , _ 25 „ ‘ 340 ’ Итак, звук долетает до зрителя, сидящего в зале театра, примерно через 0,07 с, а элек- тромагнитные волны доходят до зрителя, си- дящего у экрана телевизора в далекой Нор- вегии, примерно через 0,01 с. Следовательно, зритель в Хаммерфесте услышит музыку раньше, чем зритель в зале театра. 3. В обеих задачах нам понадобятся фор- мулы для равноускоренного движения: v=gt и s—gt2!2. Исключив t, запишем s как функ- цию от V. s=|^> v=gt, s= g ga ' 2 g2 2g 223
u2 —2gs, /2gs. a) t'i—V2gsi, Vi = )2f-9,81 -80 м/с, Oi«39,6 м/с«143 км/ч. 6) t>2=12gS2, V2=T2-9,81 -5 м/с, u2 «9,9 м/с «35,7 км/ч. Итак, парашютист к моменту раскрытия парашюта успевает в свободном падении раз- вить скорость примерно 143 км/ч, а прыгун с трамплина входит в воду со скоростью око- ло 35,7 км/ч. 4. Высоту свободного падения s вычислим по формуле v2=2gs: Так как 50 км/'ч=125/9 м/с, получаем 1252/92 «9,8 м. Итак, при свободном падении с высоты 9,8 м удар о землю будет таким же, как при наезде на препятствие со скоростью 50 км/ч. 5. Из соотношений f= -у-и получаем t=Tn, t=n —- . <=50- -— «0,114с. 440 224
Таким образом, запись длилась около 0,114 с. За это время камертон совершил 50 колебаний. 6. Анисимова пробежала дистанцию 100 м за 12,77-4-0,001 = 12,78 с, развив среднюю ско- рость v=100/12,78 м/с. Шаллер обогнала Анисимову на расстоя- ние, которое та пробежала за 0,01 с, т. е. на 1.00/12,78-0,01 «7,82 см. Итак, Шаллер на фи- нише была впереди Анисимовой с едва раз- личимым преимуществом около 8 см. 7. Вторая яхта пришла второй потому, что со скоростью 24 км/ч она шла не так долго, как со скоростью 16 км/ч. (Например, если бы дистанция гонки была равна 48 км, то со скоростью 24 км/ч вторая яхта шла бы 24/24=1 ч, а со скоростью 16 км/ч—24/16=;= = 1*/г ч.) Именно поэтому на прохождение дистанции «туда» она израсходовала больше времени, чем ей удалось выиграть на обрат- ном пути. Вокруг часов 8. Если часовая стрелка до того, как обе стрелки совпадут, успеет пройти х минутных делений, то минутная стрелка за то же время пройдет (45-f-x) минутных делений. Так как за одно и то же время часовая стрелка про- ходит */12 того, что проходит минутная, мы можем составить уравнение х= (45-(-х)/12, откуда х=47и- Минутная стрелка совпадает с часовой че- рез 497п мин. 15 ф-тз
9. Пусть »t — линейная скорость конца минутной стрелки, а — часовой. Тогда v,= £U= *J£1 = 2-iL2 =4л см/ч. А 6 1 = 2лг^_ = 2=vh5 = _л_ см/ tt t2 12 4 t<i:t>2=4n: -2- =16:1. Итак, конец минутной стрелки движется в 16 раз быстрее, чем конец часовой стрелки. 10. За 1 ч часовая, стрелка описывает угол 30°, а за 1 мин — угол 0,5°. Минутная стрелка за 1 мин описывает угол 6°. Так как 90: (6— —0,5) = 164/ib минутная и часовая стрелки образуют прямой угол в первый раз через 164/ц мин после того, как обе будут стоять на 12. Поскольку п- 164/11 = 24-60, мы полу- чаем п=88 (в это число входят углы в 0, 90°, 180° и 270°, образуемые минутной и часо- вой стрелками). В течение суток минутная и часовая стрел- ки образуют прямой угол 44 раза. 11. Пусть х—промежуток времени (в мин), который должен пройти прежде, чем стрелки расположатся на одной прямой и будут на- правлены в противоположные стороны. Ми- нусная стрелка успеет пройти за это время х минутных делений циферблата, а часовая— х/12 минутных делений. Когда стрелки рас- положатся на одной прямой и будут направ- лены в противоположные стороны, их будут разделять 30 минутных делений циферблата. Следовательно, в это время 226
x l2 —30, откуда x=328/u. Через 328/u мин после того, как минутная и часовая стрелки совпадут, они будут «смот- реть» в противоположные стороны. 12. В 5°° минутную стрелку отделяют от часовой 25 минутных делений. В тот момент, когда Клаус взглянул на часы, большая стрелка отставала от малой лишь на 3 деле- ния и, следовательно, успела пройти 22 деле- ния. За 1 мин большая стрелка проходит 1 деление, а малая */«2 деления. Следователь- но, за 1 мин минутная стрелка догоняет часо- вую на I—l/t2= H/i2 деления, а для того что- бы пройти 22 деления, минутной стрелке по- надобится 22 : п/|2=24 мин. Итак, Клаус взглянул на часы в 524. 13. Для того чтобы тело могло двигаться по круговой орбите вокруг Земли, центростре- мительная сила Л'ц должна быть равна силе тяготения Fr. Это равенство достигается толь- ко при определенной скорости, так как Pn=niv2/r, FT=mg. =mg, Г 1/Г.637ТО» «7,91 км/с. Г 1000 Итак, первая космическая скорость состав- ляет 7,91 км/с. 14. Разрешив уравнение равномерного дви- жения s=vt относительно t, получим: 227
/= ^910 000000 =(9700с==3281/ мин==5ч 28'/з мин. 300 000 Чтобы преодолеть расстояние от Солнца до Плутона, свету потребуется 5 ч 287з мин. 15. . 71 568 1 71568 + 71568 214 704 Приведем лишь некоторые из 28 решений второго криптарифма: , 825 , 745 д_ 472 , 382 1207 3419 Г 6715 7816 2032 4164 7187 8198 16. До места встречи велосипедист успе- вает проехать на 5 км больше, чем возница, поэтому 5вел.—5 = $воз. . ПоДСТИВЛЯЯ $вел. — = 15/вел.. 5воз ==10(/вел.+1), ПОЛучаеМ 15/ вел. 5= 10(/ вел. +1), откуда /вел. = 3, sвел. = 45, SBo3. = 40,Go3.==4. Пусть АС—х км и ВС—х—5 км. Тогда / _ *—5 Г вел.-и Г воз. =• - 10 Ю Кроме того, по условиям задачи /воз4 — /вел. 4~-— » □ поэтому 228
и, следовательно: х—5 х+20 10 15 ' откуда х=55. Итак, расстояние между хутором В и го- родком С равно 55 км. Велосипедист догоняет возницу в 10 ч утра, не доезжая 10 км до го- родка С. 17. Предположим, что спортсмен достигает скорости 90 км/ч за время t на расстоянии s от старта. Если и0 — начальная скорость, а — ускорение, то v=vo+at, а 72000 _25 с 3600-0,8 Определим теперь расстояние $: __J_ at2, s= И222.25+ — -0,8-252=375 м. 3600 2 Спортсмен-саночник разовьет скорость 90 км/ч через 25 с после старта, пройдя 375 м пути. 18. За 1 ч теплоход проходит против те- чения 2/9 расстояния, а по течению — Уз рас- стояния. Разность (равная 79) соответствует удвоенной скорости течения. Следовательно, за 1 ч плывущая по реке пустая бутылка проходит Vis расстояния. 229
Итак, то же расстояние пустая бутылка проплывает за 18 ч. 19. Пусть и„ — скорость пассажирского, v т— скорость товарного поезда, а До — раз- ность скоростей. Тогда оп=ит 4-Ао, —= +дс, tn tT ’ SiT = S/n4-Au^„/T - s = WT _ .240-7r5L9,5. =8550 M G 2 (мы воспользовались здесь тем, что 4 м/с= =240 м/мип). Таким образом, длина туннеля равна 8550 м. 20. Скорости встречных поездов склады- ваются: u=ui 4-02=454-36= 81 км/ч, и 81000-6 __ „ -3600 Итак, длина первого поезда составляла 135 м. БЕСЕДЫ НА ЕСТЕСТВЕННОНАУЧНЫЕ ТЕМЫ 1. Массу оболочки вычислим по формуле m=Sx, где S — площадь поверхности оболоч- ки, а х — масса ее квадратного метра: S=4№, иг=4лг1 2-х=4-3,14-42-240«48 200 г. 230
Масса оболочки шара-зонда равна при- мерно 48,2 кг. 2. На рисунке показано, как следует рас- положить 7 труб в пакете (1 труба в центре и 6 соприкасающихся труб вокруг нее), для того чтобы их можно было обвязать самой короткой лентой. Обозначим длину ленты I. Тогда /=6-10+л10=10(6-Н0 «91,4 см, т. е. минимальная длина ленты равна 91,4 см. 3. Батарея мопеда обладает зарядом Q=4,5 А-ч. Мощностью электрической лампы называется величина P=U1, где U — напря- жение, / — сила тока, откуда I=PIU=i =0,6/6=0,1 А. Поскольку заряд батареи равен заряду, который пройдет через лампу за время t, Q=it, t= -Я- =15.=45 ч. / 0,1 Итак, лампа будет гореть в течение 45 ч. 4. Л=20 кГ, В= 5 кГ, С=10 кГ, 0=20. кГ, £= 5 кГ, F= 5 кГ, 0= 10 кГ, Я= 5 кГ, /=10 кГ, А=20 кГ, £=40 кГ, М=40 кГ. 231
5. По второму закону Ньютона сила, дей- ствующая на мяч, равна F=ma. где a=vjt. т. е. F = m —= 0,7- -11—= 630 Н. t 0,02 Итак, футболист бьет по мячу с силой 630 Н. 6. Частота f — это число операций п, вы- полняемых ЭВМ в единицу времени (за 1 с): При п=1 мы получаем соотношение f=l/T, где Т — период, т. е. время, необхо- димое для выполнения одной операции. Отсюда Если на выполнение одной операции тре- буется 1,3 мкс, то это соответствует частоте около 770 кГц. За 1 мин такая ЭВМ может выполнить Гэ^То""* ^46,2 106 операций. 7. ESEL (осел), SOSSE (соус), hELL (светлый). Другие задачи такого рода мы предостав- ляем читателю придумать самостоятельно. Следует иметь в виду, что цифры 0, 1, 3, 4, 5, 7 и 9 на индикаторе перевернутого микро- калькулятора соответствуют латинским бук- вам О, I, Е, h, S, L, G. 8 8. Пусть F — сила, с которой шест давит па плечо рабочего, идущего впереди. Тогда по правилу рычага должно выполняться соотно- шение 232
F-2=9810,8 (Нм), откуда F == ?8-!.'0'8- =392,4 Н = 40 кГ. 2 Итак, на плечо рабочего, идущего сзади, шест давит с силой 588,6 Н, а на плечо ра- бочего, идущего впереди,— с силой 392,4 Н. 9. Грузы А и D поднимутся, грузы В и С опустятся. 10. По закону Архимеда выталкивающая сила Гвыт равна весу вытесненной телом жидкости: ^ВЫТ. = VPg, где р—1 г/см3, £’=9,81 м/с2, V=m/p„. m—GIg, рп=0,2 г/см3. Итак, Г8ЫТ = 35Л 176 н £Рп Рп 0,2 Подъемная сила и вес спасательного круга направлены в противоположные стороны, по- этому подъемная сила F равна их разности, т. е. ^=^выт—<?== 176—35,3» 141 Н. Итак, подъемная сила спасательного кру- га равна 141 Н (или 14,4 кГ). II. Работа W по подъему груза 6 на вы- соту h равна произведению веса груза на вы- соту: W=Gh. а) Й7, = GJii=35 • 0,38 » 13,3 кГм » 130 Дж; 13,3 кГм-6=79,8»783 Дж; б) U72 = G2ft2=10-7,2=72 кГм»706 Дж; в) Wt>W2. 233
12. Объем пирамиды связан с длиной сто- роны а лежащего в основании квадрата н вы- сотой h соотношением V=a'2hl3. По теореме Пифагора Л=У$2—(d/2)2, где s — длина реб- ра пирамиды, a d— длина диагонали основа- ния. Подставляя численные значения, полу- чаем 5^=41.42 и». 3 Из полученного нами полного объема пи- рамиды следует изъять объем бетона. Раз- ность даст нам объем камней, уложенных в пирамиду. По условиям задачи 45% объема пирами- ды приходится на промежутки между камня- ми, поэтому общий обьем камней составляет 41,42-0,55 «22,78 м3. Пусть т — общая масса, V — общий объ- ем и р — плотность камней, уложенных в пи- рамиду. Тогда m = Vp, откуда т=22,78 -2,6 «59,2 т. Итак, в пирамиду уложено около 59,2 т камней. 13. , MOON. , 9552 .VOLVO, .71671 + MEN. Т 902 FIAT, 9542 + CAN. ~t~ 382 MOTOR. 81213 REACH. 10836 Может быть, вам удастся найти другие решения? Три криптарифма со словом РЛДАР. Из (1) и (2) следует, что А=2. Из (3) н А=2 мы заключаем, что Р2Д2Р=1112, а посколь- ку 1112=12321, то Р=1 и Д=3. 234
14. Пусть fi — фокусное расстояние объек- тива, /2--фокусное расстояние окуляра. Г1о условиям задачи /2= 14, fi = 5f 2. откуда fi = 17,5 и f2=3,5. Фокусное расстояние объектива равно 17,5 см, а фокусное расстояние окуляра — 3,5 см. 15. При одном обороте шестерни, вращае- мой педалями, велосипедист проезжает рас- стояние s= (46/|л)л-0,7»6,32 м (4e/i6~~ пере- даточное отношение от шестерни, вращаемой педалями, к шестерне на втулке заднего ко- леса). Для того чтобы проехать расстояние 120 км, велосипедисту придется совершить педалями около 19 000 оборотов. 16. Объем 1 м2 сусального золота равен У=100ХЮ0Х(79о<юо) = |/<) см3, а его масса т= ('/«) -19,3 «2,14 г. Итак, на изготовление I м2 сусального зо- лота необходимо израсходовать около 2,14 г золота. 17. Пусть х—число столбов, уложенных в основание штабеля. По условиям задачи в 6 рядах должно быть уложено всего 105 столбов, причем в каждом ряду на один столб меньше, чем в предыдущем: . х+ (х— 1) + (х—2) + (х—3) + (х—4 j + + (х—5) ==105, откуда х==20. Итак, в основание штабеля следует уло- жить 20 столбов. 235
18. Во всех трех случаях вес чемодана равен произведению его массы на ускорение свободного падения: G = ing. a) 6 = 25-9,81^245 II; б) 6 = 25-9,78^244 Н; в) 6 = 25-9,83^246 II. Итак, на шпроте 45° чемодан весит около 245 Н, па экваторе - около 244 И и на Север- ном полюсе — около 246 Н. НАПРЯЖЕННОЕ РАСПИСАНИЕ 1. В двух 5-х классах («А» и «Б») 4 учи- теля за 2 дня проводят уроки по 8 различным предметам. Условия «а» и «б» и приведенная в условиях задачи выписка из расписания по- зволяют утверждать, что ни один из четырех учителей не может вести следующие пары предметов: физика — физкультура, физика — математика, физика — немецкий язык. Усло- вие «е» позволяет также исключить комбина- ции физика- география и физика — история. Остаются комбинации физика — биология и физика ~~ рисование. Так как по условию «а» комбинация немецкий язык — математика невозможна, по условию «в» фрейлейн Фишер должна преподавать физику. По условию «в» комбинация физика — биология также отпа- дает. Следовательно, фрейлейн Фишер пре- подает физику и рисование. Из условия «г» мы заключаем, что герр Райхельт не может преподавать следующие предметы: физику, немецкий язык, математи- ку, физкультуру и биологию. Так как рисова- ние преподает фрейлейн Фишер, этот предмет также отпадает для герра Райхельта. Следо- 236
вательно, герр Райхельт преподает географию и историю. Выписка из расписания показывает, что комбинации немецкий язык — рисование и немецкий язык- -биология подлежат исклю- чению. Из условия «д» и предыдущих рассужде- ний следует, что фрау Хельмерт преподает математику и биологий). Остальные два пред- мета— немецкий язык и физкультуру — пре- подает герр Вальтер. 2. При передаче 91,8 спортсмен за один оборот педалей проезжает х м, где х= L2|_2!A«7,32 м/об. При заданной частоте вращения педалей (120 об/м) спортсмен проезжает 120-7,32=878,40 м/мин. Пусть расстояние 200 м он проезжает за у мин, тогда и= ——мин» 13,7 с. * 878,4 Вычислим скорость: „=_!_ = J»»- = °-2 360-Р. «52,6 км/ч. t 13,7 13,7 Итак, велосипедист проезжает 200 м при- мерно за 13,7 с, его средняя скорость при этом равна 52,6 км/ч. 3. а) Теорема, б) Единица, в) Определе- ние. г) Ромб, д) Икосаэдр, е) Якоб (Бернул- ли). ж) Инверсия, з) Неравенство, и) Фор- мула. к) Операция, л) Радиан, м) Множе- 237
ство. и) Алгебра, о) Циркуль, п) Иррацио- нальность. р) Итерация. 4. Перевод с французского: одиннадцать-f- -|7девять=двадцать. Учтите, что одиннадцать делится на 11, девять—на 3 и двадцать — на 5. Решение криптарифма: , 4 829 8 976 'Тз 805 5. Пусть г% всего населения города со- ставляют пенсионеры, е% — остальные взрос- лые и А %—дети и подростки. По условиям задачи г=0,4(г4*е), г==0,25(г+е4-А), r~|-e-f-fe = 100, откуда г=25%; «>=37,5%; А=37,5%. Итак, 25% населения города составляют пенсионеры, 37,5% — дети и подростки. 6. Пусть ABCDE — правильный пятиуголь- ник. Разделим его на 2 равновеликие части прямой, параллельной одной из сторон. Первый шаг к решению состоит в том, что- бы из вершины С опустить на сторону АЕ перпендикуляр CF, служащий осью симмет- рии пятиугольника. Перпендикуляр CF заве- домо делит площадь пятиугольника ABCDE на 2 равновеликие части, но не удовлетворяет условиям задачи, так как не параллелен ни одной из сторон. Пусть G — точка пересече- ния прямой, проходящей через точку F па- раллельно стороне АВ, со стороной ВС. Рас- смотрим лежащую внутри пятиугольника тра- 238
пению CEFG (FG\\CE) > и пусть FG=a п CE—b. Диагональ CF трапеции разбивает ее на треугольник CFG площадью Ц и треуголь- ник CEF площадью /2» которые, очевидно, от- носятся между собой, как а : Ь, т. е. /) : h, — a : b. (I) Итак, первоначальная задача сводится те- перь к следующей: разделить трапецию CEFG прямой, параллельной основаниям, на 2 части, площади которых относятся между собой как а : Ь. Пусть Н и К—концы такой прямой и пусть НК~с. Обозначим неизвестное пока расстояние от НК до FG через х, а расстоя- ние от НК до СЕ — через у. Попытаемся найти положение прямой НК аналитически— с помощью введенных нами величин х и у. Площади Ц и /2 частей трапеции HFGK и ЕНКС вычислим по формуле площади тра- пеции: 2Л=(а+с)х, 2/2=(ft+c)y. (2) Из соотношений (1) н (2) следует, что а : Ь= (а+0х : (Ь-\-с)у. Эту пропорцию нетрудно преобразовать в уравнение'Следующего вида: 239
(a+c)bx-~(b + c)ay—0. (3> Уравнение (3) линейно относительно неиз- вестных х и у. Еще одно линейное уравнение мы получим, если применим к трапеции тео- рему о пучке прямых, пересекающих парал- лельные прямые, Непосредственно из теоре- мы следует пропорция х : у— (с—а) : (Ь—с), которую нетрудно преобразовать в уравнение (Ь—с)х+(а—с)у~0. (4) Это уравнение также линейно относитель- но х и у. Уравнения (3) п (4) должны помочь нам найти длину с отрезка НК. Запишем их в виде системы: (а-{-с)Ьх— (Ь+с)ау—0, (5) (Ь—с)х-\-(а—с)у=0. В систему (5), состоящую из 2 уравнений, входят 5 неизвестных величин. Однако это еще не означает, что мы должны отказаться от ее решения и отложить карандаш в сторо- ну. Приглядевшись к системе уравнений вни- мательнее, мы обнаружим, что входящие в нее уравнения содержат только члены, ли- нейные по х и у и не содержат свободных членов, т. е. уравнения системы (5) линейны (относительно х и у) и однородны. Мы впра- ве ожидать, что система уравнений (5) до- пускает ненулевые решения для х и у, так как расстояния между сторонами FG и СЕ трапеции и отрезком НК заведомо отличны от нуля. Для того чтобы лучше понять, как обстоит дело с общим решением системы (5), рас- смотрим сначала 2 численных примера. Систе- ма линейных однородных уравнений Зх+4у=0, (6) 6х4-7^=0 240
допускает только решение х=0, y—Q. Изме- ним один из коэффициентов системы (6) и рассмотрим систему линейных однородных уравнений 3x+4f/=0, (7) 6х-]-8у=0. Значения х=0, y—Q также удовлетворяют уравнениям (7), но это решение уже не един- ственное — существуют и другие пары чисел, например х=4, у=—3, удовлетворяющие уравнениям (7). Система линейных однород- ных уравнений (5) принадлежит к числу систем того же типа, что и уравнения (7). Система уравнений (7) обладает важным свойством: если в какой-то линейной комби- нации входящих в нее уравнений коэффи- циент при х обращается в нуль, то коэффи- циент при у также обращается в нуль, и на- оборот. Математики говорят в таких случаях, что уравнения линейно зависимы. В общем случае система двух линейных однородных уравнений имеет вид п1х+&1У=0, а2х+62у=0. Для того чтобы 'такая система допускала от- личные от нуля решения (х, у), должна вы- полняться пропорция : ci2—~• ^2- Применим теперь наши познания к систе- ме линейных однородных уравнений (5). Как следует из условия (8), система уравнений (5) допускает отличные от нуля решения х и у в том и только в том случае, если : (Ь—с) = (Ь+с)а : (с—а). (9) Эта пропорция содержит только одну неиз- вестную величину с. Выразив из соотношения (9) с через а и &, мы получим выражение, которое послужит ключом к построению от- 16 Ф-734 241
резка НК. Умножая обе части равенства (9) на (Ь—с) (а—с) и перенося все члены в ле- вую часть, преобразуем его к виду (Ь2—с2)а-Ь(а2—с2)Ь=0, откуда __ c—^ab. Итак, длина с отрезка НК, делящего пло- щадь трапеции CEFG в отношении а : Ь, есть среднее геометрическое длин а и b оснований трапеции. Для его построения нужно на от- резке длиной a-f-b как на диаметре полуок- ружности из общего конца двух отрезков вос- ставить перпендикуляр до пересечения с по- луокружностью. Длина этого перпендикуляра как раз и будет равна с. ?. а) 150 см2—0,0150 м2; F 588.6 S 0,0150 «39,24 кПа. Стоя на полу, человек массой 60 кг создает давление около 39,24 кПа (0,4 атм). б) 200-10=2000 см2=0,2 м2; р= да 2,943 кПа- (0,03 атм). Итак, давление, оказываемое лыжниками на снежную поверхность, составляет 2,943 кПа (0,03 атм). 242
в) Давления относятся между собой, как 40: 3. ' 8. Перевод с английского. Задача на раз- резание. Треугольник АВС разрезан на 3 части X, У и Z двумя прямыми, проходящими через середину М стороны АВ параллельно и пер- пендикулярно основанию ВС. Показать, как из этих трех частей можно составить прямоугольник и два различных параллелограмма. Решение показано на рисунке. 9. 18 720--4900= 13 820 26 ’ X 328= 8528 720 +4572 = 5292 10. Мощность сердца мы найдем, вычислив работу, производимую им за 1 с: Р= — = J32_ 15 53 Вт (1 6 кГм/с) t 60 11. Пусть сплав состоит из х долей се- ребра и «/ долей меди (если весь сплав при- нять за 1000 долей). x4-i/=1000. (I) Масса серебра (в г), израсходованного на изготовление медали, составляет 20,9 х/1000, а масса меди — 20,9 у/ЮОО, поэтому серебро занимает объем (в см3) 20,9 х/( 1000-10,5), 15* 243
а медь — 20,9 yl (1000 -8,92), Так как объем медали равен 2,123 см3, 20,9х .___20,91/ _g |2з 1000-10,5 "Г 1000-8,92 Подставляя «/=1000—х из (1) в (2), полу- чаем 20^x4-22^ (ЮОО—х) = 2123, (3) 10,5 8,92 7 откуда х« 624,1 и, следовательно, «/=1000—624,1=375,9. После округления получаем х«625, «/«375. Итак, сплав, из которого изготовлена па- мятная медаль, состоит примерно из 625 до- лей серебра и 375 долей меди. 12. Радиус орбиты Луны /2=384 000 км; радиус орбиты искусственного спутника Земли период обращения Луны вокруг Земли /2=27,33 суток; период обращения спутника Л = 1 сутки. По третьему закону Кеплера квадраты пе- риодов обращения двух спутников относятся как кубы радиусов их орбит (больших полу- осей эллиптических орбит): t\ ; t\=l\ : l\, откуда _ I2-384 0003 * 27,ЗЗ2 Zi «42 320 км. Высота над поверхностью Земли составляет 42 320—6370=35 950 км. Итак, искусственный спутник должен обра- 244
щаться вокруг Земли на высоте около 35 950 км. 13. Горному массиву соответствует поло- са D. ВОКРУГ ЦИРКУЛЯ И ЛИНЕЙКИ 1. Предложенное Гауссом решение исполь- зует следующую теорему: точки пересечения поляры р точки Р относительно эллипса сов- падают с точками, в которых касательные, проведенные из точки Р к эллипсу, касаются * Гаусс воспользовался теоремой проективной гео- метрии, которая изучает свойства фигур, не изменяю- щиеся при так называемых проективных преобразова- ниях — взаимно-однозначных отображениях, сохраняю- щих прямые (точки, расположенные до такого преоб- разования на одной прямой, после преобразования лежат на одной прямой, возможно, отличной от исход- ной). Рассматривая окружность, начерченную на листе бумаги, под углом, отличным от прямого, вы увидите эллипс. При этом все прямые, начерченные на том же листе бумаги, останутся прямыми. Следовательно, раз- 245
2. Ограниченный объем книги вынуждает нас отказаться от чисто геометрического ре- шения задачи, предложенного Галуа. Приведем более простое решение, исполь- зующее теорему косинусов. Так как Z~CDA = = 180°—0 и cos(180°—0) =—cos 0, приравни- вая правые части равенств х2=а2+&2—2ab cos 0 и x2=c2+d2—2cd cos(180°—0) = = c24-d2-j-2cd cos 0, получаем а24-Ь2—2ab cos 0=c24-d24-2cd cos 0, откуда 2 cos 0(ab-|-cd) — a2+&2—c2—d2. Следовательно, Л.2 = а2+ /,2. ЛЙ_1а2+&Г' ab+cd Аналогично 2_bc[a2+d2)+ad(b2+c2) У ad+bc Галуа не мог воспользоваться этим решением, глядывая окружность под любым углом, вы совершае- те проективное преобразование (называемое централь- ным проектированием), и с точки зрения проективной геометрии эллипс не отличается от окружности. Вос- пользуемся последним обстоятельством и определим поляру относительно окружности: полярой точки Р относительно окружности радиуса R с центром в точ- ке О называется прямая, перпендикулярная лучу ОР и проходящая на расстоянии R2IOP от центра О- (Примеч. пер.) 246
так как в его время предложенную задачу требовалось решить чисто геометрически. 3. Доказательство теоремы Наполеона. Прежде всего напомним следующую теорему: четырехугольник можно вписать в окруж- ность в том и только в том случае, если сум- ма двух противолежащих углов его равна 180°. Докажем, что окружности, описанные вокруг трех построенных на сторонах тре- угольника АВС вне его равносторонних тре- угольников, пересекаются в одной точке. Дей- ствительно, окружности, описанные вокруг треугольников ARB и ACQ, помимо вершины А, пересекаются еще в некоторой точке D, лежащей внутри треугольника АВС. Из тео- ремы об описанном четырехугольнике (при- мененной к четырехугольникам ARBD и ADCQ) следует, что Z_R-\-Z_ADB = 180°, ^Q+kADC= 180°. Кроме того, Z-ADB — = Z_ADC= 120° (так как, по предположению, 247
A.R — /LQ — Q00), Таким образом, Z_BDC= = 360°—(A4DS+Z^DC) = 120°, а поскольку Z_P=60°, мы получаем Z-P4-Z.BDC—180°. Используя еще раз теорему о вписанном че- тырехугольнике (в «обратном» направлении), мы заключаем, что вершина D четырехуголь- ника BPCD должна лежать на окружности, описанной вокруг треугольника ВРС. Отрезки прямых, соединяющих центры трех равносто- ронних треугольников (т. е. отрезки О[О2, ОХО3 и O2O3), перпендикулярны общим хор- дам попарно пересекающихся окружностей, т. е. хордам CD, BD и AD. Следовательно, стороны углов О3О\О2 и BDC взаимно пер- пендикулярны, а сами углы либо равны, либо в сумме составляют 180°. Аналогичное утвер- ждение справедливо относительно углов 0{0203 и ADC, О2О3О\ и ADB. Но поскольку сумма внутренних углов треугольника 0{0203 может быть равна только 180°, каждый из перечисленных нами углов равен не 120°, а 60°, что и требовалось доказать. 4. S = 3a&; S = 3a2; S=4a2. 5. Площадь четырехугольника удобнее все- го вычислять «по частям», вычитая из пло- щади прямоугольника со сторонами 17X12 248
площади Sb 32 и S3 прямоугольных треуголь- ников: Sn = 17.12=204, s —513— 65 1 2 2 ’ S2=-L.S3 = 24. Итак, площадь четырехугольника равна 144 квадратным единицам, т. е. четырехуголь- ник с вершинами в указанных точках равно- велик квадрату со стороной 12. 6. Пусть а — длина ребра куба, VK —объ- ем, куба. Каждая из 8 отсеченных от куба частей имеет форму пирамиды ABCD. в осно- вании которой лежит равнобедренный прямо- угольный треугольник АВС с катетами длиной а/3. Высота пирамиды также равна а/3, по- этому ее объем составляет V—— —— а а • а — fl3 ~ з 2 * з ’ з з 162 ' Объем оставшейся части куба равен Итак, объем оставшейся части относится к объему куба, как 77 : 81. 7. Так как х—длина прямоугольника, то его ширина у равна 20—х. По условиям за- дачи л-—(20—х)^2, откуда x^l I. Итак, длина прямоугольника должна быть не меньше 11 м, а его ширина — не больше 9 м. 249
8. Пусть PAQ— угол 63° с вершиной в точке А. Построим рав- носторонний треуголь- ник произвольных раз- меров так, чтобы одна из его вершин совпала с точкой А, а одна из сторон совпала со сто- роной угла AQ. В результате мы получим угол 3°. Удвоив его и сложив углы 6° и 3°, мы получим угол 9°. Построив угсл 9° на стороне равностороннего треугольника по ту же сто- рону, по которую лежит сторона АР исходно- го угла, мы получим угол 21°, заключенный между вертикалью и другой стороной постро- енного нами угла. Он составляет треть данно- го угла. Тем самым мы решили задачу «а». Полученный в процессе построения угол 9° составляет 7? данного угла. Тем самым зада- ча «б» также решена. (Откладывая каждый из этих углов от стороны данного угла внутрь, мы получим разбиение угла на соответствую- щее число равных частей.) 9. Так как каждый из равнобедренных тре- угольников равновелик квадрату, (7г) ha—а2. откуда h—Za. Следо- вательно, расстояние между дальними вер- шинами равнобедрен- ных треугольников, по- строенных на противо- положных сторонах квадрата вне его, равно 250
2a-f-a+2a=5a, т. e. в 5 раз больше длины стороны квадрата. 10. Внутренний угол правильного п-уголь- ника равен а = — 180°. п По условиям задачи —~2 180°-~40°~ ±12L180o, 2п п откуда п— 18. Итак, у первого многоугольника 18, а у второго 36 сторон. Внутренний угол правиль- ного 18-угольника равен 160°, внутренний угол правильного 36-угольника равен 170°. 11. Условимся обозначать каждую точку пересечения двух диагоналей четырьмя бук- вами, стоящими у вершин, которые соединя- ют эти диагонали. Например, точку пересече- ния диагоналей АС и BD обозначим ACBD. Корректна ли предложенная нами система обозначений, т. е. не может ли случиться так, что различные точки получат одинаковые обозначения? Нет, подобное совпадение исключено. Действительно, буквы Л, В, С и D могут встретиться нам только в обозначении точки пересечения диагоналей выпуклого че- тырехугольника ABCD, т. е. диагоналей АС и BD. Кроме того, для обозначения точек пе- ресечения диагоналей нам понадобятся все возможные четверки вершин нашего много- угольника. Например, четверка ACEF соот- ветствует точке пересечения диагоналей $Е и CF. Если мы хотим пересчитать все точки пересечения диагоналей, то для этого доста- 251
точно пересчитать все возможные четверки вершин. Итак, число точек пересечения диа- гоналей совпадает с числом элементов мно- жества всех возмож- ных подмножеств вер- шин, содержащих по 4 элемента. При п==6 мы найдем число та- ких четверок, равное чуть быстрее, если воспользуемся тем, что В общем случае число точек пересечения диагоналей выпуклого n-угольника, удовлет- воряющего условиям задачи, равно / п \ / п \ \ 4 / \л~-4/ 12. Стороны любого невырожденного тре- угольника удовлетворяют неравенству тре- угольника а<Ь+с. Следовательно, а -|- и и "4“ Ь —с, а< -1 (a+b+c) = _Lp, что и требовалось доказать. 13. Пусть а и b — длины сторон прямо- угольника, х — отрезок, на который нужно уменьшить одну из сторон. По условиям зада- чи 252
(a - x) (b+b )=abf \ 4 / (a—x)—b=ab, 4 a—x — — a, 5 1 20 x= —a— —a, 5 100 Итак, для того чтобы площадь прямоуголь- ника не изменилась, другую сторону необхо- димо уменьшить на 20%. ИГРА С ЧИСЛАМИ 1. 4+10+5+9+8+6+7+2+1+8=60. 2. 4 _ 1896 5 2370 3. Магический квад- рат из 8 костей доми- но составлен из 4-4 квадратов — половинок костей, т. е. из 16 кле- ток. Для того чтобы сумма очков была наи- меньшей, необходи мо взять следующие 8 ко- стей: 00, 01, 02, 11, 12, очков равна 19. Заменив увеличим сумму очков до 22, 03, 13. Сумма кость 13 на 23, мы 20 и сможем соста-
вить магический квадрат с постоянной, рав- ной 5. 4. Сектор Л. Каждое число (кроме перво- го) равно удвоенному предыдущему числу ми- нус единица. Вместо вопросительного знака нужно вписать число 145-2—1=289. Сектор В. Каждое число (кроме первого) равно квадрату предыдущего числа минус единица. Вместо вопросительного знака сле- дует вписать число 3968-3968—1=15 745 023. Сектор С. Каждое следующее число втрое больше предыдущего. Вместо вопросительно- го знака следует вписать число 162-3=486. Сектор D. Каждое следующее число на 3 больше предыдущего. Вместо вопроситель- ного знака следует вписать число 13-|-3=16. 5. 4-J-1 __ 1 , __ rt-j-1 n(n-H) п ' (п-|-1)2—1 п+2 п(к-Н) _ »+1 2«2 2п 6. Пробраться внутрь лабиринта, набрав по дороге 500 очков, можно, например, по маршруту 24, 312, 45, 47, 15, 17, 40 пли 60, 21, 38, 47, 15, 17 302. 7. 1 9 . 16.| 7 I 12 5 4 3 8 , 15 10 2 ; 6 11 ,14 8. *! = 777 777 777 777, х2=22 222 222, х3= ==777, х4=666, х5=333, хб=5555, х?=3. 254
9. Так как партнер В должен умножить свое число на 2, его слагаемое всегда четно. Каким будет второе слагаемое — четным или нечетным, зависит от числа, которое А сооб- щает партнеру С: если сумма четна, то А сообщил С четное число, а В — нечетное. Если же сумма- нечетна, то А сообщил С нечетное число, а В — четное. 10. Пусть а и Ь — два натуральных числа, обладающие требуемым свойством. Тогда ab= 10(a+-b), ab=10a+10b, ab—10a= 10b, a(b—10) = 10b, 106 _ 106—100+100 _ 6—10 ' 6—10 _ 10(6—10) + 100_10 100 6—10 +6—10 Величина a принимает натуральные значе- ния лишь при b=ll, 12, 14, 15, 20, 30, 35, 60, ПО. Итак, требуемым свойством обладают толь- ко пары чисел (11, ПО), (12, 60), (14, 35), (15, 30), (20, 20), (30, 15), (35, 14), (60, 12), (ПО, П). 11. Пусть х и у — неизвестные числа, при- чем х<у. Тогда (х+у) + (х—у)+ху+ — = 2х+хг/+ — =243, 2ху+ху2+х=243у, х(2у+у2+1)=243у, х(у+1)2=243у, 255
r_ 243у _ 35г/ (W-l)2 («Ж)2 Поскольку х—натуральное число, знаме- натель (i/4-l)2 должен быть делителем числа 243, а так как 243=З5, (у+1)2 может быть только степенью тройки, совпадающей с квад- ратом некоторого натурального числа. Следо- вательно, знаменатель (t/4-l)2 может быть ра- вен либо З2, либо 34= 92, т. е. либо у—2, либо У=8. При у=2 получаем х.= ^1=54, при у—8 243-8 _„4 Хг----91--24’ Итак, числами, над которыми производи- лись операции, были либо 2 и 54, либо 8 и 24. 12. 77:77=1, 7—(7+7): 7=5 7: 7+7: 7=2, (7-7—7): 7=6, (7+7+7) : 7=3, (7—7) : 7+7=7, 77: 7—7=4, 7+ (7+7) : 7=9, (77—7) :7=10. ОГОНЬ МАТЕМАТИЧЕСКИХ ОЛИМПИАД 1. Так как фавориты А, В и С заняли пер- вые 3 места, возможны только следующие 6 исходов заездов (в колонке справа указаны утверждения, которым противоречат эти ис- ходы) : 256
a) ABC — б) ACB (4) в) ВАС (1) г) ВСА (1), (4) Д) CAB (2) е) СВА (3) В 5 случаях предполагаемая очередность прихода фаворитов к финишу противоречит по крайней мере одному утверждению. Всем условиям задачи удовлетворяет распределе- ние мест, при котором фаворит А пришел пер- вым, В вторым и С третьим. 2. Дзета может воспользоваться, например, тем, что 5-54-5=30, (1) 5-(54-5: 5) =30 (2) Так как в левую часть равенства (1) циф- ра 5 входит 3 раза, утверждение Дзета верно при любом нечетном п, поскольку в левую часть равенства (1) можно при любом п>3 включить (п—'3)/2 слагаемых (5—5). Утверждение Дзета верно и при любом четном п, так как в левую часть равенства (2), которая содержит 4 пятерки, при любом п>4 можно включить (п—4)/2 слагаемых (5—5). 17 Ф-73 257
4. Пусть хь х2, х3, х4, Хв— номера букв, вхо- дящих в фамилию великого математика. По условиям задачи х1+х2=40, откуда х2==40—хь X;+хз=42, откуда Хз=42—хь Xi+x4==36, откуда х4=36—хь Xi-j-*5=47, откуда х5=47—хь (1) (2) Подставляя (1) в (2), получаем х1+-(40-х1) + (42—X]) + (36—Xj) 4- (47—Xi) = = 75, Зх!=90, откуда xi=30. Следовательно, х2=10, Хз=12, х4=6, х5=17. Номер 30 имеет буква Э, номер 10 — буква И, номер 12 — буква Л, номер 6 — буква Е, номер 17 — буква Р. Фамилия великого математика Эйлер. 5. Числа а=1 и Ь=—2 отличны от 0. Они удовлетворяют неравенству а>Ь и (поскольку |1| = 1, |—2|=2) неравенству |а|<|&|. Так как число а неотрицательно, то ут- верждения, высказанные А и В, ложны. Если а и b — два отличных от нуля дей- ствительных числа, таких, что аи |а|<|Ь|, то число b отрицательно. Действительно, если бы число b было неотрицательным, то выпол- нялось бы неравенство а>Ь>0 и, следова- тельно, |а|=а>Ь=|Ь|, что противоречит условию | а | < | Ь |. Тем самым доказано, что утверждение С верно, а утверждение D ложно. 258
6. Пусть Г1, г2, гз и г4— радиусы концентри- ческих кругов от центрального до самого большого. Круги .содержат соответственно 1, 3, 7 и 15 равновеликих частей. Так как площадь каждого круга равна пн (i=l, 2, 3, 4), из условий задачи следует, что площади кругов относятся между собой, как nri :яг1 8лгз : лг4==1 : 3 : 7^ 15,_откуда (так как г/>0) и: г2 :г3: г4=1 : /3 : VI :/15. 7. Пусть D, Е nF — основания перпенди- куляров, опущенных из центра О на стороны АВ, ВС и СА прямоугольного треугольника ABC, Z_4CB=90°, АВ=с, OD=OE=OF=p. Поскольку прямоугольные треугольники ODB и ОЕВ, ODA и OFA попарно конгруэнтны, С суммарная площадь всех четырех треугольни- ков равна удвоенной площади треугольника АОВ, т. е. равна ср. Четырехугольник OECF— квадрат со стороной р, поэтому площадь его равна р2. Следовательно, площадь треугольника АВС равна ср+р2. 8. Заведомо обеспечить себе выигрыш иг- рок может только в том случае, если сущест- вует стратегия, которая гарантирует выигрыш при любых обстоятельствах. Именно такая стратегия существует и в игре со спичками. Предположим, что после очередного хода од- ного из игроков, например игрока А, число 16* 259
спичек, оставшихся в коробке, будет делиться на 11. Сколько бы спичек ни взял, делая сле- дующий ход, его противник — игрок В (йо правилам игры игрок может взять от 1 до 10 спичек), игрок А всегда может дополнить это число до 11. Следовательно, перед любым ходом игрока В число спичек в коробке будет делиться на 11. После конечного числа ходов наступит момент, когда перед очередным хо- дом игрока В в коробке останется ровно 11 спичек, из которых он может взять не менее 1 и не более 10. Следовательно, после хода игрока В в спичечном коробке непременно ос- танется не менее 1 и не более 10 спичек, и иг- рок А сможет забрать их, делая свой послед- ний ход. В рассматриваемом нами случае (коробок перед началом игры вмещает 150 спичек) си- туация складывается следующим образом: иг- рок А всегда может обеспечить себе выигрыш, взяв первым ходом 7 спичек (в коробке оста- нется 143 спички, а число 143 делится на 11). Игрок В не может гарантировать себе выигрыш. Он может выиграть только в том случае, если игрок А нарушит стратегию. 9. Назовем отрезок, соединяющий две точ- ки, «двуцветным», если он соединяет голубую точку с черной или черную точку с голубой, и «одноцветным», если он соединяет 2 голу- бые или 2 черные точки. Точка является не- обыкновенной в том и только в том случае, если из нее двуцветных отрезков выходит боль- ше. Если необыкновенную точку перекрасить, то из нее будет выходить больше одноцветных отрезков, чем двуцветных, а все остальные от- резки, не выходящие из перекрашенной точки, останутся такими, какими были. Следова- 260
тельно, какую бы точку не выбрать, перекра- сив ее, мы уменьшим число двуцветных от- резков. Если бы после конечного числа шагов исходное множество точек не перестало со- держать необыкновенные точки, то это озна- чало бы, что существует множество точек, в котором возможно бесконечное число перекра- шиваний. Но тогда должна была бы сущест- вовать бесконечная строго убывающая после- довательность натуральных чисел (каждое из которых соответствовало бы числу двуцветных отрезков), что невозможно. Полученное про- тиворечие доказывает утверждение задачи. 10. Если бы первоначально в корзине было х слив, то первый жених получил бы (х/2-|-1) слив, и в корзине осталось бы х—(х/2-}-1) = =х/2—1 слив. Следовательно, второй жених получил бы (х/2—1)/2+1=х/4+1/2 слив и в корзине осталось бы (х/2—1)—(х/4-р/2) = =х/4—3/2 слив. Но тогда третий жених полу- чил бы (х/4—3/2)/2+3=x/8-f-9/4 слив. По- скольку после этого в корзине не осталось бы ни одной сливы, мы получаем уравнение (х/4—3/2)—(х/8+9/4)=0, откуда х/8=15/4, х=30. Итак, в корзине было 30 слив. И. Шестизначный телефонный номер в де- сятичной системе счисления соответствует чис- лу г=аЮ5+&104-|-с103+</1024-е10+/, где а, Ь, с, d, е, f — натуральные числа. По ус- ловию задачи О^Ь, с, d, е, /<а9, цифры Ь, с, d, е, f отличны от 1 и 2^а^9. Если бы a-j-b^lO, то сумма двух первых цифр номе- ра телефона школы начиналась бы с 1. Сле- довательно, а+&=с^9. 261
Аналогичным образом можно доказать, что 6+c=dC9, c+d=e^9, d+e=f^9. Пред- положим, что а^4. Тогда с=4, d=4, мы при- шли бы к е=8 и неравенству d-]-e=f^l2, противоречащему условию f^9. Следователь- но, а^З, т. е. либо а=1, либо а=3. Предпо- ложим, что Ь>0. Тогда должно было бы вы- полняться неравенство Ь^2, откуда с^4, </^6 и c-f-d=e^lO, что невозможно. Следо- вательно, Ь=0. Так как номер дома, где жи- вет Гюнтер, делится на 3, а=3. Значит, Гюнтер живет в доме номер 30. Телефон его школы 30-33-69. 12. Предположим, что торговец первона- чально запросил за голову скота а грошей. Поторговавшись, крестьянин сбил цену на а(а/100) грошей и уплатил торговцу д2 «-й0=21 грош. Следовательно, 100а—а2=2100, а2— 100а 4-2100=0. Итак, либо а=70, либо а=30. Если а=30, то у крестьянина было бы 90 грошей, но 90 не делится на 21, поэтому а=30 отпадает. Итак, условиям задачи может соответствовать только а=70. Поторговавшись, крестьянин сбил перво- начальную цену (70 грошей за голову скота) на 70%, т. е. на 49 грошей. Новая цена соста- вила 21 грош за голову скота. Стоимость всей покупки составила 210 грошей. На эти деньги по старой цене можно было бы купить 3 голо- вы. Так как по условиям задачи крестьянин израсходовал на покупку все деньги (210 де- лится на 21), то он купил 10 голов скота. 262
ПО РАЗНЫМ СТРАНАМ 1. Встреча между шахматистами, занявши- ми первое и второе место, закончилась вничью, так как ни один из них не проиграл ни одной партии. Следовательно, шахматист, занявший первое место, набрал не более 8’/2, а облада- тель второго места набрал не более 8 очков. Шахматисты, занявшие 4 последних места, сыграли между собой ровно 6 партий, поэто- му набрали не менее 6 очков. Следовательно, шахматист, занявший четвертое место, на- брал не менее 6 очков, а шахматист, занявший третье место,— не менее 6V2 очка. Но шахма- тист, занявший третье место, не мог набрать 7 или более очков, так как в противном слу- чае шахматисты, занявшие первое и второе место, набрали бы вместе не менее 17 очков, что невозможно. Следовательно, шахматист, занявший третье место, набрал б’/г очка, а шахматист, занявший четвертое место,— ров- но 6 очков. Шахматисты, занявшие первое и второе место, набрали в сумме I6V2 очка. Это возможно только в том случае, если занявший первое место набрал 8V2 очка, а занявший второе набрал 8 очков. Итак, все 10 участни- ков шахматного турнира набрали в сумме 45 очков (так как всего было сыграно 45 пар- тий), из которых шахматисты, занявшие 6 последних мест, получили 45—(8|/2-Ь8+61/24- -|-6) = 16 очков, а шахматисты, занявшие 4 последних места,— 6 очков. Следовательно, шахматисты, занявшие пятое и шестое места, набрали в сумме 10 очков, что возможно толь- ко в том случае, если они получили соответ- ственно 5*/г и 4*/г очка. 263
Итак, шахматисты, занявшие первые 6 мест, набрали по 8’/2, 8, б’/г. 5’/г и 4’/2 очка. Примечание. Все условия задачи бы- ли бы выполнены и в том случае, если бы пар- тии между шахматистами, занявшими первое и второе, третье и пятое, второе и шестое мес- та, закончились вничью, а во всех остальных партиях победу одержали шахматисты, заняв- шие более высокое место, чем их партнеры. 2. Во всех 3 номерах программы—в прыж- ках в длину (Д), в высоту (В) и с шестом (Ш)—приняли участие 6 спортсменов. 3. Как видно из схем, существуют 3 типа вариантов прокладки железной доррги. 1) В первом варианте одному из городов отводится роль железнодорожного узла, в ко- тором сходятся 4 линии. В зависимости от то- го, какой из городов выбран, проектировщики могут предложить 5 проектов такого типа. 2) Во втором случае одному из городов отводится роль железнодорожного узла, в ко- тором сходятся 3 линии. С тремя остальными городами узел можно связать железнодорож- ными линиями 4 различными способами (чис- ло сочетаний из 4 предметов по три). В каж- дом варианте пятый город можно связать с одним из трех городов, который соединен ли- 264
нией с первым городом, поэтому в рассматри- ваемом случае существует 4-3=12 вариантов прокладки железных дорог. Поскольку в ка- честве железнодорожного узла можно выбрать любой из 5 городов, то всего существует 5-12=60 вариантов железнодорожных сетей второго типа. 3) В третьем случае никакие 3 железно- дорожные линии не сходятся ни в одном из городов. Любой перестановке из 5 городов соответствует железнодорожная сеть, т. е. ва- риантов прокладки сети существует столько же, сколько перестановок из 5, а именно: 51=120. При этом одной и той же последова- тельности городов соответствуют 2 переста- новки (одна перестановка перечисляет города «от начала», другая—«от конца»), поэтому различных железнодорожных сетей вдвое меньше, чем перестановок, т. е. всего их 120: 2=60. Итак, общее число вариантов прокладки железнодорожной сети достигает 54-604-60= = 125. 4. Обозначим фигурки латинскими буква- ми а, Ь, с, d, е, f. Для того чтобы перестроить фигурки, достаточно совершить 15 шагов и прыжков:, с—d е—с f—е d—f f—h b—d d—f a—b b—d c—b e—c g—e f—g d—f e—d 265
5. Приводимая ниже стратегия принадле- жит швейцарцу Г.—К. Ленхарду. Он показал, что игра доводится до успеш- ного завершения, если ни сумма, ни разность цифр, образующих номер начального поля, не делится на 3. Этому условию удовлетворя- ют поля с номерами 02, 04, 13, 20, 23, 26, 31, 32, 34, 35, 40, 43, 46, 53, 62 и 64. В нашем примере (свободно от фишки поле 43) игра проводится следующим обра- зом. Фишка 45 переставляется с поля 45 на поле 43 (фишка 44 снимается). Фишка 24 переставляется с поля 24 на поле 44 (фишка 34 снимается). Фишка 43 переставляется с поля 43 на поле 45 (фишка 44 снимается). Затем следуют ходы: 46—44, 43—63, 32—52, 03—23, 32—12, 36—34, 51—53, 62—42, 12—32, 02—22, 26—24, 63—43, 23—25, 42—22, 11—31, 63—43, 34—54, 15—35, 40—42, 23—21, 55—53, 64—44, 04—24, 30—32, 20—22,... 6. Первый криптарифм в переводе с анг- лийского означает «Мешай забаву с матема- тикой», второй в переводе с французского гласит «Вперед и вперед мы идем». Криптарифмы допускают, например, сле- дующие решения: , 138 , 91650 , 91670 + 920 Т 91650 Т 91670 + 407 Т 4 670 Т 4 650 --Й65" + 4 670 4 650 192 640 192 640 266
7. День 1-й 2-й 3-й 4-й С утра К вечеру Солнце Дождь Солнце Дождь Солнце Дождь Солнце Дождь День 5-й 6-й 7-й 8-й 9-й С утра К вечеру Солнце Солнце Солнце Солнце Дождь Солнце Дождь Солнце Дождь Солнце f 6+5+7 =9 Каникулы Мбонго продолжались 9 дней. 8. Ошибка заключается в неэквивалент- ном преобразовании при извлечении квадрат- ного корня: 9. Предположим, что в доме живут х се- мей, имеющих У1<У2<Уз<-<Ух-1 <Ух№- тей. По условиям задачи Следовательно, 267
ух>1+2+3+...+ (х-1)= Х(У} , и общее число детей удовлетворяет неравен- ству у>2 А**-.1.) =х(х-1). Если во всех семьях вместе имеется и мальчиков и v девочек, то по условиям задачи y>2x>u>v>x, откуда 4x>u-j-v=y>2x. Кроме того, и^2х—1 и, следовательно, v^2x—2, y=u-t-v^4x—3, поэтому х(х—1) <$/< (4х—3), откуда х(х— 1)—(4х—3)<0. Преобразуя последнее неравенство, получаем х2—5x4-3 < О, (2х—5)2< 13<16, —4<2х—5 <4-4, 1<2х<9, 1<х^4. I) При х=2 мы получили бы 2х=4> >и>х=2 и 2х=4>о>х=2, откуда вопре- ки условиям задачи следовало бы, что u=v=3. 268
2) При х=3 мы получили бы 2х=6>и> >х=3 и 2х=6>и>х=3. Так как u>v, то и=5, о=4. Следовательно, y=u+v=9= = 1+24-6=1+3+5. Условиям задачи, согласно которым каж- дая девочка имеет по крайней мере одного брата и не более одной сестры, удается удов- летворить только в том случае, если в доме живут три семьи, причем в первой семье один ребенок, во второй семье трое детей, в третьей семье пятеро детей, при этом в первой и во второй семье по одному сы- ну, а в третьей — три сына, в первой семье 0 дочерей, а во второй и в третьей семье — по две дочери. Это единственное решение задачи, так как при х=4 мы получили бы х(х—1) = 12<у<4х—3= 13, откуда следовало бы, что у= 13=1+2+3+7. В первой семье не может быть дочерей, во второй семье может быть не более одной до- чери и в третьей семье — не более двух доче- рей. Следовательно, оС0+1+2+2=5, а так как о>х=4, мы получаем и=5, и=у—о=13—5=8=2х, что противоречит условию 2х>«. 269
10. Пусть х — число, задуманное Уве, а п— результат вычислений. Тогда [(х-5+2)-4+3]-5=п, 100x4-55= п. Следовательно, задуманное число мы полу- чили, отбросив две последние цифры (55) по- лученного результата. Пример: если в резуль- тате вычислений получилось число 1755, то было задумано число 17 (т. е. х=17).
Йоханнес Леман УВЛЕКАТЕЛЬНАЯ МАТЕМАТИКА Главный отраслевой редактор Л. Л. Ерлыкин Редактор Г. Г. Карвовский Мл. редактор Я. А. Сергеева Худож. редактор М. А. Бабичева Художник В. А. Провалов Техн, редактор С. А. Птицына Корректор Н. Д. Мелешкина ИБ 6425 Сдано в набор 06.02.85. Под* писано к печати 18.07.85. Фор* мат бумаги 75Х90‘/з2. Бумага офсетная № 1. Гарнитура ли- тературная. Печать офсет. Усл. печ. л. 10,62. Усл. кр.-отт. 21,55. Уч.-изд. л. 8,82. Тираж 100000 экз. Заказ Ф-73. Цена 50 коп. Издательство «Знание». 101835, ГСП, Москва, Центр, проезд Серова, д. 4. Индекс заказа 857716. Издательство Тат. ОК КПСС, г. Казань, ул. Декабристов, 2.
Издательство «Знание» готовит к выпуску в 1985 г.: Г. И. Мишкевич ДОКТОР ЗАНИМАТЕЛЬНЫХ НАУК (жизнь и творчество Я. И. Перельмана). С именем Я. И. Перельмана, этого за- мечательного человека, связано возник- новение и развитие особого — занима- тельного — жанра научной популяриза- ции основ знаний. Автор более ста книг и брошюр, он обладал редким даром захва- тывающе интересно рассказывать о сухих научных истинах, возбуждать жгучее лю- бопытство и любознательность — первые ступени самостоятельной работы ума. При этом он не превращал науку в заба- ву и развлечение, а, наоборот, живость и занимательность изложения ставил на службу уяснения научных истин. Сочета- ние строгой научной достоверности с за- нимательностью и нетривиальной формой подачи материла — вот секрет неизмен- ного успеха книг Перельмана.
50 коп Многие считают занимательные задачи средством для приятного времяпровож- дения, отдыха, но, если вдуматься, то становится ясной их гораздо более важ- ная роль. Несомненно, что именно зани- мательные задачи являются одним из самых мощных инструментов развития человеческого интеллекта. Если человеку в течение жизни приходится, скажем, десяток раз оказаться в затруднитель- ном положении, выход из которого мож- но найти с помощью логических рас- суждений, то задачи предоставляют ему такую возможность сотни раз уже в дет- стве и юности — именно тогда, когда формируется его интеллект.